You are on page 1of 88

JAWAPAN

Bab 1 Sukatan Membulat 5.


Radian Darjah
Zon Formatif 1.1
0.2
0.2 × 180° = 11.5°
π
1. (a) 2π = 2 × 180° 1π 0.5π × 180° = 90°
3 3 2 π
= 120° 1.65 × 180° = 94.5°
(b) 1.5π = 1.5 × 180° 1.65
π
= 270° 14π 9 × 180° = 324.0°
5 5
(c) 2.5 rad = 2.5 × 180°
π 2π 2 × 180° = 360°
= 143.24°
(d) 3 rad = 3 × 180°
π Zon Formatif 1.2
= 171.89°
2. (a) 45° = 45 × π
180
180 (
1. (a) Panjang lengkok = 75 × π × 2.6 )
= 3.403 cm
= π
4 (b) Panjang lengkok = 0.85 × 13
= 11.05 cm
(b) 150° = 150 × π
180 (c) Panjang lengkok = 0.17π × 10
= 5π = 1.7π cm
6
(c) 270° = 270 ×π 180°(
(d) Panjang lengkok = 30.5° × π × 5.5 )
180 = 2.93 cm
= 3π 2. (a) 2.5 = q(10)
2
q = 0.25 rad
(d) 315° = 315 × π (b) 12π = q(10)
180
q = 1.2π rad
= 7π 6π
4 (d) = q(10)
270 5
3. (a) 63° = ×π q = 3π rad
180 25
= 1.1 rad
3. (a) 3 π = q(4)
(b) 220° = 220 × π 4
180
q= 3 π
= 3.8 rad 16
(c) 355° = 355 × π
180 (
(b) P = 150 × π × 3.5
180 )
= 6.2 rad = 9.16 cm
(d) 178° = 178 × π (c) 12.4 = q(6)
180
= 3.1 rad q = 12.4
6
4. = 2.067 rad
Darjah Radian (d) 18 = 0.75j
j = 24 cm
178 π = π
45° 4. OP : PS = 4 : 1
180 4
60 π = π PS = 5 cm, OP = 5 × 4
60°
180 3 = 20 cm
135°
135 π = 3π
180 4
PQ = (
28
180 )
× π × 20
225 π = 5π = 9.77 cm
225°
180 4
330°
330 π = 11π (
RS = 28 × π × 25
180 )
180 6 = 12.217 cm

Jw01 Full SpotlightA+ MateTambahan Tg5.indd 1 26/10/2022 3:40 PM


Matematik Tambahan Jawapan

Nisbah PQ : RS = 9.77 : 12.217 (b) PQ = 1.2(5)


= 1 : 1.25 = 6 cm
=4:5 (c) OT = ! 52 + 12.862
5. OA : OB : BC = 1 : 2 : 3 = 13.798 cm
Panjang OA = 20 cm Perimeter kawasan berlorek
OB = 2 × 2 = 4 cm = PQ + QT + PT
OC = 3 × 2 = 6 cm = 6 + (13.798 – 5) + 12.86
(a) CD = q(OC) = 27.66 cm
8 = q(6)
q = 4 rad Zon Formatif 1.3
3
(b) AF = 2 4
3 () (
1. (a) Luas = 1 225 × π (8.6)2
2 180 )
= 8 cm = 145.22 cm2
3
4 (b) Luas = 1 (0.65)(25)2
(c) BE = (4) 2
3 = 203.125 mm2
= 16 cm
3 (
(c) Luas = 1 7 × π (16)2
2 8 )
(d) Perimeter ABEF = AF + AB + EF + BE
= 351.86 cm2
= 8 + 16 + 2 + 2

3 3
= 12 cm
(
(d) Luas = 1 150 × π (0.8)2
2 180 )

= 0.838 m2
6. (a) AC = ! 122 + 52
= 13 cm 2. (a) 1.25π = 1 (q)(42)
2

180 (
(b) BC = 90 × π × 12 ) q =
1.25π(2)
16
= 6π cm = 0.49 rad
(c) Perimeter berlorek = AB + AC + BC
= 7 + 13 + 6π (
(b) L = 1 280 × π (5)2
2 180 )
= (20 + 6π) cm = 61.087 cm2
7. (a) QR 2 = 152 + 152 – 2(15)(15) kos 50° (c) 25 = 1 q(5)2
= 160.746 2
QR = 12.68 q = 2 rad
Jejari, AR = 12.68 (d) 18 = 1 (1.2)j2
2 2
= 6.34 cm j2 = 30
Panjang lengkok QR = π(6.34) j = 5.48 cm
= 19.92 cm 3. (a) Panjang WUX = 30.26 – 8 – 8
(b) Perimeter seluruh rajah = QR + PQ + PR = 14.26 cm
= 19.92 + 15 + 15 q(8) = 14.26
= 49.92 cm q = 1.7825 rad
8. (a) kos 1.1 = OP (b) Luas sektor = 1 (1.7825)(82)
OQ 2
= 57.04 cm2
kos 1.1 = 15
OQ 4. ON = ! 102 – 7.52
OQ = 33.07 cm = 6.614 cm
QR = 33.07 – 15 Luas ∆AON = Luas sektor
= 18.07 cm (Tertunjuk) 1 (6.614)(7.5) = 1 q(102)
(b) PR = 1.1(15) 2 2
= 16.5 cm q = 0.49605 × 180°
π
PQ = ! 33.072 – 152
q = 28° 25
= 29.47 cm
Perimeter kawasan berlorek 5. (a) Panjang AB = 10(1.76)
= 29.47 + 16.5 + 18.07 = 17.6 cm
= 64.04 cm (b) Luas sektor AOB = 1 (1.76)(102)
2
9. (a) tan 1.2 = PT = 88 cm2
5
PT = 12.86 cm

Jw01 Full SpotlightA+ MateTambahan Tg5.indd 2 26/10/2022 3:40 PM


Jawapan Matematik Tambahan

(c) Luas berlorek = Luas sektor – Luas ∆QOP


(
(b) Luas sektor ABC = 1 45 π (8.485) )

2 [
= 88 – 1 (5)(5) sin 1.76 ]
2 180
= 28.27 cm2
= 88 – 12.277 (c) Luas berlorek
= 75.72 cm2 = Luas semibulatan – Luas sektor ABC
6. (a) ˙AOB = π – 0.9
= 2.242 rad 2[ ]
= 1 π(62) – 28.27
= 28.28 cm2
Luas sektor AOB = 1 (2.242)(82) ≈ 29 cm2
2
= 71.74 cm2 2. (a)
(b) AB = OB + OC + BC A B
(π – q)(8) = 8 + 8 + 8q
8π – 8q = 16 + 8q
16q = 8π – 16
q = 8π – 16 10 cm
1.0 rad
16
q = π – 1
2
= 0.571 rad O
8. (a) (i) q = 60°
Dengan menggunakan petua kosinus
(ii) q = 60 π AB2 = 102 + 102 – 2(10)(10) kos (1 rad)
180
= 91.9395
= 1 π AB = 9.59 cm
3
(b) Panjang AB = 9 π
3 () (b)
A 9.59 cm B
= 3π cm
(c) Sudut major = 2π – π
3
8 cm
= 5π θ
3
Luas sektor major = 1 5π (92)
2 3 ( ) P
= 212.06 cm2
kos q = 8 + 8 – 9.59
22 2

9. (a) Katakan ˙AOB = q 2(8)(8)


6q = 18 = 0.2815
q=3 q = 1.285 rad
˙BOC = π – 3 (c) Perimeter kawasan berlorek
= 0.142 rad = Panjang AQB + Panjang ARB
(b) Luas sektor BOC = 1 (0.142)(92) = 1(10) + 1.285(8)
2 = 20.28 cm
= 2.556 cm2 (d) Luas perentas AQB
= 1 (1)(102) – 1 (10)(10) sin 1
Zon Formatif 1.4 2 2
= 7.926 cm2
1. (a) Luas perentas ARB
B = 1 (1.785)(82) – 1 (8)(8) sin 1.255
2 2
= 10.418 cm2
Luas kawasan berlorek
= 10.418 – 7.926
45° = 2.49 cm2
D A O 6 cm C 3. (a) tan a = 5
15
BC = ! 62 + 62 a = 0.322 rad
= 8.485 cm q = π – 2a

(
Panjang AB = 45 π × 8.485
180 ) = π – 2(0.322)
= 2.498 rad
= 6.66 cm

Jw01 Full SpotlightA+ MateTambahan Tg5.indd 3 26/10/2022 3:40 PM


Matematik Tambahan Jawapan

(b) Jejari = ! 152 + 52 6. (a) AC = OC + OB + BC


= 15.811 cm qj = j + j + (π – q)j
Luas permukaan qj = 2j + πj – qj
2qj = 2j + πj
2 2 [
= 1 (2.498)(15.8112) + 2 1 × 5 × 15 ] q=2+π
= 387.2 cm2 2
(b) j = 6 cm
4. (a) j = Panjang sisi tepi kon
= ! 122 + 52 Luas sektor OCB = 1 (π – q)(62)
2
= 13 cm
(b) Panjang lengkok
2 (
= 1 π – 2 + π (62)
2 )
= Perimeter bulatan tapak kon = 9(π – 2)
= 2π(5) = (9π – 18) cm2
= 10π 7. Diberi panjang lengkok MN = qj
= 31.42 cm
Perimeter MON = 38.5
(c) 31.42 = 13q
j + j + qj = 38.5
q = 2.42 rad
2j + 8.22 = 38.5
j = 15.14 cm
Zon Sumatif Gantikan j = 15.14 cm,
q(15.14) = 8.22
Kertas 1
q = 8.22
1. (a) 60° (segi tiga sama sisi) 15.14
= 0.543 rad
(b) 60 × π = π rad
180 3 Luas sektor MON = 1 (0.543)(15.142)
2. Panjang lengkok = 30 2
q(9) = 30 = 62.23 cm2

1
q = 3.33 rad
(
8. Panjang lengkok CD = 60 π (10)
180 )
Luas sektor = (3.33)(92)
2 = 10π
= 134.87 3
Perimeter berlorek
≈ 135 cm2 = Perentas CD + Lengkok CD
3. (a) Didapati 24 cm ≈ Perimeter sektor
qj + j + j = 24
(
= 10 + 10π cm
3 )
qj = 24 – 2j 9. j = 2! 3 , PQR = Segi tiga sama sisi
= 60°
q = 24 – 2
j = π rad
3
(b) Luas sektor = 1 qj2 Luas kawasana berlorek
2
= 2[Luas sektor – Luas ∆PQR]
(
= 1 24 – 2 j2 )

2 j
= 12j – j2 (Tertunjuk) [()
= 2 1 π (2! 3 ) – 1 (2! 3 )(2! 3 ) sin π
2 3
2

2 3 ]
4. Luas sektor = 9
1 (0.5)j2 = 9 6 [
= 2 π (12) – 6 ( )]
!3
2
2 = (4π – 6! 3 ) cm2 (Terbukti)
j=6
Panjang lengkok = 0.5(6) 10. (a) kos 40° = OP
= 3 cm OQ

5. (a) OB = 8 cm, OC = 1 × 8 kos 40° = 9


2 OQ
= 20 cm OQ = 11.75 cm
Panjang lengkok CD = 0.78(20) QR = 11.75 – 9
= 15.6 cm = 2.75 cm
(b) Luas kawasan berlorek ABCD (b) QP = ! 11.752 – 92
= 7.554 cm
= 1 (0.78)(202) – 1 (0.78)(82)
2 2 Lengkok PR = 0.698(9)
= 131.04 cm2 = 6.282 cm
Perimeter kawasan berlorek
= 6.282 + 7.554 + 2.75
= 16.59 cm

Jw01 Full SpotlightA+ MateTambahan Tg5.indd 4 26/10/2022 3:40 PM


Jawapan Matematik Tambahan

(c) Luas kawasan berlorek (c) Luas ∆POB = 1 (4)(4) sin 1.35
= Luas ∆POQ – Luas sektor POR 2
= 7.806 cm2
= 1 (9)(7.55) – 1 (0.698)(92)
2 2 Luas sektor POB = 1 (1.35)(42)
= 5.71 cm2 2
= 10.8 cm2
11. (a) OF = ! 102 – 62 Perentas ∆POB daripada luas sektor minor
= 8 cm BOP
Panjang EF = 10 – 8
= 2 cm = 7.806 × 100%
10.8
sin ˙DOF = 6 = 72.28%
10
˙DOF = 0.6435 rad 14. A C
Perimeter kawasan berlorek
= DF + FE + ED
= 6 + 2 + 0.6435(10) 42 cm
= 14.44 cm 55 cm
(b) Luas kawasan berlorek α

= 1 (0.6435)(102) – 1 (8)(6)
2 2 O
= 8.18 cm2
12. Luas ∆OAT = 60 (a) 12° = 12 × π
180
1 (OA)(AT) = 60 = 0.209 rad
2
1 (10)(AT) = 60 Panjang lengkok ayunan = 13.4 cm
2 0.209j = 13.4
AT = 12 cm j = 64.11
tan ˙AOB = 12 (b) Luas rantau ayunan = 1 (0.209)(64.112)
10 2
˙AOB = 0.876 rad = 429.5 cm2
(a) Panjang lengkok AB = 0.876(10) 15. Diberi OP : PA = 2 : 1
= 8.76 cm OP = 2
(b) Luas sektor = 1 (0.876)(102) PA 1
2 OP = 2
= 43.8 cm2 PA 1
(c) Perimeter rantau ABT PA = 5 cm
= AB + AT + BT º PA = QB = 5 cm
= 8.76 + 12 + (! 122 + 102 – 1)
= 8.76 + 12 + 5.62 30° = 30 π rad
180
= 26.38 cm π
=
13. P 6

5 cm
()
Luas OAB = 1 π (152)
2 6
= 225 π
12
A B
O 4 cm ()
2 6 ()
Luas kawasan berlorek = 1 π (152) – 1 π (102)
2 6
= 125 π
12
Nisbah
= luas kawasan berlorek : luas sektor OAB
(a) Katakan sudut PAB = q
125 π : 225 π
sin q = 5 12 12
8
5:9
q = 0.675 rad
(b) ˙AOP = π – 2(0.675) Kertas 2
= 1.792 rad 1. (a) Luas kawasan berlorek
˙POB = π – 1.792
= 1 (2)(82) – 1 (8)(8) sin 2
= 1.35 rad 2 2
Panjang lengkok PB = 1.35(4) = 64 – 29.098
= 5.4 cm = 34.9 cm2

Jw01 Full SpotlightA+ MateTambahan Tg5.indd 5 26/10/2022 3:40 PM


Matematik Tambahan Jawapan

(b) Panjang lengkok PB = (π – 2) × 8 (b) Luas tembereng PAQ

2. (a)
= 9.14 cm
()
= 1 π (122) – 1 (12)(12) sin π
2 3 2 3 ()
C = 24π – 72 ( )
!3
2
= (24π – 36! 3 ) cm2
Luas semibulatan PBQ = 1 (π)(62)
2
15 cm = 18π cm2
Luas rantau berlorek = 18π – (24π – 36! 3 )
θ = 36! 3 – 6π
O B
7.5 cm (
= 18 2! 3 – π
6 )
Katakan ˙BOC = ˙AOD = q 5. (a)
B
tan q = 15 80 cm
7.5 20 cm
q = 1.107 rad
˙DOC = π – 2(1.107) A
= 0.928
≈ 0.93 rad kos ˙QBA = 20
80
(b) Jejari, OC = ! 152 + 7.52
= 16.77 cm
˙QBA = kos–1 20
80 ( )
Perimeter kawasan berlorek = CD + Lengkok CD = 1.318 rad
= 15 + 0.93(16.77) (b) ˙PAB = 2π – π – π – 1.318
= 30.6 cm 2 2
(c) Luas kawasan berlorek = 1.824 rad
Perimeter kawasan berlorek
= 1 (0.93)(6.772) – 1 (16.77) sin (0.93) = 1.824(30) + 1.318(50
2 2
= 130.77 – 112.72 = 1.120.62 cm
= 18.05 cm2 (c) PQ = ! 802 – 202
3. (a) = 77.46 cm2
A C
Luas kawasan berlorek
= Luas trapezium – Luas sektor APC
– Luas sektor CBQ
42 cm = 1 (30 + 50)(77.46) – 1 (1.318)(502)
55 cm 2 2
α – 1 (1.824)(202)
2
= 3 098.4 – 1.647.5 – 820.8
O = 630.1 cm2
kos a = 55 – 13 6. (a)
55 S
a = kos–1 42
55( )
= 0.702 rad
˙AOC = 2(0.702)
R
= 1.404 rad 5�2 cm
(b) Panjang lengkok ABC = 1.40(55)
5 cm
= 77.22 cm
(c) Luas keratan rentas P 5 cm O Q
= 1 (1.404)(552) – 1 (55)(55) sin 1.404
2 2 Jejari, OR = ! 5 + 5
2 2

= 632.04 cm2 = 5! 2 cm
4. (a) POQ = segi tiga sama sisi Panjang lengkok PAQ = π(5)
˙POQ = 60° = 5π cm
= π ˙PAQ = 90° = π
3 2
Panjang lengkok PAQ = π (12) Sudut major = 270° = 3π
3 2
= 4π

Jw01 Full SpotlightA+ MateTambahan Tg5.indd 6 26/10/2022 3:40 PM


Jawapan Matematik Tambahan

Panjang lengkok PSQ = 3π (5! 2 )


2 (
8. (a) Luas sektor OAB = 1 20° × π (102)
2 180° )
= 33.32 cm = 17.45 cm2
Perimeter kawasan berlorek
= 33.32 + 5π (
(b) Panjang lengkok DE = 1 35° × π (6)
2 180° )
= 49.03 cm = 35.87 cm
2 2 ( )
(b) Luas sektor PSQ = 1 3π (5! 2 ) OD = DE = 6 cm
Apabila OB = 10 cm, CD = 10 – 6
= 117.81 cm2
= 4 cm
Luas dua perentas
= Luas semibulatan – Luas segi tiga (
˙BOC = π – 20° × π
2 180° )
= 1.571 – 0.349
= 1 (π)(52) – 1 (5! 2 )(5! 2 )
2 2 = 1.222 rad
= 25 π – 25 Panjang lengkok BC = 1.222(10)
2 = 12.22 cm
= 14.22 cm2 Perimeter kawasan berlorek
Luas kawasan berlorek = 117.81 – 14.27 = CD + DE + EO + OB + BC
= 103.54 cm2 = 4 + 3.665 + 6 + 10 + 12.22
7. (a) Sudut setiap sektor = 180° = 35.89 cm
3 (c) Luas kawasan berlorek
= 60°
Dalam darjah, MOD = 60°
() ( )
= 1 π (102) – 1 35° × π (62) – 17.45
2 2 2 180°
2 = 78.54 – 10.996 – 17.45
= 30° = 50.09 cm2
Dalam radian, MOD = 30° × π 9. (a) Diberi lengkok QR = 15
180°
q(20) = 15
= π rad
6 q= 3
4
(b) Luas sektor MOD = 1
2 6
π
()
(92) = 0.75 rad
(b) Luas kawasan berlorek
= 27 π cm2
4 = 1 (0.75)(202) – 1 (0.75)(162)
(c) 2 2
= 54 cm2
M
10. (a) sin ˙COB = 6
10
˙COB = 0.644 rad
9 cm
˙AOB = 2π – 0.644 – 240° × π
= 1.449
180° ( )
≈ 1.45 rad
30°
O X (b) Lengkok ANB = 1.45(5)
= 7.25 cm
kos 30° = OX AB2 = 52 + 52 – 2(5)(5) kos 1.45
9 = 43.975
OX = kos 30°(9) AB = 6.63 cm
= 7.794 cm Perimeter tembereng ANB
MX = ! 92 – 7.7942 = 7.25 + 6.63
= 4.5 cm = 13.88 cm
Luas kawasan berlorek (c) Luas ANB
= Luas sektor MOD – Luas ∆MOX = 1 (1.45)(52) – 1 (5)(5) sin 1.45
2 2
2 6 ()
= 1 π (92) – 1 (4.5)(7.794)
2 = 5.716 cm2
= 3.67 cm2 OE = ! 102 – 62
(d) Perimeter kawasan berlorek = 8 cm
= MX + Lengkok MD + XD
Luas CBED = 1 (6)(8) – 1 (0.644)(52)
= 4.5 + π (9) + (9 – 7.794) 2 2
6 = 15.95 cm2
= 10.42 cm
Luas kawasan berlorek = 15.95 + 5.72
= 21.67 cm2

Jw01 Full SpotlightA+ MateTambahan Tg5.indd 7 26/10/2022 3:40 PM


Matematik Tambahan Jawapan

11. (a) Diberi lengkok BC : AE = 3 : 4 (e)


BC = 3 x 0.1 0.01 0.001 0.0001
AE 4
y 9.61 9.0601 9.0060 9.0006
q(8 + AB) 3
= had (x 2 + 6x + 9) = 9
2q(8) 4
x˜0
8 + AB =3 (f)
16 4
x 0.1 0.01 0.001 0.0001
AB = 3 (16) – 8
4 [ ] y 4.9 4.99 4.999 4.9999
= 4 cm
had 5 – x = 5
(b) Lengkok AD = 6 cm x˜0
q(8) = 6 2. (a) had 5 = 0
x
q = 3 rad x˜∞
4
(b) had 5x – x2 = had x(5 – x)
x x
()
Luas berlorek = 1 3 (122) – 1 3 (82)
2 4 2 4 ()
x˜0 x˜0
= had (5 – x)
= 54 – 24 x˜0
= 30 cm2 =5
12. (a) Diberi diameter litar = 100 m (c) had x2 + 5x – 14 = –14
x˜0
Maka, jejari litar = 50 cm
(d) had x + 1 = had 0 + 1
Panjang lengkok AB = 1 rad (50) x˜0 x–1 x˜0 0–1
= 50 m = –1
(b) 50 m = 50 km = 0.05 km (x + 1)(x – 1)
(e) had x – 1 = had
2
1 000
x˜1 x–1 x˜1 x–1
15 minit = 15 jam = 0.25 jam
60 = had x + 1
x˜1
Kelajuan = 0.05 =2
0.25
(x – 3)(x + 1)
(f) had x – x – 3 = had
2
= 0.2 km j–1
x˜3 x–3 x˜3 x–3
= had (x + 1)
Bab 2 Pembezaan x˜3
=4
Zon Formatif 2.1 3. (a) (i) Apabila x = 0, f(0) = 0 + 5
0+2
1. (a) = 5
2
x 0.1 0.01 0.001 0.0001
(ii) Apabila x ˜ 0 sama ada dari arah kiri
y 2.3 2.03 2.003 2.0003 atau kanan, f(x) ˜ 5 .
2
had (3x + 2) = 2
x˜0 Maka, had f(x) adalah wujud.
(b) x˜0
(–3) + 5
x 2.1 2.01 2.001 2.0001 (b) (i) had x + 5 =
x ˜ –5 x + 2 (–5) + 2
y 0.4535 0.4950 0.4995 0.5000
=0
had 22 = 0.5
x˜2 x (ii) had x + 5 = 3 + 5
x˜3 x+2 3+2
(c)
= 8
x 2.1 2.01 2.001 2.0001 5
y 5.41 5.0401 5.004001 5.0004
(iii) had x + 5 = 2
()
1 +5

had x 2 + 1 = 5
(d)
x˜2 x˜ 1
2
x+2
2 ()
1 +2

x 0.1 0.01 0.001 0.0001 = 11


5
y 0.9091 0.9901 0.9990 1.0000
1 =1 (iv) had x + 5 = 3
()
5 +5

had
x˜0 1+x x˜ 5
3
x+2
3 ()
5 +2

= 20
11

Jw01 Full SpotlightA+ MateTambahan Tg5.indd 8 26/10/2022 3:40 PM


Jawapan Matematik Tambahan

4. (a) y = x …1 –x3 – x2dx – x(dx)2 – (dx)3 + x3


= had
y + dy = x + dx …2 dx ˜ 0 3dx
2 – 1: dy = dx –x2dx – x(dx)2 – (dx)3
dy = had
=1 dx ˜ 0 3dx
dx dx[–x2 – xdx – (dx)2]
dy dy = had
Maka, = had dx ˜ 0 3dx
dx dx ˜ 0 dx –x2 – xdx – (dx)2
= had
=1 dx ˜ 0 3dx
(b) y = x …1 = –x 2

y + dy = 2(x + dx) dy f(x + dx) + f(x)


y + dy = 2x + 2dx …2 5. (a) = had
dx dx ˜ 0 dx
2 – 1: dy = 2dx (–x + dx)2 + x2
dy = had
=2 dx ˜ 0 dx
dx –x2 – 2xdx + (dx)2 + x2
dy dy = had
Maka, = had dx ˜ 0 dx
dx dx ˜ 0 dx dx(–2x + dx)
dy = had
=2 dx ˜ 0 dx
dx
= had –2x + dx
(c) y = 1 x2 …1 dx ˜ 0
2
= –2x
y + dy = 1 (x + dx)2 (b) y = 3x2 – x + 2 …1
2
y + dy = 3(x + dx)2 – (x + dx) + 2
y + dy = 1 x2 + xdx + 1 (dx)2 …2 y + dy = 3x2 + 6xdx + 3(dx)2 – x – dx + 2
2 2
1 …2
2 – 1: dy = xdx + (dx) 2
2 2 – 1: dy = 6xdx + 3(dx)2 – dx
dy 1 dy
= x + dx = 6x + 3dx – 1
dx 2 dx
dy dy dy dy
Maka, = had Maka, = had
dx dx ˜ 0 dx dx dx ˜ 0 dx
=x = 6x – 1
– 3 + 3
x + dx x dy – 3 +4+ 3 –4
(d) fʹ(x) = had
(c) = had x + dx x
dx ˜ 0 dx dx dx ˜ 0 dx
–3x + 3x + 3dx
–3x + 3x + 3dx
x(x + dx)
= had
= had x(x + dx)
dx ˜ 0 dx
dx ˜ 0 dx
3dx
3dx
x2 + xdx
= had x2 + xdx
dx ˜ 0 dx = had
dx ˜ 0 dx
= had 2 3
dx ˜ 0 x + xdx = had 2 3
dx ˜ 0 x + xdx
= 32
x = 32
(e) y = 2x2 + 5 …1 x
y + dy = 2(x + dx)2 + 5
y + dy = 2[x2 + 2xdx + (dx)]2 + 5 …2
Zon Formatif 2.2
2 – 1: dy = 2xdx + (dx)2 1. (a) y = 3x
dy dy
= 2x + dx =3
dx dx
dy (b) y = 5x – 4
Maka, = had 2x + dx
dx dx ˜ 0 dy
= 2x =5
dx
dy –1(x + dx)3+ x3 !x – 3
(f) = had (c) y =
dx dx ˜ 0 3dx 6
( )
1
–1[x3 + 3x2dx + 3x(dx)2 + (dx)3] + x3 y = 1 x2 – 3

= had 6
dx ˜ 0 3dx

Jw01 Full SpotlightA+ MateTambahan Tg5.indd 9 26/10/2022 3:40 PM


Matematik Tambahan Jawapan

dy
( ) ( )
d 2x3 – 5x2 – 1 = d (2x3 – 5x2 – x–1)
1
= 1 x 2

(c)
dx 12 dx x dx
= 6x2 – 10x + x–2
= 1
12! x = 6x 2 – 10x + 12
x
(d) Petua hasil darab: dx ( x )
(d) d –2x3 + 22 + 6x = d (–2x3 + 2x–2 + 6x)
dx
Jadikan u = 2x2 – 4x = –6x2 – 4x–3 + 6
du = 4x – 4 = –6x2 – 43 + 6
dx x
( )
(e) d x + 6x = d x + 6x ( )
2 2
dan v = x4
dv dx 3x dx 3x 3x
= 4x3
dy
dx
= uvʹ + vuʹ
dx 3(
= d x +2 )
dx = 1
= 12x 5 – 20x 4 3

( d x – 2x
)
+ 1
(
= d x2 – 2x2 + 12 )
3 3
(e) Petua hasil bahagi: (f)
Jadikan u = x dx x 2
dx x x x
du = 1
dx
= d (x – 2x–1 + x–2)
dx
dan v=x+5 = 1 + 2x–2 – 2x–3
dv =x = 1 + 22 – 23
dx x x
dy uvʹ – vuʹ 5. (a) Katakan u = 2x3 – 6
=
dx v2
(x + 5)(1) – (x)(1) du = 6x2
dx
=
(x + 5)2 dan y = u5
= 5 dy
(x + 5)2 = 5u4
du
(f) y = 4x5 – 3x2 + 23 dy dy du
x = ×
dx du dx
dy dy 5 dy 2 dy 2
=
dx dx
(4x ) –
dx
(3x ) + ( )
dx x 3
= 5u × 6x2
= 30x2u4
4

= 20x 4 – 6x – 64 = 30x 2(2x 3 – 6)4


x (b) Katakan u = 3x – 2
(g) y = 2xn + x3 – n + 13
dy du = 3
= 2nxn – 1 + (3 – n)x3 – n – 1 dx
dx dan y = u–2
= 2nxn – 1 + (3 – n)x 2 – n
dy
dr 3 ()
2. dV = 2 1 πr = 2 πr
3


du
= –2u–3(3)
= –6u–3
Apabila r = 6, = –6(3x – 2)–3
dV = 2 π(6)
dr 3 (c) Katakan u = 2x – 5 dan y = 33
u
= 4π du dy
=2 = –9u–4
3. f(x) = 3x2 + 2x–1 dx du
fʹ(x) = 6x – 2x–2 dy
= –9u–4(2)
= 6x – 22 dx
x
= –18u–4
Apabila x = 2, fʹ(x) = 6(2) – 22 = –18 4
2 (2x – 5)
= 12 – 2 (d) Katakan u = 4x2 – 3
4
= 11 1 du = 8x
2 dx
1

4. (a) d (5 – 7x – 8x ) = d (5) – d (7x2) – d (8x 4)


2 4 dan y = ! u = u2
dx dx dx dx dy 1 – 12
= –14x – 32x 3 = u
du 2
(b) d (–8x – x2) = d (–8x) – d (x2)
dx dx dx
= –8 – 2x

10

Jw01 Full SpotlightA+ MateTambahan Tg5.indd 10 26/10/2022 3:40 PM


Jawapan Matematik Tambahan

dy 1 – 12 (c) Katakan u = x2 – x
= u (8x)
dx 2 du = 2x – 1

1
dx
= 4xu 2 dan v = 3x2 + 2x
= 4x
dv = 6x + 2
! 4x 2 – 3 dx
(e) Katakan u = 3x + 5 dy
= (x2 – x)(6x + 2) + (3x2 + 2x)(2x – 1)
du = 3 dx
= 6x3 + 2x2 – 6x2 – 2x + 6x3 – 3x2 + 4x2 – 2x
dx
1 = 12x 3 – 3x 2 – 4x
dan y = 8 = 8u 2 (d) Katakan u = 3x + 4
!u
dy 1 1 du = 3


= (8)u 2 dx
du 2 dan v = (x – 2)3
1

dv = 3(x – 2)2
= 4u 2
dx
dy –
1 dy
= 4u 2 (3) = (2x + 4)3(x – 2)2 + (x – 2)3(3)
dx dx

1 = 3(x – 2)2[3x + 4 + x – 2]
= 12u 2
= 3(x – 2)2(4x + 2)
= 12 = 6(x – 2)2(2x + 1)
(! 3x + 5 ) (e) Katakan u = x3
(f) Katakan u = x + 2 du = 3x2
dx
du = 2 dan v = (3x – 2)5
dx
dv = 5(3x – 2)4(3)
1
dan y = 2 = 2u 2 dx
!u = 15(3x – 2)4
dy –
1
dy
=u 2 = x3(15)(3x – 2)4 + (3x – 2)5(3x2)
du dx
dy –
1
= 3x2(3x – 2)4[5x + 3x – 2]
= 2u 2
dx = 3x2(3x – 2)4(8x – 2)
= 2 = 6x 2(3x – 2)4(4x – 1)
!x + 2 (f) Katakan u = 2x 4
6. y = 4 (5x – 2) du = 8x3
5 dx
= 4x – 8 dan v = (x2 – 3x)2
5 dv
dy = 2(x2 – 3x)(2x – 3)
=4 dx
dx dy
= 2x 4(2)(x2 – 3x)(2x – 3) + (x2 – 3x)2(8x3)
7. (a) Katakan u = x2 – 2 dx
= 4x3(x2 – 3x)[x(2x – 3) + (x2 – 3x)(2)]
du = 2x = 4x3(x2 – 3x)(2x2 – 3x + 2x2 – 6x)
dx
dan v = x3 + 1 = 4x 3(x 2 – 3x)(4x 2 – 9x)
(g) Katakan u = 3x2 + 1
dv = 3x2
dx du = 6x
dy dx
= (x2 – 2)(3x2) + (x3 + 1)(2x) dan v = 2(x + 5)3
dx
= 3x 4 – 6x2 + 2x 4 + 2x dv = 3(2x + 5)(2)
= 5x 4 – 6x 2 + 2x dx
(b) Katakan u = 1 – 2x3 dy
= (3x2 + 1)(2x + 5)2(2) + (2x + 5)3(6x)
dx
du = –6x2 = 6(2x + 5)2[(3x2 + 1) + x(2x + 5)]
dx
= 6(2x + 5)2(3x2 + 1 + 2x2 + 5x)
dan v=x+2
= 6(2x + 5)2(5x 2 + 3x + 1)
dv =1 (h) Katakan u = x – 2
dx
dy du = 1
= 1 – 2x3 + (x + 2)(–6x2) dx
dx dan v = (3x + 1)2
= 1 – 2x3 – 6x3 – 12x2
= 1 – 8x 3 – 6x 2 dv = 2(2x + 1)(3)
dx
= 6(3x + 1)

11

Jw01 Full SpotlightA+ MateTambahan Tg5.indd 11 26/10/2022 3:40 PM


Matematik Tambahan Jawapan

dy (b) y = x2(3x – 2)
= (x – 2)6(3x + 1) + (3x + 1)2 Katakan u = x2 dan v = 3x – 2
dx
= (3x + 1)[6(x – 2) + (3x + 1)]
du = 2x dv = 3
= (3x + 1)(6x – 12 + 3x + 1) dx dx
= (3x + 1)(9x – 11) dy
= x2(3) + (3x – 2)(2x)
dy dx
8. (a) Dalam sebutan t, = 3 – 2t = 3x2 + 6x2 – 4x
dx
= 9x2 – 4x
(b) x = 3t + 2,
dy
t= x–2 Diberi
dx
=5
3
dy 9x – 4x = 5
2
Dalam sebutan x, = 13 – 2x
dx 3 9x2 – 4x – 5 = 0
9. y = 3 ; x = 1, – 5
1 – t3 9
y = 3(1 – t3)–1 (c) y = 6(1 – 2x)–1
dy dy
= –3(1 – t3)–2(–3t2) = –6(1 – 2x)–2(–2)
dx dx
9t = 12(1 – 2x)–2
= 12
(1 – t3)2 =
(1 – 2x)2
10. Jadikan u = x3 + x dan v = 2x2
Maka, nilai k ialah k = 12
du = 3x2 + 1 dv = 4x (1 – 2x)2 (1 – 2x)2
dx dx
dy 2x2(3x2 + 1) – (x3 + x)(4x) k = 12
=
dx (2x2)2
=
2x[x(3x2 + 1) – 2(x3 + x)] Zon Formatif 2.3
(2x2)2
3x 3
+ x – 2x 3
– 2x 1. (a) f(x) = 10 x–3
= 3
2x 3

fʹ(x) = (–3) 10 x–4


= x –3 1
3
3
2x
= – 104
= x –2 1
2
x
2x
f ʹʹ(x) = (–4) 10 x–5
10. Jadikan u = 2x – 1 dan v = 1 – x 3
du = 2 dv = –1 = 405
dx dx x
(1 – x)(2) – (2x – 1)(–1) (b) f(x) = (2x + 1)5
fʹ(x) = fʹ(x) = 5(2x + 1)4(2)
(1 – x)2
2 – 2x + 2x – 1 = 10(2x + 1)4
=
(1 – x)2 f ʹʹ(x) = 10(4)(2x + 1)3(2)
= 1 = 80(2x + 1)3
(1 – x)2 (c) f(x) = 4x 4 –2x3 + 4x2 – 5x + 1
Apabila x = 2, fʹ(2) = 1
fʹ(x) = 12x3 – 6x2 + 8x – 5
(1 – 2)2
f ʹʹ(x) = 36x 2 – 12x + 8
=1
(d) f(x) = ! x + 1
12. (a) y = (x2 + 1)(3x2 – 4x + 1) 1
Katakan u = x2 + 1
f(x) = (x + 1) 2
1
du = 2x fʹ(x) = 1 (x + 1) 2 (1)

dx 2
dan v = 3x2 – 4x + 1 = 1
dv = 6x – 4 2! x + 1
dx 3

( )
f ʹʹ(x) = 1 – 1 (x + 1) 2 (1)

dy (6x – 4)(2x) – (3x2 – 4x + 1)(x2 + 1) 2 2
=
dx (6x – 4)2 1
= –
= 12x – 8 – 3x – 4x + x2 + 3x – 4x + 1
2 4 3 2 2
4! (x + 1)3
(6x – 4)
(e) f(x) = (x2 + 1)(x – 1)
= 16x – 7 – 3x – 24x – 4x
2 4 3

(6x – 4) = x3 – x2 + x – 1
dy fʹ(x) = 3x2 – 2x + 1
Apabila x = 2, = – 31
dx 9 f ʹʹ(x) = 6x – 2

12

Jw01 Full SpotlightA+ MateTambahan Tg5.indd 12 26/10/2022 3:40 PM


Jawapan Matematik Tambahan

(f) f(x) = ! x + 5x2 4. y = 8x – x2


1 dy
fʹ(x) = 1 x + 5(2)x

2 = 8 – 2x
2 dx
= 1 + 10x d2y
= –2
2! x dx2
dy
( )
dy 2
3 2

( )
f ʹʹ(x) = 1 – 1 (x + 1) 2 + 10

2y 2 + + 32 = 2y(–2) + (8 – 2x)2 + 32
2 2 dx dx
= –4(8x – x2) + (8 – 2x)2 + 32
= – 1 + 10 = 8x2 – 64x + 96
4! x3 = 8(x2 – 8x + 12)
(g) f(x) = x(x – 6) + 10 Selesaikan:
= x2 – 6x + 10 8(x2 – 8x + 12) = 0
fʹ(x) = 2x – 6 x2 – 8x + 12 = 0
(x – 6)(x – 2) = 0
f ʹʹ(x) = 2
x = 6, x = 2
2. (a) y = 4x3
dy
= 12x2 Zon Formatif 2.4
dx
d2y dy
= 24x 1. (a) (i) = 4x
dx2 dx
(b) y = 3x2 + 1 x (ii)
dy
= 3x 2 + 2
5 dx
dy 1 dy 6
= 6x + (iii) =
dx 5 dx (1 – 2x )2
d2y dy
=6 (iv) = 9(3x + 1)2
dx2 dx
(c) y = (x – 2)(3x + 1)2 dy
(b) (i) Apabila x = 0 ˜ =0
Biarkan u = x – 2 dx
du = 1 Apabila x = –1,
dy
= –4
dx dx
dan v = (3x + 1)2 dy
dv (ii) Apabila x = 0 ˜ = –1
= 2(3x + 1)(3) dx
dx dy
= 6(3x + 1) Apabila x = –1, =5
dx
dy
= (x – 1)6(3x + 1) + (3x + 1)2(1) dy
dx (iii) Apabila x = 0 ˜ = –1
= (3x + 1)[6(x – 1) + (3x + 1)] dx
= (3x + 1)(6x – 6 + 3x + 1) dy
Apabila x = –1, =2
= (3x + 1)(9x – 5) dx
= 27x2 – 24x – 11 dy
(iv) Apabila x = 0 ˜ =9
d2y dx
= (3x + 1)(9) + (9x – 5)(3)
dx2 dy
= 27x + 9 + 27x – 15 Apabila x = –1, = 36
dx
= 54x – 6 2. y = ! x
3

1
(d) y = (x2 – 3x)2 = x3
2
dy
fʹ(x) = 1 x 3

= 2(x 2 – 3x)(2x – 3)
dx 3
= (4x – 6)(x2 – 3x)
= 1
dy
2
3 3! x2
= (4x – 6)(x2 – 3x) + (x2 – 3x)(4)
dx2
= 8x2 – 12x – 12x + 18 + 4x2 – 12 (a) Apabila x = 5,
kecerunan tangen = 1
= 12x2 – 36x + 18
3 3! 52
3. f(x) = 24 – 1 = 0.114
x x
= 2x–4 – x–1 (b) Apabila x = –4,
fʹ(x) = –8x–5 + x–2 kecerunan tangen = 1
3 3! (–4)2
fʹʹ(x) = 406 – 23
x x = 0.132
Apabila x = –1, fʹʹ(–1) = 42

13

Jw01 Full SpotlightA+ MateTambahan Tg5.indd 13 26/10/2022 3:40 PM


Matematik Tambahan Jawapan

(c) Apabila x = – 1 ,
2 ( )
Pada titik 1, – 5 , mtangen = –2.
2
kecerunan tangen = 1 Persamaan tangen:
y + 5 = –2(x – 1)
!( )
33 – 1
2

2 2
= 0.529 y = –2x – 1
2
(d) Apabila x = 1, 2y = –4x – 1
kecerunan tangen = 1
(b) 3y = x3 – 4
3 3! 12
y = 1 x3 – 4
= 1 3 3
3 yʹ = x2
3. (a) Katakan f(x) = 3x3 – 2x
Pada x = 1, yʹ = 4
fʹ(x) = 9x2 – 2
Pada titik (2, 20), fʹ(2) = 9(2)2 – 2 Koordinat-y, y = 1 (–2)2 – 4
3 3
= 34 y = –4
fʹ(x) . 0. Maka, kecerunannya adalah positif Pada titik (–2, –4), mtangen = 4.
dan garis tangen condong ke kanan.
Persamaan tangen:
(b) Katakan f(x) = 2 – 32
x x y + 4 = 4(x + 2)
= 2x–1 – 3x–2 y + 4 = 4x + 8

fʹ(x) = –2x–2 + 6x–3 y = 4x + 4
= – 22 + 63 (c) y = ! x – 3
x x 1

( )
Pada titik 2, 1 , fʹ(2) = – 2 2 + 6 3
4 (2) (2)

= (x – 3) 2
1
yʹ = 1 (x – 3) 2


= 1 2
4 1
fʹ(x) . 0. Maka, kecerunannya adalah positif =
2! x – 3
dan garis tangen condong ke kanan.
Pada x = 12, yʹ = 1
4. Katakan f(x) = x + 1
2

x–1 2! 12 – 3
Jadikan u = x2 + 1 dan v = x – 1 = 1
du = 2x dv = 1 6
dx dx Koordinat-y, y = ! 12 – 3
dy (x – 1)(2x) – (x2 + 1)(1) y=3
=
dx (x – 1)2 Pada titik (12, 3), mtangen = 1 .
6
= 2x – 2x – x2 – 1
2 2

(x – 1) Persamaan tangen:
= x – 2x –2 1
2
y – 3 = 1 (x – 12)
(x – 1) 6
Diberi y = 1, x + 1 = 1
2
y – 3 = 1x – 2

x–1 6
x2 + 1 = x – 1 y = 1 x + 1
6
x2 – x + 2 = 0 6y = x + 6
x= 1
1 2 6. (a) Diberi x = –3,
Pada x = , kecerunan tangen = 18
2
() ()
1 2–2 1 –1
4p(–3) + 9 = 18
kecerunan tangen, fʹ ()
1 = 2 2
–12p + 9 = 18
2
( )1 – 12
2 –12p = 9
= –7 p=–3
4
5. (a) 2y = x2 – 6x (b) Apabila p = – , 3
4
y = 1 x2 – 3x dy

2
yʹ = x – 3 dx ( )
=4– 3 x+9
4
Pada x = 1, yʹ = (1) – 3 = –3x + 9
= –2 dy
Pada titik (1, 3), = –3(1) + 9
dx
Koordinat-y, y = 1 (1)2 – 3(1) =6
2
y=–5
2

14

Jw01 Full SpotlightA+ MateTambahan Tg5.indd 14 26/10/2022 3:40 PM


Jawapan Matematik Tambahan

Persamaan tangen: mtangen × mnormal = –1


y – 3 = 6(x – 1) –9 × mnormal = –1
y = 6x – 6 + 3
y = 6x – 3 mnormal = 1
9
dy Maka, kecerunan tangen ialah –9 dan
7. = 2x + 3
dx kecerunan normal ialah 1 .
dy 9
Pada x = 1, =5 (b) Persamaan tangen:
dx
y – 4 = –9(x – 1)
Persamaan tangen:
y = –9x + 13
y + 5 = 5(x – 1)
Persamaan normal:
y = 5x – 5 – 5
y = –5x – 10 y – 4 = 1 (x – 1)
9
Kecerunan normal = – 1 x – 9y + 35 = 0
5 dy
Persamaan normal: 11. = 4x – 4
dx
y + 5 = – 1 (x – 1) dy
5 Apabila x = 2, =4
5y + 25 = –x + 1 dx
x + 5y + 24 = 0 Pada x = 2, y = 2(2)2 – 4(2) + 1
=1
8. y = 2x2 – 5
x mnormal = – 1
= 2x2 – 5x–1 4
Persamaan normal:
dy
= 4x – 52 y – 1 = – 1 (x – 2)
dx x 4
Pada titik A(1, –3), y = – 1x + 3
dy 4 2
= 4(1) – 5 Titik persilangan antara normal dan lengkung:
dx 1
= –1
Pada titik A(1, –3) dan B(k, 2), – 1 x + 3 = 2x2 – 4x + 1
4 2
2 + 3 = –1 –x + 6 = 8x2 – 16x + 4
k–1 8x2 – 15x – 2 = 0
5 = –k + 1
(8x + 1)(x – 2) = 0
k = –4
x = – 1, x = 2
9. Pada garis y – x = 2, kecerunannya ialah 1. 8
mtangen × 1 = –1 Maka, titik persilangan antara normal dan
mtangen = –1 lengkung yang satu lagi adalah pada x = – 1 .
8
dy dy
Bagi y = 7x – 3x2, = 7 – 6x 12. = –1 – 4x,
dx dx
dy dy
Apabila = –1 Apabila =7
dx dx
7 – 6x = –1 –1 – 4x = 7
–4x = 8
x= 4 x = –2
3
Apabila x = –2,
4
3 3 ( ) ( )
Pada x = , y = 7 4 – 3 4
3
2
y = 1 – (–2) – 2(–2)2
y=4 y = –5
Persamaan normal kepada lengkung: Maka, a = –2 dan b = –5.
13. y = x + 4
y–4=1x– 4 ( 3 ) dy
x
y=x+ 8 = 1 – 42
3 dx x
10. (a) y = 6 + 5x – 7x2 dy
dy Apabila x = 4, = 1 – 42
dx (4)
= 5 – 14x
dx 3
=
Pada titik P, x = 1 4
dy
Kecerunan tangen, = 5 – 14(1) Oleh itu, mnormal = – 3
dx 4
= –9
Koordinat-y pada x = 4, y = 4 + 4
4
y=5

15

Jw01 Full SpotlightA+ MateTambahan Tg5.indd 15 26/10/2022 3:40 PM


Matematik Tambahan Jawapan

Persamaan normal melalui titik (4, 5): 18. (a) Diberi V = 72, biarkan tinggi kotak ialah t.
y – 5 = – 4 (x – 4) 2x 2t = 72
3
t = 362
y – 5 = – 4 x + 16 x
3 3 Gantikan ke dalam
y = – 4 x + 31 A = 4xt + 2xt + 4x 2
3 3
4x + 3y – 31 = 0
( )
= 6x 362 + 4x 2
x
dy = 4x 2 + 216 [tertunjuk]
14. = 3x2 – 6x + 3 x
dx (b) Untuk A ialah minimum . 0
dy d2 A = 8 + 216
Apabila =0 dx2 x
dx
Apabila x = 3, d A2 = 24 . 0 (minimum)
2
3x – 6x + 3 = 0
2

x2 – 2x + 1 = 0 dx
Maka, nilai x = 3 apabila A ialah minimum.
(x – 1)2 = 0
x=1 19. y = x2 + 4x + 3
Titik pegun ialah (1, 0). Titik tersebut adalah titik dy
= 2x + 4
lengkuk balas. dx
dy Gantikan x = 2 dan dx = 0.5 unit s–1
15. = – 4 – 2x, dt
dx dy dy dx
dy = ×
Apabila = 0, dx dx dt
dx
= [2(2) + 4] × 0.5
– 4 – 2x = 0
= 4 unit–1
x = –2
x = –2 ialah titik pusingan. 20. Isi padu sfera, V = 4 πj3
3
Koordinat-y, y = 1 – 4(–2) – (–2)2 dV = 4πj2
y=5 dj
d 2y dj
= –2 , 0 Diberi = –2 cm s–1.
dx 2 dt
d 2y dj
Oleh sebab , 0, maka titik (–2, 5) ialah titik Gantikan dV dan ,
dx 2 dj dt
maksimum. dV = dV × dr
16. f(x) = x3 – x2 dt dr dt
fʹ(x) = 3x 2 – 2x = 4π(8)2 × –2
Apabila fʹ(x) = 0, = –512π cm3 s–1
3x 2 – 2x = 0 21. y = 272
x(3x – 2) = 0 x
x = 0 dan x = 2 = 27x–2
3 dy
d 2y = – 543
Untuk nilai minimum, = 6x – 2. dx x
dx 2
Apabila dx = 3.01 – 3 = 0.01
d 2y
Pada x = 0, = –2 , 0 (titik maksimum) Gantikan x = 3 dan dx = 0.01,
dx 2 dy
d 2y dy ≈ × dx
Pada x = 2 , = 1 . 0 (titik minimum) dx
3 dx 2
Maka, Nilai minimum: dy = – 543 × 0.01
(3)
( )
f 2 =– 4.
3 27 dy = –0.02
dR v 22. T = x3 – 2x2 + 1
17. = – 500 ,
dv 2 v2 dT = 3x2 – 4x
dx
Apabila dR = 0
dv Apabila x = 2, dT = 3(2)2 – 4(2)
v – 500 = 0 dx
2 v2 dT = –4
v3 – 1 000 = 0 dx
2v2 Jika x = 2, T = (2)3 – 2(2)2 + 1
v3 = 1 000 T=1
v = 10 dT = 1.98 – 2 = –0.02

16

Jw01 Full SpotlightA+ MateTambahan Tg5.indd 16 26/10/2022 3:40 PM


Jawapan Matematik Tambahan

Gantikan ke dalam: 2. (a) had 6 – 6 = 0


dx
T(x + dx) ≈ T(x) + dT dx dx ˜ 0
dx 5(x + dx) – 5x
≈ 1 + (–4)(–0.02) (b) had =5
dx ˜ 0 dx
≈ 0.92 5(x + dx) + 2(x + dx) – (5x2 + 2x)
2

Maka, nilai hampir bagi T ialah 0.92. (c) had


dx ˜ 0 dx
23. Luas bulatan, A = πj2 10xdx + 5(dx)2 + 2dx
= had
dA = 2πj dx ˜ 0 dx
dj = had 10x + 5dx + 2
Apabila jejari, j = 8, dA = 2π(8) dx ˜ 0
dj = 10x + 2
dA = 16π
dj 1 – 1
Kadar penyusutan luas: (x + dx)2 x 2
(d) had
dx ˜ 0 dx
dA = dA × dj
x2 – (x + dx)2
dt dr dt = had 2
dx ˜ 0 x (x + dx) dx
2
= 16π × (–0.4)
= –6.4π cm2 s–1 (x – x – dx)(x + x + dx)
= had
dx ˜ 0 x2(x + dx)2dx
24. y = x3 – 2x2 + 5 –dx(2x + dx)
dy = had
dx ˜ 0 x (x + dx) dx
2 2
= 3x2 – 4x
dx 2x + dx
dy = had 2
dx ˜ 0 x (x + dx)
2
Apabila x = 4, = 3(4)2 – 4(4)
dx
= 32 = – 2x
x 2 x2
Kadar penyusutan luas:
dy dy dx = – 23
= × x
dt dx dt k(x + dx)3
= 32 × 2 3. had
dx ˜ 0 dx
= 64 unit s–1
k[x3 + 3x2dx + 3x(dx)2 + (dx)3] – kx3
25. Isi padu silinder, I = πj2h = had
dx ˜ 0 dx
Diberi jejari silinder = 4, I = 16πh dx[3kx2 + 3kxdx + 3k(dx)2]
dI = 16π = had
dx ˜ 0 dx
dt
dI = dI × dh = had 3kx2 + 3kxdx + 3k(dx)2
dx ˜ 0
dt dh dt
= 16π × (–0.25) = 3kx2 [Tertunjuk]
= –4π cm3 s–1 4. (a) y = x3 – 2x2 + 5
dy
= 3x 2 – 4x
Zon Sumatif dx
(b) y = 1
Kertas 1 x
dy
2(–1) + 1 = – 12
1. (a) had 2x + 1 + 3 = +3 dx x
x–2
x ˜ –1 (–1) – 2 (c) y = (–5x + 7)4
= 10 y = 4(–5x + 7)3(–5)
3 = –20(–5x + 7)3
(b) had x2 – x – 2 = –2
x˜0 (d) y = (2 – x 4)–3,
(c) had 2x – 5x + 2 = ∞
3 dy
x˜∞ = –3(–4x3)(2 – x 4)–4
dx
(x + 3)(x – 1)
= 12x 4 4
3
(d) had = had x + 3
x˜0 (x – 1) x˜0 (2 – x )
=3 1

(x – 5)(x + 5) (e) y = (1 + x) 2
(e) had = had x + 5 dy 1
x ˜ 5 (x + 6)(x – 5) x˜0 x+6 =
dx 2! 1 + x
= 10
11

17

Jw01 Full SpotlightA+ MateTambahan Tg5.indd 17 26/10/2022 3:40 PM


Matematik Tambahan Jawapan

(f) y = (2x + 1)3(x – 8)7 (b) y = 2 + 3x – x3


Petua hasil darab: dy
Jadikan u = (2x + 1)3 = 3 – 3x2
dx
uʹ = 6(2x + 1)2 d 2y
dan v = (x – 8)7 = –6x
dx 2
vʹ = 7(x – 8)6 dy
dy Apabila =0
= uvʹ + vuʹ dx
dx
= (2x + 1)37(x – 8)6 + (x – 8)76(2x + 1)2 3 – 3x = 0
2

= (2x + 1)2(x – 8)6[7(2x + 1) + 6(x – 8)] x2 = 1


= (2x + 1)2(x – 8)6(20x – 41) x = 1, –1
(g) y = x–3(5 + 2x) Gantikan x = 1, y = 2 + 3(1) – (1)3
y=4
Jadikan u = x–3 = 13 º (1, 4)
x
Gantikan x = –1, y = 2 + 3(–1) – (–1)3
uʹ = – 34 y=0
x
dan v = 5 + 2x º (–1, 0)
vʹ = 2 d 2y
Pada x = 1, = –6 , 0 (maksimum)
Petua hasil darab dx 2
dy d 2y
= uvʹ + vuʹ Pada x = –1, = 6 . 0 (minimum)
dx dx 2
= 13 (2) + (5 + 2x) – 34
x x ( ) Maka, (1, 4) ialah titik maksimum dan (–1, 0)
ialah titik minimum.
2 15
= 3 – 4 – 3 6
(c) y = x + 1
2
x x x
x
= –4x – 15 dy
x4 = 1 – 12
dx x
(h) y = 3x2 = 3x2(x – 1)–4 d 2y 2
(x – 1)4 =
dx 2 x 3
Jadikan u = 3x2 dan v = (x – 1)–4 dy
uʹ = 6x vʹ = –4(x – 1)–5 Apabila =0
dx
Petua hasil bahagi:
x –2 1 = 0
2

dy uvʹ – uvʹ x
= x2 = 1
dx v2
(x – 1)–4(6x) – 3x2(–4(x – 1)–5) x = 1, –1
= (1)2 + 1
(x – 1)–5 Gantikan x = 1, y =
1
6x(x + 1) y=2
= –
(x – 1) 5 º (1, 2)
5. (a) y = x2 – x (–1)2 + 1
Gantikan x = –1, y =
(–1)
dy y = –2
= 2x – 1
dx º (–1, –2)
dy
Apabila =0 d 2y
dx Pada x = 1, = 2 . 0 (minimum)
dx 2
2x – 1 = 0
d 2y
x= 1 Pada x = –1,
dx 2
= –2 , 0 (maksimum)
2
Maka, (–1, –2) ialah titik maksimum dan (1, 2)
Gantikan x = 1 , y = 1 – 1
2 2
2

2 () ialah titik minimum.


y=– 1 6. f(x) = x3 – 2x2 – x + 1
4 fʹ(x) = 3x2 – 4x – 1
(
Titik pegun pada lengkung ialah 1 , – 1 .
2 4 ) 2 ( )
Apabila x = 1 , fʹ 1 = – 9
2 4
d 2y Maka, kecerunan tangen kepada lengkung ialah
= 2 . 0
dx 2
d 2y – 9.
Oleh sebab
dx 2 2 (
. 0, titik pegun 1 , – 1
4 ) 4
7. f(x) = x(x2 + 4)
ialah titik minimum. Jadikan u = x dan v = x2 + 4
uʹ = 1 vʹ = 2x

18

Jw01 Full SpotlightA+ MateTambahan Tg5.indd 18 26/10/2022 3:40 PM


Jawapan Matematik Tambahan

Petua hasil darab: 11. y = x3 + px2 + 2


dy dy
= x(2x) + (x2 + 4)(1) = 3x2 + 2px
dx dx
= 3x 2 + 4
Apabila x = 4, 3(4)2 + 2p(4) = 0
Pada x = 1, fʹ(1) = 7.
p=6
Gantikan x = 1, y = 1(12 + 4)
Gantikan x = 4 dan p = 6,
=5
y = (4)3 + (6)(4)2 + 2
Persamaan tangen melalui (1, 5): y – 5 = 7(x – 1)
y = 162
y = 7x – 12
Titik pegun ialah (4, 162).
8. Diberi y = x3 + ax d 2y
dy = 6x + 12
= 3x2 + a …1 dx 2
dx d 2y
mnormal × mtangen = –1 Apabila x = 4, = 36 . 0
dx 2
( )
mtangen × 1 = –1
2 Oleh sebab
dy
2

dx 2
. 0, maka titik (4, 162) ialah titik
mtangen = –2 …2 minimum.
1 = 2: 3(3)2 + a = –2 12. y = x2 – 6x + 2
a = –14 dy
= 2x – 6
Diberi y = b, b = 23 + (–14)(2) dx
dy
b = –20 Gantikan , dx = –0.5 dan x = 3,
Maka, nilai a = –14 dan b = –20. dx dt
9. (a) y = x2 + 4x + 3 dy dy dx
= ×
dy dt dx dt
= 2x + 4 …1 = [2(3) – 6] × (–0.5)
dx
=0
Daripada y = 4x + k,
m = 4 …2 13. Luas, A = πj2
1 = 2: 2x + 4 = 4 dA = 2πj
x=0 dj
dj
Apabila x = 0, y = (0)2 + 4(0) + 3 Gantikan dA , = 2 dan j = 3,
y=3 dj dt
Persamaan tangen melalui (0, 3): dA = dA × dj
y – 3 = 4(x – 0) dt dj dt
y = 4x + 3 = 2π(10) × 2
Bandingkan dengan y = 4x + k, maka k = 3. = 40π
(b) Didapati titik P ialah (0, 3).
Maka, kadar perubahan luas sfera itu ialah
mnormal × mtangen = –1
40π cm2 s–1.
mtangen × 4 = –1
14. Diberi dh = –0.25
mtangen = – 1 Isi padu silinder, V = πr2h
4
dV = πr2
Kecerunan normal = – 1 . dh
4
Persamaan normal:
Kadar perubahan, dV = dV
y – 3 = – 1 (x – 0) dh dh
4
x + 4y – 12 = 0 dV = dV × dh
dh
10. y = 9x2 + 18x – 7 = π(4)2 × (–0.25)
dy = –4π
= 18x + 18
dx Maka, perubahan kecil pada isi padu silinder ialah
dy menyusut sebanyak 4π.
Apabila = 0,
dx
15. y = 272 = 27x–2
18x + 18 = 0 x
dy
x = –1 = –54x–3 = – 543
Pada x = –1, y = 9(–1)2 + 18(–1) – 7 dx x
y = –16 dx = 3.01 – 3 = 0.01
Maka, titik pegun ialah (–1, –16).

19

Jw01 Full SpotlightA+ MateTambahan Tg5.indd 19 26/10/2022 3:40 PM


Matematik Tambahan Jawapan

dy dp
Gantikan , dx dan x = 3, 2. (a) =3
dx dx
dy dx = dx × dp
dy = × dx
dx dt dp dt
= – 543 × 0.01 = 1 × 4
(3) 3
= –0.02 = 4 unit s–1
3
16. y = 83 = 8x–3
x (b) y = –3(3x + 2)–2
dy
= – 244 dy 18
dx x =
dx (3x + 2)3
dy
Apabila x = 4, =– 3 (c) dx = 1.98 – 2 = –0.02
dx 32
(a) dx = 2.01 – 2 = 0.01 dy
dy = × dx
Apabila x = 2, y = 0.125, dx
dy = 18 × –0.02
dy = × dx (3(1) + 2)3
dx
= 0.00288
= – 244 × 0.01
x 3. (a) Biarkan lebar = y
= –0.015 Perimeter = 2x + y = 20
Nilai hampir: y = 20 – 2x
y + dy = 0.125 – 0.015 Luas, A = xy
= 0.985 = x(20 – 2x)
(b) dx = 1.98 – 2 = –0.02 = 20x – 2x2 [tertunjuk]
Apabila x = 2, y = 1, (b) dA = 20 – 4x
dy dx
dy = × dx d 2 A = – 4 , 0 (maksimum)
dx
dx 2
= – 244 × –0.02 dy
x =0
= 0.03 dx
Nilai hampir: 20 – 4x = 0
y + dy = 1 + 0.03 x = 5
= 1.03 Apabila x = 5,
Kertas 2 Lmaksimum = 20(5) – 2(5)2
= 50 cm2
1. (a) x = 3 – p
p=3–x 4. (a) Isi padu, x2t = 8
Gantikan p = 3 – x, t = 82
x
y = 2(3 – x) – (3 – x)2
Luas permukaan, A = 2x2 + 4xt
= –x2 + 4x – 3
dy
dx
= –2x + 4 ( )
= 2x2 + 4x 82
x
dy 32
(b) dx = dx × = 2x + [tertunjuk]
2
x
dt dy dt (b) A = 2x2 + 32
1 x
= × 0.6
–2(1) + 4 dA = 4x – 32
= 0.3 dx x2
(c) Apabila dA = 0,
Maka, kadar perubahan pada x ialah dx
0.3 unit s–1.
(c) dp = 1.96 – 2 = –0.04 4x – 322 = 0
x
dy 4x3 = 32
= 2 – 2p
dp x=2
dy d A = 4 + 64 ,
2
dy = × dp
dp dx 2 x2
= [2 – 2(2)] × –0.04 Apabila x = 2, d A2 = 12 . 0 [nilai minimum]
2

= 0.08 dx
Untuk xminimum,
Luas permukaan terkecil, Amin = 2(2)2 + 32
2
= 24 cm2

20

Jw01 Full SpotlightA+ MateTambahan Tg5.indd 20 26/10/2022 3:40 PM


Jawapan Matematik Tambahan

5. (a) V = 1 (92)(10) Bab 3 Pengamiran


3
= 270 cm3
dV = –9.5 × 20
Zon Formatif 3.1

= –190 cm3 s–1 1. Diberi f(x) = 7 – 3x 4

3 10 ( )
1 9 h2 h = 80 fʹ(x) = –12x 3

h = 20 cm ∫
Seterusnya –12x3 dx = f(x)
3
(b) dh = dh × dV ∫ –12x 3
dx = 7 – 3x 4
dt dV dt
dh = 200 × –20 2. Diberi f(x) = x + 1
x
2
( )
dt 81h2
fʹ(x) = 2x – 23
dh = 100 × –20 x
dt 81 20 2
( )
3 Seterusnya 2x – 23 dx = f(x)
x∫
= – 5 cm s–1
9 2x – 3 dx = x + 1
∫2
x x ( )2

dy
6. = 6x – 4
2
3. Diberi f(x) = x2 – 5x + c
dx
fʹ(x) = 2x – 5
dy
2
= 12x
dx 2 ∫
Seterusnya 2x – 5 dx = f(x)
(a) Apabila x = 2,
dy
= 6(2)2 – 4 = 20
∫ 2x – 5 dx = x 2
– 5x + c
dx
d 2y
= 12(2) = 24
4. Diberi f(x) = 42 + 5x
x ( )
dx 2 fʹ(x) = 4(x)
dy
(b)
dx
= 20 ∫
Seterusnya g(x) dx = f(x)

!
1+
d 2y
dx 2
! 1 + 24
∫ g(x) dx = x4 2 + 5x
= 20 5. Diberi y = f(x)
! 25 dy
=4 = 3h(x)
dx
7. (a) Halaju bola, v = h(t) = 20 – 10t
(b) v = 0, 20 – 10t = 0

Seterusnya 3h(x) dx = f(x)

t=2
Bola itu mencapai tinggi maksimum pada saat
∫ h(x) dx = 13 f(x)
ke-2.
(c) 20t – 5t2 = 15
Zon Formatif 3.2

(t – 3)(t – 1) = 0
t = 3, t = 1
1. (a) ∫ – 23 dx = – 23 + c
∫ 3x dx = 3x
8+1
Bola itu mencapat ketinggian 15 meter dan (b) 8
+c
pada t = 1 saat atau t = 3 saat. Oleh itu bola itu 8+1
= x +c
9
mula jatuh pada t = 3 saat.
3
Apabila t = 3,
3 dx = 3x dx
∫ ∫
–2
v(3) = 20 – 10(3) (c)
4x2 4
= –10 m s–1
= 3x
–2 + 1

Oleh sebab bola itu jatuh semula ke atas +c


–2 + 1
permukaan tanah, halajunya adalah negatif.
Maka, pecutan bola tersebut yang jatuh dari =–3 +c
x
ketinggian 15 meter itu ialah –10 m s–1.
2. (a) (3x – 1) dx = 3x

2+1
2
–x+c
2+1
= x3 – x + c

21

Jw01 Full SpotlightA+ MateTambahan Tg5.indd 21 26/10/2022 3:40 PM


Matematik Tambahan Jawapan

(b)
x∫ (6x 3
)
+ 42 – 3 dx = (6x3 + 4x–2 – 3) dx ∫ lengkung melalui titik A(4, 1), gantikan x = 4, y = 1
1 = 3(4) – 42 + c
= 6x + 4x c=5
3+1 –2 + 1
+c
3+1 –2 + 1 y = 3x – x2 + 5
= 3x – 4 + c
4
6. Fungsi kecerunan = fʹ(x) = 1
2 x (3 – x)2
(c) ∫ 5(x 2

+ 2) dx = (5x2 + 10) dx
Persamaan lengkung f(x) = 1 dx ∫( )
(3 – x)2
= 5x
2+1

2+1
+ 10x + c
f(x) = 1
(3 – x)2
dx ∫( )
= 5x + 10x + c
3

3 = ∫ (3 – x) dx
–2

(d) ∫ (x – 2)(x + 3) dx = ∫ (x 2
+ x – 6) dx =
(3 – x)–1
–1(–1)
+c
= x + x
2+1 1+1
– 6x + c 1
2+1 1+1 = +c
(3 – x)
= x + x – 6x + c
3 2
lengkung melalui titik (4, 2), gantikan x = 4, f(x) = 2
3 2
1
∫ ( 1 +x x ) dx = ∫ ( 1 + x 4 dx
)
4
(e) 2= +c
2
x2 x2 (3 – 4)
c=3
= ∫ (x –2
+ x ) dx2
1
f(x) = +3
(3 – x)
= x + x
–2 + 1
2+1
+c
–2 + 1 2 + 1
=–1 + x +c
3

x 3 Zon Formatif 3.3

(7x + 5) 4+1
3. (a) ∫ (7x + 5) 4
dx =
7(4 + 1)
+c 1. (a) ∫
2
1
2 x dx = 2x2 2
3 3(2) 1 [ ]
(7x + 5)5
=
35
+c
3 (
= 1 (2)2 – 1 (2)2
3 )
5(4 – 7x) 4+1 =1
(b) ∫ 5(4 – 7x) 4
dx = +c
–7(4 + 1)
∫ ( )
4 dx = 3 4x–3 dx

3
(b)
(4 – 7x)5 –1 x3 –1
= +c
4
7 = – 22
3

x 1 [ ]
(c) ∫ dx = 4(3x – 7)–3 dx ∫
(3x – 7)3
4(3x – 7)–3 + 1
= – 22 – – 22
3 1( ) ( )
= +c
3(–3 + 1)
2
= 16
9
=– +c
(2 – 7x) dx = 2x – 7x
∫ [ ]
2 2 2
3(3x – 7)2 (c)
0 2 0
dy
4. (a) Fungsi kecerunan =
dx
= x3 – 1
= 2(2) –
7(2)2
2 (– 2(0) – ) (
7(0)2
2 )
Persamaan lengkung dy = (x3 – 1) dx ∫ ∫ = –10

y= x –x+c
3


1
4 2. (a) x(3x2 – 1) dx
–2
lengkung melalui titik A(2, –1), gantikan x = 2,

1
= (3x3 – x) dx
y = –1 –2

[
= 3x – x ]
4 2 1
(2)4
–1 = –2+c 4 2 –2
4
c = –3 = (
3(1)4 (1)2
4

2

4

2) (
3(–2)4 (–2)2
)
y= x –x–3
3
39
4 =–
dy 4
5. Fungsi kecerunan = = 3 – 2x
dx
Persamaan lengkung dy = (3 – 2x) dx ∫ ∫
y = 3x – x2 + c

22

Jw01 Full SpotlightA+ MateTambahan Tg5.indd 22 26/10/2022 3:40 PM


Jawapan Matematik Tambahan

3x 4 – 5 dx =
∫ ∫ ∫
3 3 1
(b) (3x2 – 5x–2) dx (c) Luas A = x(x – 1)(x – 2) dx
1 x2 1 0

[ ]
= x3 + 5
x
3
1
=∫
1
0
x3 – 3x2 + 2x dx

= ((3) + 5 ) – ((2) + 5 ) [
= x – x3 + x2 ]
4 1
3 3
3 1 4 0

= 68
3
= 1 unit 2
4

2

2
(c) (1 – 3x)2 dx Luas B = x(x – 1)(x – 2) dx
1 1

=∫
2

2
= (1 – 6x + 9x2) dx x3 – 3x2 + 2x dx
1 1

[ ] [
= x – x3 + x2 ]
2 4 2
= x – 3x2 + 3x3
1 4 1

= [2 – 3(2) + 3(2) ] – [1 – 3(1) + 3(1) ]


2 3 2 3
= 1 unit 2
4
= 13
Luas rantau berlorek = Luas A + Luas B

∫ 1
∫ = 1 + 1
2 2
(d) dx = (3x – 4)–3 dx
1 (3x – 4)4 1 4 4

[ (3x–2(3)
– 4)
] = 1 unit 2
–2 2
= 2
1

= [–
6(3(2) – 4) ] [ 6(3(1) – 4) ] ∫( )
1 – – 1 3 4
2 2 5. (a) Luas = dx
1 y2

=–1
8 = – 4 [ ]
y 1
3

∫ ∫
4 7
3. (a)
7
g(x) dx = – g(x) dx = –7
4 8
= unit 2
3
(b) ∫ 2g(x) dx = 2 ∫ g(x) dx = 10
4 4

2
0 0 (b) Luas = (y2 – 4) dx
–1

(c) ∫ g(x) dx – ∫ g(x) dx = 5 – [ ]


4 x 4 2 4

[ y3 – 4y ]
3 2
0 2 0 0 =
–1
=5–8
= –9 unit 2
(negatif menunjukkan kawasan
= –3 berlorek berada di kiri paksi-y)
= 9 unit 2
∫ ∫ ∫
4 4 4
(d) [g(x) + 1] dx = g(x) dx + 1 dx
0 0 0 (c) Bahagikan kawasan berlorek kepada 2 bahagian
=5– x [] 4
0
iaitu luas A dan luas B


1
=5+4 Luas A =
0
x(x – 1)(x – 2) dx
=9
=∫
1
(x3 – 3x2 + 2) dx
∫ ∫ ∫
7 4 7 0
(e) g(x) dx = g(x) dx + g(x) dx


0
=5+7
0 4

4[
= x – x3 + 2x
4
] 1
0
= 12
= 1 unit 2
4
∫ ∫
4 4
(f) kx dx + 3g(x) dx = 10

1
0 0 Luas B = x(x – 1)(x – 2) dx
k[ ] + 3 ∫
x 2 4 0

4
g(x) dx = 10
=∫
1
2 0 0 (x3 – 3x2 + 2) dx
0
8k + 3(5) = 10
k=–5
8

4 [
= x – x3 + 2x
4 1
0
]
1

2
4. (a) Luas = 3x2 dx = – unit (negatif menunjukkan
2
4
1 kawasan berlorek berada
= x3 [ ] 2
1
= 1 unit 2
di kiri paksi-y)

= 7 unit 2
4

(b) ∫ (x – 2x) dx = [ x ]
2 3 2 º Luas = Luas A + Luas B
2
– x2
0 3 0
= 1 + 1
(
= 8 –4 –0
3 ) 4 4
= 1 unit 2
= – 4 unit 2
3
2

23

Jw01 Full SpotlightA+ MateTambahan Tg5.indd 23 26/10/2022 3:40 PM


Matematik Tambahan Jawapan

6. (a) 2x2 + 1 = 4 – x

9
2x + x – 3 = 0
2 (b) Isi padu = π x2 dy
5
(2x + 3)(x – 1) = 0
= π∫
9
(9 – y) dy
x = 1, y = 4 – 1 = 3 5
A(1, 3), B(0, 4)
(b) Luas trapezium = 1 (4 + 3)(1) = 7 unit 2
[
= π 9y –
y2
2 ]
9
5
2 2 = 8π unit3

1

2
Luas di bawah lengkung = (2x2 + 1) dx (c) Isi padu = π (6y – y2)2 dy
0 0

[
= 2x + x
3
3
]1
0 = π∫
2
0
(36y2 – 12y2 + y4) dy
= 5 unit 2
3 [
= π 12y3 – 3y4 +
y5
]
2

Luas berlorek = 7 – 5 5 0
2 3 = 54.4π unit3
= 11 unit 2

2
6 (d) Isi padu = π y2 dx
0
7. (a) 6x – x2 = 6 – x
= π∫
2
x2 – 7x + 6 = 0 (x2)2 dx
0
(x – 6)(x – 1) = 0
= π∫
2
x = 1, y = 6 – 1 = 5 Q(1, 5) x 4 dx
0
x = 6, y = 6 – 6 = 0 P(6, 0)
(b) Luas segi empat tepat = 1 (5)(5) = 25 unit 2
=π x [ ]
5 2

5 0
2 2
1
Luas di bawah lengkung = (6x – x2) dx ∫ = 32 π unit3
5
0

= 3x2 – x
3
3
[ ] 1
0
Zon Formatif 3.4

= 8 unit 2
3 1. (a) ds = ht + 2,
dt
Luas kawasan berlorek = 25 + 8 2h + 2 = 14
2 3
h = 6
1
= 15 unit 2 ds = ht + 2, 2h + 2 = 14
6
dt

3 h=6
8. (a) Isi padu = π (x – 2) dx
2
º ds = 6t + 2

2 [
= π x – 2x
3
2
2
]
dt

s = (6t + 2) dt
= 1 π unit3
2 s = 3t2 + 2t + c
t = 2, s = 8, 8 = 3(2)2 + 2(2) + c

(b) Isi padu = π y2 dx
c = –8
º s = 3t2 + 2t – 8
= π ∫ (! 3x + 2 ) dx
5 2

1 (b) 3t2 + 2t – 8 = –3
= π ∫ (3x + 2) dx
5
3t2 + 2t – 5 = 0
1 (3t + 5)(t – 1) = 0
[
= π 3x + 2x
2
2 5
1
] t = – 5, t = 1
3
= 44π unit3 2.
y


1
(c) Isi padu = π (4 – x ) dx
2 2
9
–1

= π  ∫
1
(16 – 8x2 + x 4) dx
–1

[
= π 16x – 8 x3 + x
3 5
5
]1
–1 –3 0 3
x

= 27 1 π unit3
15

3
(a) Luas = y dx

3
9. (a) Isi padu = π (y – 2) dy –2
2
= ∫
3

[ y2 – 2y] (9 – x2) dx
2 3
=π –2
2

= 1 π unit3
2
= 9x – x
3
3
[ ]3
–2

= 36 unit 2

24

Jw01 Full SpotlightA+ MateTambahan Tg5.indd 24 26/10/2022 3:40 PM


Jawapan Matematik Tambahan

(b) Isi padu = π y2 dx ∫ (b) ∫


3
4 dx – ∫
3
h(x) dx = 4x – 3 [ ] 3
1 1 1
= π∫
3
(9 – x2)2 dx = [4(3) – 4(1)] – 3
–3
=8–3
= π∫
3
(81 – 18x2 + x 4) dx =5
–3

[
= π 81x – 6x3 + x
5
5
]
3
–3
2. ∫
p
3
f(x) dx + ∫
p
3
5 dx = 9


dy
= 259.2 π unit3 4 + 5x [ ] p
3
=9
3. (a) = x2 – px – q = 0 4 + (5p – 15) = 9
dx
5p + 11 = 9
(1, 1) (2, 3)
5p = 20
1–p–q=0 4 – 2p – q = 0
p=4
p + q = 0 …1 2p + q = 4 …2
∫ ∫
5 1
Selesaikan 1 dan 2: 3. (a) g(x) dx = – g(x) dx
1 5
p = 3, q = –2
dy = –10
(b) = x2 – 3x + 2
dx

5
(b) [g(x) – k] dx = 26
y = (x – 3x + 2) dx
∫ 2 1

∫ ∫
5 5
g(x) dx – k dx = 26
= x – 3 x 2 + 2x + c
3
1 1
3 2
Gantikan mana-mana titik (1, 1) atau (2, 3) 10 – kx [ ] 5
1
= 26
º y = x – 3 x 2 + 2x + 1
3
10 – 4k = 26
3 2 6
4k = –16
4. u = x v=x+3
du = 1 dv = 1 k = –4
8
dx
dy
dx
(x + 3)(1) – (x)(1)
4. ∫
(px – 5)n
dx = –2(px – 5)2 + c
= 8(px – 5)–n + 1
dx (x + 3)2 = –2(px – 5)2
= 3 (–n + 1)p
(x + 3)2 Dengan perbandingan,
∫ 3
(x + 3)2
dx = x
x+3
–n + 1 = 2 …1
n = –1
8
∫ 6 dx = 2 3

3 3
º dx = –2 …2
5(x + 3)2
2 5 2 (x + 3)2 (–n + 1)p
8
= 2 x 3
5 x+3 2 [ ] (–(–1) + 1)p
= –2, gantikan n = –1
4p = 8
= 1
25 p=2
5. (a) x + 1 = 2 + x – x2
x2 – 1 = 0
5. ∫ 2g(x) = ∫ dy
∫ 2g(x) dx = [ 4 3x– x ]
2 2
(x + 1)(x – 1) = 0
1 1
x = –1, x = 1
x = 1, y = 1 + 1 = 2
A(–1, 0), B(1, 2)
= (
3(2)
4–2

3(1)
4–2 ) ( )
=2
(b) Luas rantau berlorek

2
g(x) dx = 1

1
= (2 + x – x2) dx – 1 (2)(2) 1
–1 2

b
[
= 2x + x – x
2
2 3 1

3 –1 ] 6.
a
2f(x) dy = 2(–9)
= –18
1
=3 –2

k
3 7. (2x – 8) dx = –9
1
= 1 1 unit 2
3 [ x – 8x ]
2
2
1
= –9

Zon Sumatif k – 8k – (1 – 8) + 9 = 0
2

k2 – 8k + 16 = 0
(k – 4)(k – 4) = 0
∫ ∫
3 1
1. (a) h(x) dx = – h(x) dx
1 3 k=4
= –3

25

Jw01 Full SpotlightA+ MateTambahan Tg5.indd 25 26/10/2022 3:40 PM


Matematik Tambahan Jawapan

8. dv = 2 – 5t 12.
dt y

v = (2 – 5t) dt (5, 8)

= 2t – 5t + c
2

2
Diberi v = 4 apabila t = –2,
(2, 3)
5(–2)2
4 = 2(−2) − +c
2
x
4 = −4 − 10 + c O
c = 18

3
Luas = x dy
1
Maka, v = 2t − 5t2 +18.
=∫
3
dy (y2 + 1) dy
9. = P +1 1
dx x2
= 10 2 unit 2
x ∫
y = P2 + 1 dt
dy
3
13. (a) = kx – 5, (–2, 2), (0, 0)
y=–P +x+c dx
x
dy Tangen pada (–2, 2), m = 2 = –1
Diberi = 3 dan x = 1 –2
dx k(–2) – 5 = –1
P +1=3 –2k = 4
x2 k = –2
P +1=3 dy
(1)2 (b) = –2x – 5
dx
P=2
Maka, 3 = – 2 + (1) + c

y = (–2x – 5) dx
(1) y = –x2 – 5x + x
c=4
(–2, 2)
Bila x = 4, y = – 2 + (4) + 4 +2 = –(–2)2 – 5(–2) + c
(4)
2 = –4 + 10 + c
y = 15 c = –4
2
10. (a) Diberi fungsi kecerunan, f'(x) = 3x2 − 6x º y = –x2 – 5x – 4
Pada titik (2,−4), f'(2) = 3(2)2 − 6(2) d 2y
14. = 16x + 4
f'(2) = 0 dx2
(b) f'(x) = 3x2 − 6x dy
dx ∫
= (16x + 4) dx
f(x) = 3x2 − 6x dx
∫ dy 16x2
= + 4x + c
= 3x – 6x
3 2
dx 2
3 2 = 8x + 4x + c
2
= x3 – 3x2
dy
11. (a) Diberi = 6 dan x = –1,
dx
y 6 = 8(–1)2 + 4(–1) + c
(5, 8) c=2
dy
Maka, = 8x2 + 4x + 2.
dx
dy
= 8x2 + 4x + 2
dx
(2, 3)

y = 8x2 + 4x + 2 dx
= 8x + 4x + 2x + c
3 2
x
O 3 2
y = 8x + 2x2 + 2x + c
3


8
x dy 3
3
Diberi y = 1 dan x = 1,
∫ ∫
5 8
(b) y dx + x dy = 40 – 6 8(1)3
2 3 1= + 2(1)2 + 2x + c
= 34 3
c = – 17
3
Maka, y = 8 x3 + 2x2 + 2x – 17 .
3 3

26

Jw01 Full SpotlightA+ MateTambahan Tg5.indd 26 26/10/2022 3:40 PM


Jawapan Matematik Tambahan

15. Dengan menggunakan petua bahagi.


Katakan u = x dan v = x + 7 2 [
18. Luas rantau = 1 (1 + 4)(3) + (x – 2)2 dx
3
0
] ∫
du = 1 dv = 1
dx dx
= 15
2
– x
3
3
– 4x
2 [
2
+ 4x
3
0
]
dy (x + 7)(1) – (x)(1) 15
= = –3
dx (x + 7)2 2
=
7 = 4 1 unit 2
(x + 7)2 2
1
∫ ∫
–2 –2
–4 (x + 7)2
dx =
–4
(x + 7)–2 dx
2 [
19. Luas rantau = 1 (1 + 4)(3) + (x – 2)2 dx
3
0
] ∫
=
–1[
(x + 7)–1 –2
–4
] = 15
2
– x
3
3
– 4x
2[2
+ 4x
3
0
]
= – 1
[ –2

x + 7 –4 ] = 15
2
–3

= –1 – –1
5 [( ) ( )]
3
= 4 1 unit 2
2
2
=
15 2 [ 0
] ∫
20. Luas rantau = 1 (9)(1) + 9y3 dx
1

16. (a) y + x = 5
y = –x + 5
9
= –
2
9y2 3
4 0 [ ]
Kecerunan, m = –1 9 9
Oleh sebab kecerunan adalah selari, = –
2 4
dy
dx
= kx2 − 4x dan melalui titik (1, 3), = 2 1 unit 2
4
kx2 − 4x = −1
∫ (! 4 – x )
2
2 2
k(1)2 − 4(1) = −1 21. Isi padu janaan = π dx
0
k = −1 + 4
= π ∫ 4 – x dx
2
2
k=3 0
dy
(b) Gantikan k = 3,
dx
= 3x2 − 4x
[
= π 4x – x
3
3
]
2
0
Persamaan lengkung
= 16 π unit3

y = 3x2 − 4x 3


2
y = 3x – 4x + c 22. Isi padu janaan = π (y + 1) dx
3 2
–1
3 2
[ y2 + y]
2 2
y = 3x2 − 2x2 + c =π
–1
Melalui titik (1, 3),
3 = (1)3 − 2(1)2 + c = 9 π unit3
2
c=4
23. Isi padu janaan = 3π
Maka, persamaan lengkung ialah y = x3 – 2x2 + 4.
∫ ( )
P 2 2
17. Titik persilangan ialah (3, 9) π dx = 3π
1 x

y
[ ]
π – 4 = 3π
x 1
P

y = x2 – 4 – (–4) = 3
15 P
– 4 = –1
(3, 9) P
y = 15 – 2x P=4
x 24. Isi padu janaan = 4π
O 15
––
∫ ( 4y )
4 2
2 π dy = 4π
k

2 2 [ (
Luas rantau = 1 15 – 3 (9) + ) ] ∫ x dx
3
0
2
[ ]
π – 16 = 4π
y k
4

2 [
= 1 (4.5)(9) + x
3
3
] [ ]3
0 ( )
–4­ – – 16 = 4
k
= 20 1 + 9 16 = 8
4 k
1
= 29 unit 2 k=2
4

27

Jw01 Full SpotlightA+ MateTambahan Tg5.indd 27 26/10/2022 3:40 PM


Matematik Tambahan Jawapan

25. Diberi x2 + y2 = 4 x(5 − x) = 0


Apabila x = 0, y2 = 4 x = 0, 5
y = ±2 Maka, titik persilangan adalah pada (0, 0) dan
y = 2, −2 (Ditolak) (5, 15).
∫ ∫
2 5
Isi padu janaan, m = π 4 – y2 dy Luas kawasan yang dibatasi = 5x – x2 dx
0 0
y3
3 [
= π 4y – ]
2
0
[
= 5x – x
2
2 3 5

3 0 ]
= 16 π unit3
3 = [
5(5)2 (7)3
2

3
–0 ]
Maka, nilai m ialah 16 π.
3 = 125 unit 2
Kertas 2 6
4. (a) y = 162 …1
1. (a) y = 10x − x2 …1 x
y = 21 …2 y = 2x …2
1 = 2: 10x − x2 = 21 1 = 2: 162 = 2x
10x − x2 − 21 = 0 x
10x − x2 − 21 = 0 2x3 = 16
(x − 3)(x − 7) = 0 x3 =8
x = 3, 7 x=2
Maka, koordinat A dan B masing-masing ialah Apabila x = 2, y = 2(2)
(3, 21) dan (3, 7). =4
Maka, koordinat A ialah (2, 4).
(b) Luas kawasan yang dibatasi
(b) Luas kawasan yang dibatasi

7
= (10x – x2) – 21 dx
= 1 (2)(4) – 162 dx ∫
4
3

=∫
7 2 2 x
10x – x2 – 21 dx
3
=4+ – 2 16 4
[ ]
[
= 5x2 – x – 21x
3
3 7
3
] x 2
= 4 + (–4 – (8))
[ (7) (3)3
] [ ] = 4 – 12
3
= 5(7)2 – – 21(7) – 5(2)2 – – 21(3)
3 3 = –8
= 32 unit 2 = 8 unit 2
3
∫ ( 16x ) dx
4 2
2. (a) x = (y − 3)2 …1 (c) Isi padu janaan = 2
2
x = 1 …2
= π ∫ 256 dx
4
1 = 2: (y − 3)2 = 1
x2 4
y2 − 6y + 9 = 1
y2 − 6y + 8 = 0 [ ]
= π – 2563
3x 2
4

(y − 2)(y − 4) = 0
y = 2, 4 [( ) (
= π – 2563 – – 2563
3(4) 3(2) )]
Maka, koordinat P dan Q masing-masing ialah = 28 π unit3
(1, 2) dan (1, 4). 3
5. (a) y = 9 − x2 …1
(b) Luas kawasan yang dibatasi
x + y = 7 …2

4
= (y – 3)2 – 1 dx 1 = 2: 9 − x2 = 7 − x
2
2 + x − x2 = 0
=∫
4
y2 – 6y + 8 dx (x + 1)(x − 2) = 0
2
x = −1, 2
[ y3 – 3y + 8y]
3 4
= 2
Apabila x = 2, y = 9 − (2)2
2
=5
= [ 4 – 3(4) + 8(4)] – [ 2 – 3(2) + 8(2)]
3 3
2 2
3 3 Apabila x = –1, y = 9 − (−1)2
=8
= –4
3   Maka, koordinat P dan Q masing-masing ialah
4 (2, 5) dan (–1, 8).
= unit 2
3 (b) Luas kawasan berlorek
3. y = x(8 − x) …1

2
y = 3x …2 = 2 + x – x2 dx
–1
1 = 2: x(8 − x) = 3x
8x − x2 − 3x = 0 [
= 2x + x – x
2
2

3
3
]2
–1
5x − x2 = 0

28

Jw01 Full SpotlightA+ MateTambahan Tg5.indd 28 26/10/2022 3:40 PM


Jawapan Matematik Tambahan

[
= 2(2) +
(2)2 (2)3
2

3 ] [
– 2(–1) +
(–1)2 (–1)3
2

3 ] (b) P1 = 6!
6
2!
= 9 unit 2 = 6 × 5 × 4 × 3 × 2!
2 2!
= 360

9
(c) Isi padu janaan = π 9 – y dy
0
(c) nP0 = n!
= π 9y – [y3 9
3 0 ] (n – 0)!
= n!
[
= π 9(9) –
(9)3
2
–0 ] = 1
n!

= 81 π unit3 (d) 6! = 6 × 5 × 4!
2 4! 4!
6. (a) Diberi fungsi kecerunan = 5 – 6x = 30
Persamaan lengkung, y = 5 − 6x ∫ (e) 100! = 100 × 99 × 98!
98!2! 98!2 × 1
y = 5x − 6x + c
2

2 = 4 950
y = 5x − 3x2 + c (n – 1)!
(f) n – 1P0 =
Pada titik P(0, 2), 2 = 5(0) − 3(0)2 + c (n – 1 – 0)!
c=2 =1
Oleh itu, persamaan lengkung ialah 6. (a) 5! = 120
y = 5x − 3x2 + 2.
(b) 8! = 20 160
Menggunakan rumus, x = − b 2!
2a
(c) 6! = 720
x=− 5
2(–3) 7. (a) (7 – 1)! = 6!
x= 5 = 720
6 (b)
5
Apabila x = , y = 5
6
5
6 ( ) ( )
−3 5 +2
6
2

y = 49
12
Maka, a = 5 dan b = 49 .
6 12
(b) Melalui P(0, 2),
Kecerunan tangen, y' = 5 – 6x
= 5 – 6(0) 2! × 4! = 48
=5 8. (a) 3 × 3 × 3 = 27
mnormal × mtangen = −1 (b) 3P3 = 3 × 2 × 1 = 6
mnormal × 5 = −1 9. (a) (i) 5 × 5 × 5 × 5 × 5 = 3 125
mnormal = − 1 (ii) 5 × 5 × 5 = 125
5 (b) (i) a. 5 × 4 × 3 × 2 × 1 = 120
Persamaan normal, b. 5 × 4 × 3 = 60
y − y1 = mnormal(x − x1)
10. (a) KEMANUSIAAN
y − 2 = − 1 (x − 0)
11! = 3 326 400
5 3!2!
y = – 1 x + 2
5 (b) KEMANUSIAAN
11! = 1 663 200
Bab 4 Pilih Atur dan Gabungan 2!3!2!
11. (a) (3 – 1)! = 2!
Zon Formatif 4.1 =2
(b) (6 – 1)! = 5!
1. Bilangan padanan = 3 × 4 = 12 = 120
2. Bilangan padanan = 3 × 2 = 6 12. (a) P5 =
5 5!
0!
3. Bilangan padanan = 3 × 3 × 2 = 18 = 120
4. Bilangan padanan = (2 × 2) + (2 × 2) = 8 cara (b)
5. (a) 4! = 4 × 3 × 2 × 1 P
= 24 1 4
P4

29

Jw01 Full SpotlightA+ MateTambahan Tg5.indd 29 26/10/2022 3:40 PM


Matematik Tambahan Jawapan

Bilangan cara = 1 × 4 P4 • 6 perempuan dan 1 lelaki


= 1 × (4 × 3 × 2 × 1) 6
C6 × 6C1
= 24 Bilangan cara
(c) = (6C4 × 6C3) + (6C5 × 6C2) + (6C6 × 6C1)
O/A = 300 + 90 + 6
= 396
2!
3! 7. 10
C10 × 6C3 × 3C2 = 60
Bilangan cara = 2! × 3! × 4
=2×3×2×4 Zon Sumatif
= 48
Kertas 1
Zon Formatif 4.2 1. (a) P4 = 5!
5
1!
= 120
1. (a) C2 = 12!
12
2!10! (b) 4 P4 = 24
= 66 2. (a) 7! = 5 040
(b) C1 × C2 = 5! × 8!
5 8 (b)
1!4! 2!6! Perempuan
= 140
(c) 4C1 × 3C1 × 3C1 = 36

2. (a) C3 = 7!
7
4!3!
= 35
(b) 5C3 = 5!
3!2!
= 10
(c) 6C3 = 6!
3!3!
= 20
3. (a) 9C7 = 9! 3
P3 4
P4
7!2!
= 20 Bilangan cara = P4 × P3 × 4 4 3

= 576
(b) C2 × 5C3 = 4! × 5!
4
2!2! 3!2! 3. 8
C6 × C4 × C3 = 5 880
7 4

= 60
4. Bilangan perempuan , Bilangan lelaki
(c) 4C1 × 3C3 × 5C1 = 4! × 3! × 5! • 5 lelaki dan 3 perempuan
1!3! 3!0! 3!2!
= 40
6
C 5 × 5C 3
• 6 lelaki dan 2 perempuan
4. (a) 6C3 × 3C3 = 20 6
C 6 × 5C 2
(b) 2C2 × 4C1 × 3C3 = 4
(c) Bilangan cara “mereka tidak bersama” Bilangan cara = (6C5 × 5C3) + (6C6 × 5C2)
= Jumlah cara – “Mereka bersama” = 60 + 10
= 20 – 4 = 70
= 16 5.
5. (a) 9C5 = 9! 3/6/7
5!4! 3
P1
= 126 4
P3
(b) 7C5 = 7! Bilangan cara = 3P1 × 4 P3
5!2!
= 72
= 21
(c) (4C3 × 5C2) + (4C4 × 5C1) 6. (a)
= 40 + 5 O
= 45
1
6. Bilangan perempuan . Bilangan lelaki 5
P5
• 4 perempuan dan 3 lelaki Bilangan cara = 5P5 × 1P1
6
C 4 × 6C3 = 120
• 5 perempuan dan 2 lelaki
6
C 5 × 6C2

30

Jw01 Full SpotlightA+ MateTambahan Tg5.indd 30 26/10/2022 3:40 PM


Jawapan Matematik Tambahan

(b) (b)
Huruf vokal disusun bersama
5
P1 5
P5
Bilangan cara = P5 × 5P5 5

= 600
7. (a)
3/4 3/5 A I
2 5 5 2
Bilangan cara = 2 × 5 × 5 × 2
3
P2 2
P2
= 100 Bilangan cara = P2 × P2 × 3
2 3

(b) = 36
3 5 Bilangan huruf vokal tidak boleh disusun
bersama
1 3
P2 1 = 120 – 36
= 84
atau
2. (a) (i) 9 × 10 × 10 = 900
4 3/5 (ii) 9 × 9 × 8 = 648
(b) (i)
1 3
P2 1 0/5
(3P2 × 1) + (3P2 × 2P1) 9 10 2
P1
= 6 + 12 Bilangan cara = 9 × 10 × 2P1
= 18 = 9 × 10 × 1
8. (a) 7P7 = 5 040 = 180
(b) 2P2 × 5P5 × 2! = 2 × 120 × 2 (ii)
= 480 0
9. 2
P2 × 2P2 × 5P5 × 3! = 48 9
P1 8
P1 1
10. (a) Didapati huruf A diulang 3 kali = 7! Bilangan cara = 9P1 × 8P1 × 1
3!
= 72
= 840
(b) 5
8
P1 8
P1 1
4
P2 P3
3 Bilangan cara = P1 × P1 × 1
8 8

3! = 64
Bilangan cara = 72 + 64
A diulangi 3 kali = 136
3. (a) P1 =
8 18!
= 12 × 120 10!8!
6 = 43 758
= 240
(b) 11C4 × 7C4 = 11! × 7!
11. (6 – 1)! = 5! 7!4! 3!4!
= 120 = 11 550
12. 7
C6 + 6C5 + 5C4 + 4C3 + 3C2 + 2C1 + 1C1 (c) • 0 perempuan dan 8 lelaki
=7+6+5+4+3+2+1 7
C 0 × 11C8
= 28 • 1 perempuan dan 7 lelaki
11
C7 × 7C1
Kertas 2
Bilangan cara = (7C 0 × 11C8) + (11C7 × 7C1)
1. (a) P4 = 5!
5
= 165 + 2 310
1!
= 120 = 2 475
Sekurang-kurangnya 2 perempuan
= 43 758 – 2 475
= 41 283

31

Jw01 Full SpotlightA+ MateTambahan Tg5.indd 31 26/10/2022 3:40 PM


Matematik Tambahan Jawapan

(d) Bilangan lelaki . Bilangan perempuan (c)


• 5 lelaki dan 3 perempuan Perempuan duduk
11
C 5 × 7C 3 bersama
• 6 lelaki dan 2 perempuan
11
C 6 × 7C 2
• 7 lelaki dan 1 perempuan
11
C7 × 7C1
• 8 lelaki dan 1 perempuan
11
C 8 × 7C 0
Bilangan cara = (11C5 × 7C3) + (11C6 × 7C2) +
(11C7 × 7C1) + (11C8 × 7C 0)
= 16 170 + 9 702 + 165 + 2 310
= 28 347
4. (a) 26 × 10 × 10 × 10 × 10 = 260 000 3
P3 2
P2
(b) 26 × 10P4 = 131 040
(c) (i) 5 × 10P4 = 25 200 Bilangan cara = P3 × P2 × 4 3 2

(ii) 26 × 5P4 = 3 120 = 48


Maka, bilangan perempuan tidak duduk
5. (a) 5
P5 = 120
bersama = 120 – 48
(b)
= 72
2/6/8 7. (a) 7P7 = 5 040
(b) (i)
3
P1 3!
2
P1
E/O/A
Bilangan cara = P1 × 3! × P1
3 2
3
P2 5
P3
2
P1
= 36
(c) Bilangan cara = 3P2 × 5P3 × 2P1
Digit 2 dan 5 disusun = 360
bersebelahan (ii)

4
P2 4
P2
Bilangan cara = P2 × P3 × 3 4 3

= 216
3! 2! (c) 7C4 = 35
Bilangan cara = 3! × 2! × 4 8. (a) 6
C4 × 7C4 = 525
= 48
(b) • Dua-dua dipilih
Maka, bilangan cara digit 2 dan 5 tidak 6
C 4 × 2 C 2 × 5C 2
disusun bersebelahan = 120 – 48
• Dua-dua tidak dipilih
= 72 6
C 4 × 5C 4
6. (a) 5! = 120 Bilangan cara = (6C4 × 2C2 × 5C2) + (6C4 × 5C4)
(b) = 150 + 75
3 lelaki 2 perempuan = 225
(c) Dalam bentuk pasangan, iaitu 4 lelaki dan 4
perempuan
Bilangan cara = 6C3 × 3C2 × 2C1
= 24
9. (a)
2
P2 3
P3 6/9
Bilangan cara = P3 × P2 × 2
3 2 2
P1
= 24
5
P4

32

Jw01 Full SpotlightA+ MateTambahan Tg5.indd 32 26/10/2022 3:40 PM


Jawapan Matematik Tambahan

Bilangan cara = 2P1 × 5P4 5. (a)


= 240 X=r 0 1 2 3 4
(b)
1 1 3 1 1
6/9 1/5/9 P(X = r)
16 4 8 4 10

5
P4
3
P1
P (X = x)

Bilangan cara = P1 × P4 3 5
3

= 360 8
1

(c) Kurang daripada 40 000 bermaksud digit 1 4
atau 2 perlu diletakkan di hadapan
Jika digit 1 di hadapan, 1
––
16
1 5/9 x
0 0 1 2 3 4
1 4
P3
2
P1
(b)
Bilangan cara = 1 × P3 × P1 4 3
X=r 0 1 2 3 4
= 48 1 2 1 3 1
P(X = r)
10 10 5 10 10
Jika digit 2 di hadapan,
2 1/5/9 P
3
––
1 4
P3
3
P1 10

Bilangan cara = 1 × 4 P3 × 3P1 2


––
= 72 10
Maka, bilangan cara = 48 + 72
= 120 1
––
10
Bab 5 Taburan Kebarangkalian
x
0 0 1 2 3 4
Zon Formatif 5.1
5. (a) n = 3, p = 0.36, q = 0.64
1. (a) {0, 1, 2} Katakan X mewakili bilangan murid aliran
(b) Terima mana-mana jawapan yang betul Sains
(c) Terima mana-mana jawapan yang betul P(X = 0) = 3C 0(0.36)0(0.64)3 = 0.2621
(d) {4, 5, 6, 7} P(X = 1) = 3C1(0.36)1(0.64)2 = 0.4424
(e) {2, 3} P(X = 2) = 3C2(0.36)2(0.64)1 = 0.2488
2. (a) Pemboleh ubah rawak diskret P(X = 3) = 3C3(0.36)3(0.64)0 = 0.0467
(b) Pemboleh ubah rawak selanjar
(c) Pemboleh ubah rawak diskret P(X = x)
(d) Pemboleh ubah rawak diskret
(e) Pemboleh ubah rawak selanjar 0.5

3. (a) Set B: {Mendapat nombor 1, 3 atau 5}


(b) Gambar rajah pokok: 0.4
1
1
3 1 0.3
3 3

1 0.2
3
5
(c) 0.1
r 1 3 5
1 1 1 x
P(B = r)
3 3 3 0 0 1 2 3

33

Jw01 Full SpotlightA+ MateTambahan Tg5.indd 33 26/10/2022 3:40 PM


Matematik Tambahan Jawapan

5. Biarkan X menjawab soalan aneka pilihan dengan


(b) n = 4, p = 1 , q = 1 betul.
2 2
Katakan X mewakili bilangan nombor ganjil X ~ B(3, 0.2)
yang diperoleh (a) P(X = 3) = 3C3(0.2)3(0.8)
(
P(X = 0) = 4C 0 1 1 = 1
0

2 2
4
)(
16 ) = 0.008
(b) P(X > 2) = P(X = 2) + P(X = 3)
P(X = 1) = C1
4 1
(
1 1 3

2 2
= )(
1
4 )

= 0.096 + 0.008
= 0.104
P(X = 2) = C2
4 1
(
2 1 2

2 2 )(
= 3
8 ) (c) P(X , 2) = P(X = 0) + P(X = 1)
= 0.512 + 0.384
(
P(X = 3) = 4C3 1 1 = 1
3

2 2
1
)(
4 ) = 0.896
6. (a) X = bilangan orang yang bermata hijau
P(X = 4) = C4
4 1
(
4 1 0

2 2 )(
= 0
16 ) (b)
X P(X = r)
P(X = x)
0 0.8179
3

8 1 0.1652

2 0.01586
1

4 3 0.00096

4 0.00004125

5 ≈0
1
––
16 6 ≈0
x
0 0 1 2 3 4 . ≈0

. ≈0

Zon Formatif 5.2 20 ≈0


Selepas x = 4, kebarangkalian adalah semakin
1. (a) Bukan taburan binomial menghampiri 0. Ini menunjukkan bilangannya
(b) Taburan binomial adalah sangat kecil.
2. Biarkan pemboleubah rawak X supaya (c)
X = Bilangan kepala

(
X ~ B 10, 1
2 ) 0.8179
P(X = 5) = 10C5 1 1
5

2 2
5
( )( )
= 63
256
= 0.2461
3. ‘Kejayaan” diwakili oleh sekolah yang menawarkan
makanan tengah-hari berserta buah-buahan pada
setiap hari.
0.1652
4. Biarkan X = bilangan orang yang hidup.

( )
X ~ B 5, 2
3
0.0158
0 0 1 2
(a) P(X = 5) = 5C5 2 1
5

3 3
0
( )( ) (d) m = np = 20 × 0.01

= 32 = 0.2
243
Dijangkakan secara puratanya daripada 20
(b) P(X > 3) = P(X = 3) + P(X = 4) + P(X = 5)
orang, kurang dari seorang yang bermata hijau
= 0.3292 + 0.3292 + 0.1317
Varians = npq
= 0.7901
= 20 × 0.01 × 0.99
(c) P(X = 2) = 5C2 2 1
2

3 3
3
( )( ) = 0.198
= 0.1646 Sisihan piawai = ! 0.198 = 0.445

34

Jw01 Full SpotlightA+ MateTambahan Tg5.indd 34 26/10/2022 3:40 PM


Jawapan Matematik Tambahan

7. Jika X ialah bilangan kemenangan, maka X (b)


mempunyai nilai 0, 1, 2, 3, …, 20. 73 85 0.91
8.9485
Kebarangkalian ‘kejayaan’, p = 0.55.
Kebarangkalian ‘kegagalan’, q = 1 – p (d)
93 5 0.90
= 0.45 99.41
Bilangan percubaan, n = 20.
Kebarangkalian untuk soalan boleh ditulis sebagai (
3. (a) P(X = 14) = P Z = 14 – 18
5 )
P(X = 15). = P(Z = –0.8)
Maka, skor-z = –0.8
8. n = 30, p = 1
3 (b) Minuman ini 0.80 bersisihan piawai sebanyak
Nilai jangkaan, m = np 32 minuman di bawah min.
= 10 (c) z = –0.5 dalam Java’s Roasts. Minuman
3 ini berada di bawah min StarBuck
s 2 = npq
memandangkan skor-z adalah lebih rendah.
= 10 × 2 Minuman ini mempunyai kandungan gula
3 3
yang lebih rendah daripada StarBuck.
= 20
9 4. (a)
Sisihan piawai, s = 20
9 ! f(z)
= 1.4907
9. X ~ B 10, 3(
5 )
(a) (i) P(X = 6) = 0.2508
(ii) P(X , 9) = 1 – P(X = 9) – P(X = 10)
= 1 – 0.04031 – 0.0060466 z
0 2.375
= 0.9536
(b) Biarkan Y mewakili bilangan pelanggan yang
tidak memerlukan air. P(Z , 2.375) = 1 – P(Z . 2.375)
(
Y ~ B 50, 2
5 )

= 1 – 0.0088
= 0.9912
Jika P(X , n) > 9, (b)
P(Y . 50 − n) > 0.9 f(z)
1 − P(Y < 50 − n) > 0.9
P(Y < 50 − n) < 0.1
50 − n < 15
n > 35
Maka, nilai terkecil n ialah 35.
z
Zon Formatif 5.3 0 2.375

P(Z . 2.375) = 0.0341


(
1. (a) P(X = 80) = P Z = 80 – 350
100 ) (c)
= P(Z = –2.7) f(z)
Maka, skor-z = –2.7.
(
(b) P(X = 68) = P Z = 68 – 67
5 )
= P(Z = 0.2)
Maka, skor-z = –0.2.

2 (
(b) P(X = 43) = P Z = 43 – 43 ) –1.182 0 2.144
z
= P(Z = 0)
Maka, skor-z = 0. P(–1.182 , Z , 2.144)
= P(Z . –1.182) – P(Z . 2.144)
2.
= [1 – 0.1186] – 0.016
Min Sisihan piawai X P = 0.8814 – 0.016
(c) = 0.8654
85 4.5 0.68
87.12
(a)
1 16 0.05
17.645

35

Jw01 Full SpotlightA+ MateTambahan Tg5.indd 35 26/10/2022 3:40 PM


Matematik Tambahan Jawapan

(d) P(1.254 , Z , 2.224)


f(z)
= P(Z . 1.254) – P(Z . 2.224)
= 0.1049 – 0.0131
= 0.0918
5. (a)
f(z)
z
–1.081 0

P(Z , –1.081) = P(Z . 1.081) 0.2467


= 0.1398
(e)
f(z) z
a 0

P(Z , a) = 0.2467
a = –0.685
(b)
z f(z)
–2.711 0

P(Z . –2.711) = 1 – P(Z . 2.711)


= 1 – 0.0034 1.043
= 0.9966
(f)
f(z) z
0 a

P(Z . a) = 1.043
a = 1.043
(c)
f(z)
z
–2.0179 0 2.0179
0.7717
P(|Z| . 2.0179)
= P(–2.0179 . Z . 2.0179)
= 2 × P(Z . 2.0179)
= 2 × 0.0218 z
= 0.0436 –1.18 0 a
(g)
f(z) P(–1.18 < Z < a) = 0.7717
P(Z . –1.18) – P(Z . a) = 0.7717
[1 – 0.119] – P(Z . a) = 0.7717
P(Z . a) = 0.1093
a = 0.123
(d)
z f(z)
–1.625 0 1.625

P(|Z| , –1.625) 0.833


= P(–1.625 , Z , 1.625)
= P(Z . –1.625) – P(Z . 1.625)
= [1– 0.0521] – 0.0.0521
= 0.9479 – 0.0521 z
–1.382 0 a
= 0.8958
(h) f(z) P(|Z| , a) = 0.833
P(–a , Z , a) = 0.833
P(Z . –a) – P(Z . a) = 0.833
[1– P(Z . a)] – P(Z . a) = 0.833
–2P(Z . a) = –0.167
P(Z . a) = 0.0835
z a = 1.382
0 1.254 2.224

36

Jw01 Full SpotlightA+ MateTambahan Tg5.indd 36 26/10/2022 3:40 PM


Jawapan Matematik Tambahan

(e)
f(z) (iii) P(X , 85) = P Z , 85 – 110
25 ( )
= P(Z , –1)

f(z)
0.0059

z
0 a

P(Z . a) = 0.0059
a = 2.521 z
–1 0
(f)
f(z)
P(Z , –1) = P(Z . 1)
0.4856 = 0.15866
(
(b) P Z , a – 110 = 0.1
25 )
6
z z 0 2 Tolak
a0 1.243

P(a , Z , 1.243) = 0.4856 2.3 0.0102 2


P(Z . –a) – P(Z . 1.243) = 0.4856
[1 – P(Z . a)] – 0.1069 = 0.4856 skor-z = 2.326
P(Z . a) = –0.4076 a – 110 = 2.326
a = –0.234 25
20 – m a – 110 = 58.15
6. –1.645 = a = 168.15
4.5
m = 27.4025 Maka, IQ Elizabeth ialah 168.15.
7. 1.282 = 94 – 83 9. (a) P(X . 195) = 0.01606
s = 1.606%
s = 8.5803 (b) P(163 , X , 195)
8. X ~ N(110, 25) = P(−2.429 , z , 2.143)
(a) (i) P(X . 110) = P Z . 110 – 110
25 ( ) = 1 − P(z . 2.143) − P(z . 2.429)
= 1 − 0.01606 − 0.00757
= P(Z . 0) = 0.97637
Daripada sifir taburan normal, = 97.637%
f(z) (c) P(171 , X , 187) = P(−1.286 , z , 1)
= 0.74212
= 74.212%

(
10. P Z > X – 100 = 15%
15 )
z
(
PZ> X –
15 )
100 = 0.15

0 Daripada sifir taburan normal,

P(Z . 0) = 0.5 f(z)


(ii) P(X . 160) = P Z . 160 – 110
25 ( )
= P(Z . 2)
Daripada sifir taburan normal, 0.15

f(z)
z
0 1.04

z 0 4 Tolak
z
0 2 1.0 0.1492
P(Z . 0) = 0.0228

37

Jw01 Full SpotlightA+ MateTambahan Tg5.indd 37 26/10/2022 3:40 PM


Matematik Tambahan Jawapan

skor-z = 1.04 4. X ~ B(4, 0.1)


X – 100 = 1.04 P(X = 0) = 4C 0(0.1)0(0.9)4 = 0.6561
15 P(X = 1) = 4C1(0.1)1(0.9)3 = 0.0486
X – 100 = 15.6 P(X = 2) = 4C2(0.1)2(0.9)2 = 0.0486
X = 115.6 P(X = 3) = 4C3(0.1)3(0.9)1 = 0.0036
11. Pada pendapat saya, sisihan piawai ialah 5 adalah P(X = 4) = 4C4(0.1)4(0.9)0 = 0.0001
lebih baik. Hal ini kerana sisihan piawai ini 5. X ~ B(30, 0.9)
menunjukkan hanya 2.3% daripada populasi yang (a) P(X = 30) = 30C30(0.9)30(0.1)0
menduduki ujian yang mempunyai IQ lebih tinggi = 0.04239
dan Suraya tergolong dalam bilangan minoriti ini. (b) P(X > 28) = P(X = 28) + P(X = 29) + P(X = 30)
Jika sisihan piawai ialah 15, 25.1% daripada populasi = 0.41135
mempunyai IQ yang tinggi dan kelompok ini lebih
besar.
6. p = 4
20 5
1
(
= , X ~ B­ 20, 1 )5
P(X < 4)
(
12. (a) P(X . 170) = P Z . 170 – 160
8 ) = P(X = 0) + P(X = 1) + P(X = 2) + P(X = 3)
= P(Z . 1.25) = 0.01153 + 0.05765 + 0.13691 + 0.20536 + 0.2182
Daripada sifir taburan normal, = 0.61862
Kebarangkalian untuk lebih daripada 4 orang
f(z)
murid yang mahu menggunakan COVID ialah
P(X . 4) = 1 – P(X < 4)
= 0.38138
iaitu, 38% dalam sesuatu masa untuk lebih
daripada 4 orang pelajar mahu menggunakan
perisian. Bahagian pemasaran membuat tanggapan
z
0 1.25 yang keterlaluan terhadap penggunaan lesen
perisian untuk 4 orang.
P(Z . 1.25) = 0.1065
(b) P(170 , X , 180) = P(1.25 , z , 2.5) 7. (a) P(X = 2) = 10C2 1 2

( )( )
3 3
2 8

= P(z . 1.25) – P(z . 2.5) = 0.1951



(c) P(X , h) =
= 0.09944
0.10565
( )(
(b) nCn 1 2 = 1
3 3
n
) 0

243

h = 173 cm
2 (1) ()1
3
n
(1) = 1
243

Zon Sumatif
( 1
3) n
= 1
243

Kertas 1
1 = 15 ( 3) n

1. X ~ B(5, 0.3, ( 1
3) ()
n
= 1
3
5

P(X < 2) = P(X = 0) + P(X = 1) +P(X = 2) n=5


= 0.16807 + 0.36015 + 0.3087 8. (a) X ~ B(10, 0.5)
= 0.83692 (a) P(X = 6) = 10C6(0.5)6(0.5)4
2. (a) m = 60 × 1 = 0.2051
4
= 15 (b) Pelajar perlu meneka dengan tepat untuk
(b) s 2 = 60 × 1 × 3 sekurang-kurangnya 60% daripada soalan,
4 4 iaitu 0.60 × 10 = 6 soalan.
= 11.25 P(X > 6) = 1 – P(X < 5)
s = 3.3541 = 0.3770
3. (a) Kejayaan = Mendapat 6, 9. P(X > x) > 0.95
Kegagalan = Tidak mendapat 6 P(X < 3) = 0.9547
Setiap lambungan berdiri sendiri Peralatan gantian yang diperlukan ialah 3.
Bilangan percubaan tetap, n = 4
10. Diberi m = 7.2 dan n = 8,
Kebarangkalian kejayaan adalah sama, 1 .
6 np = m
X adalah taburan binomial. 8p = 7.2

6( )
X ~ B 4, 1 , n = 4, p = 1
6
p = 0.9
11. np = 90 …1
(b) P(X = 0) = 4C 0 1 1

0

6 6 ( )( )
4

! npq = 3! 7 …2
= 0.4823

38

Jw01 Full SpotlightA+ MateTambahan Tg5.indd 38 26/10/2022 3:40 PM


Jawapan Matematik Tambahan

! npq = ! 9 × 7 X – 60 = – 15
2
! npq = ! 63
X = 52 1 /52.5 kg
npq = 63 …3 2
X–m
15. (a) =Z
Gantikan 1 ke dalam 3: s
(90)q = 63 X – 200 = 1

q = 63 30 2
90 X – 200 = 15
q= 7 X = 215 g
10
p=1–q (
(b) P(X > 194) = P Z . 194 – 200
30 )
=1– 7 = P(Z > –0.2)
10
Berdasarkan jadual sifir,
= 3
10 f(z)
Gantikan p = 3 ke dalam 1
10
( )
n 3 = 90
10
n = 300
Maka, p = 3 , q = 7 , n = 300
10 10 z
12. (a) X = Bilangan tembakan Kelvin yang mengenai –0.2 0
sasaran
P(Z > –0.2) = 0.5 + [1 – P(Z > –0.2)]
X = {0, 1, 2, 3}
= 0.5 + 0.0793
(b) X ~ B(3, 0.6)
= 0.5793
P(X = 0) = 3C 0(0.6)0(0.4)3 = 0.064,
P(X = 1) = 3C1(0.6)1(0.4)2 = 0.288, 16. (a) P(Z . k) = 0.5 – 0.3238
P(X = 2) = 3C2(0.6)2(0.4)1 = 0.432, = 0.1762
P(X = 3) = 3C3(0.6)3(0.4)0 = 0.216 k = 0.93
(c) (b) X – 80 = 0.93
3
P (X = r)
X = 82.79
0.432 Kertas 2
1. (a) (i) np = 15 …1
0.288
! npq = 3 ! 5 …2
0.216 2
Daripada 2:
! npq = 3 ! 5
2
0.064
r
! npq = ! 94 × 5
0 0 1 2 3

X–m
! npq = 45! 4
13. P(Z) = P
s( ) npq = 45 …3
4
(
= P 35 – 50
10 ) Gantikan 1 ke dalam 3:
= –1.5 (15)q = 45
X–m 4
(
14. (a) P(Z) = P
s ) q= 3
4
( =P 65 – 60
15 ) Nilai p,
p=1–q
= 1
X–m
3 =1– 3
(b) =Z 4
s
= 1
X – 60 = – 1 4
15 2

39

Jw01 Full SpotlightA+ MateTambahan Tg5.indd 39 26/10/2022 3:40 PM


Matematik Tambahan Jawapan

Gantikan p = 1 ke dalam 1 3. (a) (i) X ~ B(10, 0.15)


4 P(X = 0) = 10C 0(0.15)0(0.85)10
()
n 1 = 15

= 0.1969
4 (ii) X ~ B(20, 0.15),
n = 60
P(X . 3)
Maka, p = 1 , n = 60. = 1 – P(X = 0) – P(X = 1) − P(X = 2)
4
− P(X = 3)
(ii) X ~ B 10, (1
4 ) = 1 – 0.03876 – 0.1368 – 0.22934
= 0.3523
P(X = 3) = 10C3 1 3
3

4 4
7
( )( ) (b) (i) np > 5
= 0.2503 n> 5
0.15
(b) (i) Berdasarkan sifir taburan normal, n > 33.3333
f(z) Bilangan minimum panggilan ialah 34.
(ii) P(X > 1) . 0.95
1 − P(X = 0) . 0.95
P(X = 0) , 0.05
0.1515 n
C 00(0.15)0(0.85)n , 0.05
(0.85)n , 0.05
z n log10 0.85 , log10 0.05
0 k
log10 0.05
0.5 – 0.3485 = 0.1515 n.
log10 0.85
n . 18.4331
z 0 3 Tolak
Bilangan minimum n ialah 19.
4. (a) W ~ N(232, 5)
1.0 0.1515 2
(
P(W , 224) = P Z , 224 – 232
5 )
Maka, k = 1.03. = P(Z , –1.6)
(ii) X ~ N(79, 3) Berdasarkan sifir taburan normal,
X – 79 = 1.03
3 f(z)
X = 82.09
2. (a) (i) np = 315 …1
npq = 126 … 2
Gantikan 1 dan q = 1 – p,
315(1 − p) = 126
z
315 − 315p = 126 –1.6 0
−315p = −189
p = 0.6 P(Z , –1.6) = P(Z . 1.6)
(ii) X ~ B(8, 0.6), = 0.0548
P(X . 5) (b) P(W . w) = 0.30
= P(X = 6) + P(X = 7) + P(X = 8) P(W . 0.524) = 0.30
= 0.20902 + 0.08958 + 0.01680 w – 232 = 0.30
= 0.3154 5
w = 234.62
(b) X ~ N(5, ! 0.8 ) w ≈ 235
(
P(X . 6) = P Z . 6 – 5
! 0.8
) (c)

= 1.118
0.2

Berdasarkan sifir taburan normal,


0.2 0.8
f(z)

0.8 0.2

0.8

P(hanya satu) = 0.2 × 0.8 + 0.8 × 0.2


z
0 1.118 = 0.32 = 8
25
P(X . 1.118) = 0.1314

40

Jw01 Full SpotlightA+ MateTambahan Tg5.indd 40 26/10/2022 3:40 PM


Jawapan Matematik Tambahan

5. (a) m – 45 = – 3 Berdasarkan sifir taburan normal,


8 4
m – 45 = –6 f(z)
m = 39
(b) P(39 , X , 55)

(
= P – 3 , z , 55 – 45
4 8 )
3
=P– ,z,
4 ( 5
4 ) z
0 1
Daripada sifir taburan normal,
f(z) P(Z . 1) = 0.1587
(ii) P(210 , X , 225)

(
= P 210 – 220 , Z , 225 – 220
! 100 ! 100
)
= P(–1 , Z , 1)
z Berdasarkan sifir taburan normal,
–0.75 0 1.25
f(z)

(
=Pz.–3 –Pz. 5
4 ) ( ) 4
[ ( )] ( )
= 1–Pz. 3 –Pz. 5
4 4
= [1 – 0.2266] – 0.1056
= 0.0668
z
(
(c) P(X , 33) = P z , 33 – 45
8 ) –1 0 0.5

= P(z , –1.5)
Daripada sifir taburan normal,
P(−1 , Z , 0.5)
= P(Z . −1) − P(Z . 0.5)
f(z) = [1 − P(Z . 1)] − P(Z . 0.5)
= (1 − 0.1587) − 0.3085
= 0.5328
(c) Untuk 90%, iaitu kebarangkalian ialah 0.9
unuk pir yang mempunyai berat lebih daripada
h g,
z P(X . h) = 0.9
–1.5 0 P(X , h) = 1 – 0.9
= 0.1
P(z , –1.5) = P(z . 1.5) Berdasarkan sifir taburan normal,
= 0.06681
f(z)
Bilangan pelajar = 0.0681 × 180
= 12.0258
≈ 12 Luas = 0.10 Luas = 0.90
6. (a) (i) X ~ B(12, 0.05)
P(X > 2)
= 1 − P(X < 1) z
= 1 − P(X =0) − P(X = 1) –0.4602 0
= 1 − 0.54036 − 0.34128
P(Z . 0.4602) = 0.1
= 0.1184
P(Z , –0.4602) = 0.1
(ii) P(X > 1) . 0.85
1 – P(X = 0) . 0.85 h – 220 = −0.4602
10
n . 36.98 h = 215.4
n = 37
7. (a) n = 12, p = 0.72, q = 0.28
(b) (i) P(X . 230) = P Z . 230 – 220
! 100
( ) Katakan X mewakili bilangan murid lulus
= P(Z . 1) ujian Matematik
(i) P(X = 10) = 12C10(0.72)10(0.28)2
= 0.1937

41

Jw01 Full SpotlightA+ MateTambahan Tg5.indd 41 26/10/2022 3:40 PM


Matematik Tambahan Jawapan

(ii) P(X < 10) = 1 − P(X = 11) − P(X = 12) 2. (a) –285°
= 1 − 12C11(0.72)11(0.28)1 − 12C12(0.72)12(0.28)0 y
= 1 − 0.0906 − 0.0194
= 0.89 –285°
(b) Katakan X mewakili panjang paku yang x
dihasilkan O

(i) P(X . 3.85) = 1 − P Z , 3.85 – 3.6


0.12( )
= 1− P(Z . 2.083) (b) 880° – 720° = 160°
= 0.0189 Maka, 880° = 2(360° + 160°)
(ii) P(X , k) = 0.882 y
P(X > k) = 1 − 0.882

(
P Z > k – 3.6 = 0.118
0.12 ) x
k – 3.6 = 1.185 O
–880°
0.12
k = 3.74
(c) –50° – (–360°) = –145°
8. (a) Amri meneka secara rawak untuk 6 soalan,
Maka, –505° = –(360°) + (–145°)
maka n =6.
y
p = 1, q = 3
4 4
Kebarangkalian mendapat bilangan tekaan –505°
yang betul x
O

( ) ( 34 ) = 4729
P(X = 0) = 6C 0 1
4
0

029
6

P(X = 1) = C ( 1 ) ( 3 ) = 1 458 (d) –1.25 rad


1 5
6
4 4 1
4 029 y
P(X = 2) = C ( 1 ) ( 3 ) = 1 215
2 4
6
4 4 2
4 029
x
P(X = 3) = C ( 1 ) ( 3 ) = 540
3 3
6
O
4 4 3
4 029 –1.25 rad
P(X = 4) = C ( 1 ) ( 3 ) = 135
4 2
6
4 4 4
4 029
(e) 15 π = 675°
P(X = 5) = C ( 1 ) ( 3 ) = 18 4
5 1
6
= 360° + 315°
4 4 5
4 029
y
P(X = 6) = C ( ) ( ) = 1
1 3 6 0
6
4 4 6
4 029 15
–– π
4
(b) Amri menjawab 15 soalan dengan betul kerana
x
kebarangkalian untuk meneka 1 soalan dengan O
betul adalah paling tinggi.
(Terima sebarang jawapan yang sesuai)

Bab 6 Fungsi Trigonometri (f) – 11 π rad = –330°


6
y
Zon Formatif 6.1

1. x
O
Sudut dalam Sudut dalam – 11
–– π
Sukuan 6
unit darjah unit radian
(a) 373° 2.072π/ 6.51 I 3. (a) 9 π = 405°
4
(b) 83.08° 1.45 I = 360° + 45°
(c) 486° 2.7π II y
(d) 703° 3.906π/12.27 IV 9
–π
(e) –902° 5.011π/15.74 III 4
x
O
(f) –820.08° –4 556π III

42

Jw01 Full SpotlightA+ MateTambahan Tg5.indd 42 26/10/2022 3:40 PM


Jawapan Matematik Tambahan

(b) 11 π = 490° (ii) sek 227° = 1


4 sin 227°
= 360° + 135°
= –1.466
y (b) (i) –0.9688
( )
(ii) kot – 7 π = 1

O
x
8
( )
tan – 7 π
8
11 = 2.414
–– π
4 3. (a) 1
2
(c) – 1 π = –45°
4 (b) kosek 5π = 1
y 6
sin 5π
6
=1÷ 1
x 2
O
–1
–π =2
4
( )
(c) sek – π = 1

(d) – 3 π = –135°
3
kos – π ( )
3
4 1
y =
kos π ( )
3
=2
O
x
( )
(d) kot 5 π = 1
–3
–π
4
6
tan 5π ( )
6
= 1
4.
y
( ) (
1 × – 2
2 !3
)
= –! 3
4. (a) kos 103° = –kos (180° – 103°)
11 9 = –kos 77°
–– π –π
4 4 = –0.2250
x 1
O (b) kot 283° =
3π tan 283°
–– –1
–π
4 4 = 1
–tan (360° – 283°)
= 1
–tan 77°

Zon Formatif 6.2 = 1


–4.331
= –0.2309
1. (a) (i) sek 63° = 1
(c) kosek 257° = 1
kos 63°
sin 257°
= 1
= 1
sin 27°
–sin (257° – 180°)
= 1 1
0.454 =
–sin 77°
= 2.203
1 = –4.444
(ii) kosek 50.5° = 1
sin 50.5° (d) sek (–77°) =
1 kos (–77°)
= 1
sin 39.5° =
kos 77°
= 1 = 4.444
0.772
= 1.295 sin 7 π
(b) kot q = 5.185 5. (a) kos 7 π = 18
Diketahui tan 79° 5ʹ = kot (90° – 79° 5ʹ) 18 tan 7 π
= 5.185 18
= 0.94
Maka, q = (90° – 79° 5ʹ) = 10° 55ʹ
2.747
2. (a) (i) –0.3640 = 0.342

43

Jw01 Full SpotlightA+ MateTambahan Tg5.indd 43 26/10/2022 3:40 PM


Matematik Tambahan Jawapan

(
(b) kosek – 7 π = ) 1 (c) Diketahui kosek 30° = 2
18 sin – 7 π
18 ( ) Jika kosek adalah positif, maka x dalam
sukuan I dan sukuan II
= 1 x = 30° dan 180° – 30°
–sin 7 π
18 ( ) = 30° dan 150°
(d) Diketahui sekan 45° = ! 2
=– 1
0.94 Jika sekan adalah negatif, maka x dalam
= –1.064 sukuan II dan sukuan III
1 x = 180° – 45° dan 180° + 45°
(c) sek 29 π =
= 135° dan 225°
18 kos 29 π
18 8. q dalam sukuan IV
= 1
y
(
kos 2π – 29 π
18 )
= 1
θ 4
kos 7 π O
x
18
–3
= 1 5
0.342
= 2.924
(d) kot 25 π = 1
(a) sek q = 5
18 tan 25 π 4
18
1 (b) kosek q = – 5
= 3
tan 25
18 (
π–π ) 9. (a) sek (–225°) = –sek (225° – 180°)
1 = –sek 45°
=
tan 7 π = –! 2
18
(b) kot (600°) = kot (600° – 540°)
= 1 = kot 60°
2.747
= 0.364 = 1
!3
6. ! 1 – (1 – k)2 = ! 2k – k2
(c) kosek 15 π = kosek (4π – 15 π)
y 4 4
= –kosek π1
4
θ (1 – k)
x = –! 2
O
–2k – k 2
5
Zon Formatif 6.3

1. y = 2 kos 3x + 1 untuk 0 < x < 2π.


(a) kosek q = 1
sin q y
1 y = 2 kos 3x + 1
=–
3
! 2k – k2
(1 – k) 2
(b) kot q = –
! 2k – k2 1
!3 x
7. (a) Diketahui sin 60° = 0 π π 3π π 5π 3π 7π 2π
2 –
4

2
––
4
––
4
––
2
––
4
–1
Jika sinus adalah negatif, maka x dalam
sukuan III dan sukuan IV
x = 180° + 60° dan 360° – 60° 2. (a) • Bentuk: Graf sinus
= 240° dan 60° –1 – (–5)
• Amplitud: a = =2
(b) Diketahui tan 30° = 1 2
!3 • Bilangan kitaran dalam 0 < x < 2π; b = 1
Jika tangen adalah positif, maka x dalam
sukuan I dan sukuan III
• Translasi:
0
–3 ( )
, c = –3
x = 30° dan 180° + 30° º y = 2 sin x – 3
= 30° dan 210°

44

Jw01 Full SpotlightA+ MateTambahan Tg5.indd 44 26/10/2022 3:40 PM


Jawapan Matematik Tambahan

(b) • Bentuk: Graf tangen (e) (i) Pantulkan bahagian negatif graf pada
• Amplitud: a = 1 paksi-x.
• Bilangan kitaran graf dalam 0 < x < π y
hanya 1 , sepatutnya ialah DUA; b = 1 y = |2 sin 3x|
2 2 2
• Translasi:
0
1 ()
,c=1 1
x
º y = tan 1 x + 1 0 π

π

π


––

–– π 7π
––

––

––
2 6 3 2 3 6 6 3 2
(c) • Bentuk: Graf kosinus
(ii) Selepas itu, pantulan semua bahagian graf
• Amplitud: a = 3 – 1 = 1 pada paksi-x.
2
• Bilangan kitaran graf dalam 0 < x < 2π: y

b= 1 2
2
• Translasi:
0
2 ()
,c=2
1

0 π π π 2π 5π 7π 4π 3π
x
π
º y = kos 1 x + 2 –1

6

3

2
––
3
––
6
––
6
––
3
––
2
2
3. (a) y = 2 sin 3x –2
y = –│2 sin 3x│
• Kala: 2π = 2π
b 3
• Bilangan kitaran dalam 0 < x < 2π ialah 3. 4. (a) y = 1 sin x + 2 untuk 0 < x < 2π
2
Bilangan kitaran dalam 0 < x < 3 π: y
2

=

2 ( ) 3

3 ( ) 2
1
y = – sin + 2x
1
= 2π × 3 2
3 2π x
9 0 π

π


–– π 5π
––

––

–– 2π
= 4 2 4 4 2 4
4
(b) y = –tan 2x untuk 0 < x < π
= 2 1 kitaran
4 y
(b) Amplitud = 2
y = – tan 2x
(c) Titik maksimum: 3

( )(
6 6 )(
π , 2 , 5π , 2 , 2π , 2
6 ) 2
Titik minimum:
1
( )(
π , –2 , 7π , –2
2 6 ) x
(d) • Bentuk: graf sinus (Bermula di (0, 0)) 0 π

π


–– π
–1 4 2 4
• Amplitud = 2
• Bilangan kelas = 3 × 3 × 2 –2
2
=9 –3

• Saiz selang kelas = 2
9 | 2 2 |
(c) y = – 3 kos 1 x untuk 0 < x < 3 π
π
= y
6
y
3 y =│3 kos 1
– x│
2 2
2
1
1
x
0 π

π

π


––

–– π 7π
––

––

––
–1 6 3 2 3 6 6 3 2 x
0 π

π


–– π 5π
––

––
–2 4 2 4 4 2
y = 2 sin 3x

45

Jw01 Full SpotlightA+ MateTambahan Tg5.indd 45 26/10/2022 3:40 PM


Matematik Tambahan Jawapan

(d) y = |tan x| – 1 untuk 0 < x < 2π 6. (a)


y π π 2π 5π
x 0
6 2 3 6
4
y =│tan x│– 1 y 0 1.73 ∞ –5.2 0
3
2 7π 4π 3π
x π
6 3 2
1
y 0 1.73 5.2 ∞
x
0 π

π


–– π 5π
––

––

–– 2π
–1 4 2 4 4 2 4
y

(e) y = –|2 kos 2x| untuk –π < x < π 5 y = 3 tan x


y 4
3
1
2
x
–π 3π π π
– –– – – – – 0 π

π


–– π 1
4 2 4 4 2 4
–1
x
–2 0 1
–π
1
–π
1
–π
2
–π
5
–π π 7
–π
4
–π
3
–π
y = –│2 kos 2x│ –1 6 3 2 3 6 6 3 2

–2
5. y = kos 2x
–3
π π 3π 5π 3π
x 0 π –4
4 2 4 4 2
y 1 0 –1 0 1 0 –1 –5

y = 3 – 3x (b) x – 3 tan x = 5 π
4 2π 6
x 0
π 3 tan x = x – 5 π
2 6
3 ºy=x– 5π
y 0 6
4
(c) Lukis graf y = x – 5 π di (a).
6
Pada paksi yang sama, lukis graf y = kos 2x

dan y = 3 – 3x untuk 0 , x , 3 π. x 0
6
4 2π 2
y y –2.62 0
y = kos 2x y
1

5 y = 3 tan x
x 4
0 π
– π 3π
––
2 2 3 5
y=x– –π
2 6
–1 3 3x
y = – – –– 1
4 2π
x
Seterusnya, tentukan bilangan penyelesaian bagi 0 1
–π
1
–π
1
–π
2
–π
5
–π π 7
–π
4
–π
3
–π
persamaan trigonometri: –1 6 3 2 3 6 6 3 2

6x + 4 kos 2x = 3 –2
π
–3
4 kos 2x = 3 – 6x
π –4
kos 2x = 3 – 6x –5
4 4π
kos 2x = 3 – 3x Penyelesaian: x = 3.30 rad
4 2π
Persilangan graf y = kos 2x dan y = 3 – 3x adalah
4 2π
2.
Maka, bilangan penyelesaian ialah 2.

46

Jw01 Full SpotlightA+ MateTambahan Tg5.indd 46 26/10/2022 3:40 PM


Jawapan Matematik Tambahan

7. (a) Untuk mencari bilangan penyelesaian


y
kos 3 x = 3 x – 1
2 4π
2
2 kos x = 2 3 x – 2(1)
3
2 4π ( ) 1
y = tan 2q

2 kos 3 x = 3 x – 2 q
2 2π
0 π

π


–– π
Lakarkan graf y = 2 kos 3 x dan y = 3 x – 2 –1 4 2 4
2 2π
untuk 0 < x < 2π. –2 y=2–q

3
y= 3 x–2

Bilangan penyelesaian = 2
x 0 2π
y –2 0 Zon Formatif 6.4
y
1. y = sin q
3 3 x = kos q
2 y = 2 kos – x y = –– x – 2
2 2π x 2 + y2 = 1
1 sin q + kos2 q = 1 …1
2

x
0 π
– π 3π
–– 2π Bahagikan setiap sebutan dalam 1 dengan kos2 q
–1 2 2
sin2 q + kos2 q = 1
–2 kos2 q kos2 q kos2 q
tan2 q + 1 = sek 2 q …2
Bilangan penyelesaian = 3 Bahagikan setiap sebutan dalam 1 dengan sin2 q.
(b) Lakarkan graf y = |sin a| dan y = kos 2a sin2 q + kos2 q = 1
dalam julat 0 < a < 2π. sin2 q sin2 q sin2 q
y 1 + kot 2 q = kosek 2 q …3
y =│sin a│ 2. (a) 5 sin2 337° + 5 kos2 337°
1 = 5 (sin2 337° + kos2 337°)
= 5(1)
a =5
0 π

π


–– π 5π
––

––

–– 2π (b) 1 – sek 2 193° + tan2 193°
4 2 4 4 2 4
= 1 – (1 + tan2 193°) + tan2 193°
–1 =0
y = kos 2a
(c) kosek 2 3π – kot 2 3π + sin2 2π
5 5
Bilangan penyelesaian |sin a| – kos 2a = 0
dalam julat tersebut ialah titik persilangan graf = 1 + kot 2 3π – kot 2 3π + sin2 2π
5 5
y = |sin a| dan y = kos 2a. =1+0
Bilangan penyelesaian = 4 =1
(c) Untuk mencari bilangan penyelesaian bagi (d) 2 + sin2 2.9 rad + kos2 2.9 rad
3 tan 2q + q = 6: =2+1
3 tan 2q + q = 6 =3
tan 2q = 6 – q 3. Diberi tan A = 1 – m.
3 1
(a) kot 2 A =
tan 2q = 2 – q tan2 A
3 1
1 Lakar y = tan 2q =
(1 – m)2
2 Lakarkan y = 2 – q (b) sek 2 q = 1 + tan2 A
3
= 1 + (1 – m)2
x 0 π = 1 + 1 – 2m + m2
y 2 0.95 = 2 – 2m + m2
(c) kos2 q = 12
sek q
= 1
2 – 2m + m2

47

Jw01 Full SpotlightA+ MateTambahan Tg5.indd 47 26/10/2022 3:40 PM


Matematik Tambahan Jawapan

4. (a) Sebelah kiri = 3 + 2 tan2 q (b) sin 0.75π kos 0.25π – kos 0.75π sin 0.25π
= 3 + 2(sek 2 q – 1) = sin (0.75π – 0.25π)
= 3 + 2 sek 2 q – 2 = sin 0.5π
= 1 + 2 sek 2 q =1
(b) Sebelah kiri = kos x kot x (c) kos 30.5° kos 29.5° – sin 30.5° sin 29.5°
= kos x kos x
sin x ( ) = kos (30.5° + 29.5°)
= kos 60°
= kos 2
x = 1
sin x 2
= – sin x
1 (d) kos 2 π kos π + sin 2 π sin π
2

sin x 3 2 3 2
= kosek 2 x – sin x
(c) Sebelah kiri = kot 2 q – kos2 q (
= kos 2 π – π
3 2 )
π
= kos2 q – kos2 q
2
= kos
sin q 6
!3
= kos q – kos q sin2 q
2 2
=
sin q
2 2
kos2 q(1 – sin2 q) (e) tan 0.8π + tan 0.2π = tan (0.8π + 0.2π)
=
sin2 q 1 – tan 0.8π tan 0.2π
= kot q – kos2 q
2 = tan π
tan a =0
(d) Sebelah kiri =
1 + tan2 a (f) 1 + tan 228° tan 183° = kot (228° – 183°)
sin a tan 228° – tan 183°
= = kot (228° – 183°)
kos a sek 2 a
sin a kos2 a = kot 45°
= =1
kos a
= kot a sin a 2. (a) kosek 195°
(e) Sebelah kiri = 1
= (sek x + tan x)(kosek x – 1) sin (150° + 45°)
= sek x kosek x – sek x + tan x kosek x – tan x = 1
sin 150° kos 45° + kos 150° sin 45°
= 1 – 1 + sin x – sin x
kos x sin x kos x kos x sin x kos x = 1
1 !2 + – !3 !2
= 1 – sin x
kos x sin x kos x ( )( ) (
2 2 2 2 )( )
= 4
= 1 – sin x
2

kos x sin x !2 – !6
(b) sek 165° = 1
= kos2 x
kos 165°
kos x sin x
= kot x = 1
1 – 1 kos (120° + 45°)
(e) Sebelah kiri = 1
tan q kot q =
q sin 120° kos 45° – sin 120° sin 45°
= kos – sin q 1
sin q kos q =
q q
kos – sin
(– 12 )( !22 ) + ( !23 )( !22 )
2 2
=
sin q kos q
= 1 × sin q = 1
sin q kos q kos q
= sin q
sin2 q kos q
( !42 ) – ( !46 )
= 4
= kosek 2 q tan q
!2 – !6

Zon Formatif 6.5 (c) kot 11 π = kot 165°


12
= 1
1. (a) sin 24.7° kos 20.3° + kos 24.7° sin 20.3° tan 165°
= sin (24.7° + 20.3°) 1
= sin 45° =
tan (120° + 45°)
!2 = 1 – tan 120° tan 45°
= tan 120° + tan 45°
2

48

Jw01 Full SpotlightA+ MateTambahan Tg5.indd 48 26/10/2022 3:40 PM


Jawapan Matematik Tambahan

1 – (–! 3 )(1) (b) Sebelah kiri


=
–! 3 + 1 = kos B – kos A kos (A – B)
= kos B – kos A (kos A kos B + sin A sin B)
1 + !3 = kos B – kos2 A kos B – kos A sin A sin B
=
1 – !3 = kos B(1 – kos2 A) – kos A sin A sin B
3. Diberi bahawa sin A = 5 dan kos B = – 3 . = kos B(sin2 A) – kos A sin A sin B
13 5 = sin A(sin A kos B – kos A sin B)
y
y = sin A(sin kos B –)
= sin A sin (A – B)
= Sebelah kanan
4 5
13 A B (c) Sebelah kiri
5
x x sin (P – Q)
–12 O –3 O =
kos P kos Q
3. (a) kos (A + B) = kos A kos B – sin A sin B sin P kos Q – kos P sin Q
=
kos P kos Q

13 5 ( )( ) ( )( )
= – 5 – 3 – – 12 4
13 5
= (tan P – tan Q)
= Sebelah kanan
=– +16 48
65 65 6. (a) Diberi tan q = t dan q adalah tirus.
= – 33 y
65
Maka, kosek (A – B) = – 65
33 t2
tan B – tan A +
(b) tan (B – A) = �1 t
1 + tan B tan A θ
x
O 1

=
–3 – – 5
5 12 ( ) ( )
1+ – 4
3
– 5
12 ( )( ) sin q =
t
, kos q =
1
– 11 !1 + t 2
! 1 + t2
= 12
14 (i) kos 2q = kos2 q – sin2 q
9
= – 11 × 9
= 1
1 + t2 ( – t2
1 + t2 ) ( )
12 14
= 1 – t 2
2

= – 33 1+t
56
4. Diberi m = tan 14° dan n = tan 40°. (ii) kosek 2q = 1
sin 2q
(a) tan 54° = tan (14° + 40°) 1
=
= tan 14° + tan 40° 2 sin q kos q
1 – tan 14° tan 40° 1
=
= m + n 2 t
(
– 1
) ( )
1 – mn ! 1 + t2 ! 1 + t2
(b) kot 26° = 1
= 1+t
2
tan 26°
2t
= 1
tan (40° – 14°) (iii) kos q = 2 kos2 q – 1
2
= 1 + tan 40° tan 14° 2 kos 2 q
= kos q + 1
tan 40° – tan 14° 2
= + nm 1
kos2 q = 1 (kos q + 1)
n–m 2 2
5. (a) Sebelah kiri
=
kos (a – b) – kos (a + b) = 1 1
2 ! 1 + t2 (
+1 )
sin (a + b) + sin (a + b)
(kos a kos b – sin a sin b) 1 + ! 1 + t2
= =
sin a kos b + kos a sin b 2! 1 + t2
2 sin a sin b
=
2 sin a kos b
= tan b
= Sebelah kanan

49

Jw01 Full SpotlightA+ MateTambahan Tg5.indd 49 26/10/2022 3:40 PM


Matematik Tambahan Jawapan

(b) Jika sudut b adalah cakah dan kos b = –! 1 – p2,


p . 0,
=– 8!
10
y
=– 4!
5
=– 2
1
p β !5
x !5
�1 – p 2 O Maka, sek A = – .
2 2
(c) kot 4A = 1
sin 2b = 2 sin b kos b tan 4A tan 2A = 2 tan A
1 – tan2 A
= 1 – tan 2A
2
= 2p(–! 1 – p2)
1
2 tan 2A 2–4
3 ( )
(i) sek 2b =
kos 2b
=
1 – 24( )
7
2 =
1 – 16
9


kos 2b = kos2 b – sin2 b
= (1 – p2) – p2
( )
2 24
7
=– ×–9
8
3 7
= 1 – 2p2 =– 527 × 7 = 24
49 48 7
Maka, sek 2b = 1
1 – 2p2 =– 527
(ii) sin 3b = sin (2b + b) 336
= sin 2b kos b + kos 2b sin b (d) kosek 3A = 1
sin 3A
= –2p! 1 – p2(–! 1 – p2) + (1 – 2p2)p
= 1
= 2p(1 – p2) + p(1 – 2p2) sin (2A + A)
= 2p – 2p3 + p – 2p3 1
=
= 3p – 4p3 sin 2A kos A + kos 2A sin A
b = 1
= kos b
(iii) 1 – 2 sin2
2 ( )( ) ( )( )
– 24 3 + – 7 – 4
25 5 25 5
b 1
2 sin2 = 1 – kos b =
2
b 1 – (–! 1 – p2) (
– 72 + 28
125 ) ( )
125
sin2 =
2 2 =– 125
1 + ! 1 – p2 44

2
= 8. (a) Sebelah kanan = 1 (tan q + kot q)
2
7. Diberi bahawa kos A = 3 dan π < A < 2π.
5 = 1
(
sin q + kos q
2 kos q sin q )
y 1
(
sin q + kos q
)
2 2
=
2 kos q sin q
= 1
A 3 2 sin q kos q
x
O
= kosek 2q
–4
5 (b) Sebelah kanan = (1 – kos 2A) kot A
= (1 – (1 – 2 sin2 A) kos A
sin A
(a) sin 2A = 2 sin A kos A = 2 sin 2
A kos A
=2– 4 3 ( )( )
5 5 = sin 2A
sin A

= – 24 = Sebelah kiri
25
A
(b) sek = 1 (c) Sebelah kanan = 1 – kos 2x + sin x
sin 2x + kos x
2 kos A 1 – (1 – 2 sin2 x) + sin x
2 =
Gunakan rumus sudut separuh 2 sin x kos x + kos x
2 sin2 x + sin x
2 !
kos A = 1 + kos A , A sudut cakah.
2 2
=
kos x (2 sin x + 1)
sin x(2 sin x + 1)
( )
! 1+ 3 =
5 kos x(2 sin x + 1)
=
2 = tan x

50

Jw01 Full SpotlightA+ MateTambahan Tg5.indd 50 26/10/2022 3:40 PM


Jawapan Matematik Tambahan

tan 2y (b) sek 2A = 1


(d) Sebelah kiri = kos 2A
1 + sek 2y
tan 2y = 1
= kos2 A – sin2 A
1+ 1 1
kos 2y =
!5 2
=
tan 2y
kos 2y + 1
( ) ( )
3
– 2   
3
2

kos 2y = 1

=
tan 2y(kos 2y) 9( ) ( )
5 – 4
9
kos 2y + 1 =9
sin 2y
= (c) kos 4A = 2 kos2 2A – 1
(2 kos2 y – 1) + 1

=
2 sin y kos y
9( )
=2 1 –1
2

2 kos2 y
sin y = – 79
= 81
kos y (d) kot B = 1 + kos B
= tan y 2 sin B
4! 5
2 !
9. (a) kos q = ± 1 + kos q
2 =
(
1+ –
9 )
1
Bukti: 2 kos2 q – 1 = kos q 9
2
2 kos2 q = kos q + 1 =
9 – 4! 5
×9
2 9
kos2 q = kos q + 1 = 9 – 4! 5
2 2


2 !
kos q = ± kos q + 1
2 Zon Formatif 6.6

(b) tan q = 1 – kos q 1. (a) sin x = kos 72° = sin 18°


2 sin q (* Rumus sudut pelengkap)

Bukti: 1 – kos q =
(
1 – 1 – 2 sin2 q
2 ) Sudut rujukan, a = 18°
sin x positif, x dalam sukuan I dan II
sin q
2 sin q kos q Maka, x = 18° dan 180° – 18
2 2
x = 18° dan 162°
2 sin2 q (b) kos 2x = –0.4673
= 2
Sudut rujukan, a = kos–1 0.4673
2 sin q kos q = 62.14°
2 2
kos 2x negatif, 2x dalam sukuan II dan III
= tan q
2 Bagi 0 < 2x < 720°
10. Diberi sin A = 2 dengan A ialah sudut tirus dan 2x = 180° – 62.14° = 117.86°
3 = 180° + 62.14° = 242.14°
sin B = 1 dengan B ialah sudut cakah. = 360° + 117.86° = 477.86°
9
= 360° + 242.14° = 602.14°
y y x = 58.93°, 121.07°, 238.93°, 301.17°
(c) tan (x + 30°) = –0.7402
Sudut rujukan = tan–1 0.7402
3 9 = 36.51°
2 1
tan (x + 30°) negatif,
x x
O �5 –4�5 O (x + 30°) dalam sukuan II dan IV.
Sukuan II: x + 30° = 180° – 36.51° = 143.49°
x = 113.49°
(a) sin (A + B) = sin A kos B + kos A sin B Sukuan IV: x + 30° = 360° – 36.51° = 323.49°
4! 5 !5 x = 293.49°
( )(
= 2 –
3 9
– ) ( )( 19 )
3 2. (a) 4 sin x = sek x

=–
8! 5
+
!5 4 sin x = 1
27 27 kos x
7! 5 4 sin x kos x = 1
=–
27

51

Jw01 Full SpotlightA+ MateTambahan Tg5.indd 51 26/10/2022 3:40 PM


Matematik Tambahan Jawapan

2 sin x kos x = 1 Sudut rujukan = 71.57°


2 2x + 10° = 71.57°, 251.57°, 431.57°, 611.57°
sin 2x = 1 2x = 61.57°, 241.57°, 421.57°, 601.57°
2 x = 30.79°, 120.79°, 210.79°, 300.79°
Sudut rujukan = sin–1 0.5 (f) 3 sin z – 1 = kos z
= 30° 2
2x = 30°, 150°, 390°, 510° 3 sin z = kos z + 1
2
x = 15°, 75°, 195°, 255°
x = 1 π, 5 π, 13 π, 17 π

2 (
3 sin z = 1 – 2 sin2 z + 1
2 )
12 12 12 12 2 sin2 z + 3 sin z – 2 = 0
(b) sin A + 3 kos 2A = 2 2 2
sin A + 3(1 – 2 sin2 A) = 2 sin z = 0.5, –2 (Ditolak)
2
sin A + 3 – 6 sin2 A – 2 = 0 z = 30°, 150°

–6 sin2 A + sin A + 1 = 0 2
sin A = – 1 , 1 z = 60°, 300°
3 2
Jika sin A = – . 1 z = 1 π, 5 π
3 3 3
Sudut rujukan = 19.47°
A = 180° + 19.47°, 360° – 19.47° ( )
(g) sin f – π = 2 kos π – f
6 3 ( )
= 199.47°, 340.53°
sin f kos π – kos f sin π
Jika sin A = 1 6 6
2
Sudut rujukan = 30° (
= 2 kos π kos f + sin π sin f
3 3 )
A = 30°, 150°
!3
Maka, A = 30°, 150°, 199,47°, 340.53° sin f – 1 kos f = 2
2 2
A = π , 5 π, 1.108π, 1.8918π
6 6 !3 !3
(
(c) sin x = kos x + 1 π ) 2
sin f – 2
2 ( ) 2 2( )
sin f = 2 1 kos f + 1 kos f
6
!3
sin x = kos x kos 1 π – sin x sin 1 π – sin f = 3 kos f
6 6 2 2
!3
sin x = kos x – 1 sin x tan f = – 3 × 2
2 2 2 !3
2 sin x = ! 3 kos x – sin x tan f = – ! 3
3 sin x = ! 3 kos x Sudut rujukan = 60°
!3 f = 60°, 240°
tan x =
3
= 1 π, 4 π
Sudut rujukan = 30° 3 3
(h) 5 kos x sin x – 2 = 0
x = 30°, 210° atau 1 π, 7 π
6 6 5 kos x sin x = 2
(d) kot 2 y + 4 = 4 kosek y 2 kos x sin x = 2 × 2
5
kosek 2 y – 1 + 4 = 4 kosek y sin 2x = 4
kosek 2 y – 4 kosek y + 3 = 0 5
(kosek y – 3)(kosek y – 1) = 0 Sudut rujukan = 53.15°
kosek y = 3, 1 2x = 53.15°, 126.84°, 413.13°, 486.84°
x = 26.57°, 63.42°, 206.57°, 243.42°
Jika kosek y = 3, sin y = 1
3 kos (A + B)
Sudut rujukan = 19.47° 3. Diberi = 1.
kos (A – B) 4
y = 19.47°, 180° – 19.47°
(a) (i) 4(kos A kos B – sin A sin B)
y = 19.47°. 160.53°
= kos A kos B + sin A sin B
Jika kosek y = 1, sin y = 1
3 kos A kos B = 5 sin A sin B
y = 90°
(ii) Dari (a)(i): sin A sin B = 3
y = 19.47°, 90°, 160.53° kos A kos B 5
y = 0.108π, π , 0.8918π tan A tan B = 0.6
6 (b) tan (A + B) = tan 135°
(e) sin (2x + 10°) – 3 kos (2x + 10°) = 0 tan A + tan B = –1
sin (2x + 10°) = 3 kos (2x + 10°) 1 – tan A tan B
sin (2x + 10°) tan A + tan B = –(1 – 0.6)
=3
kos (2x + 10°) tan A + tan B = –0.4
tan (2x + 10°) = 3

52

Jw01 Full SpotlightA+ MateTambahan Tg5.indd 52 26/10/2022 3:40 PM


Jawapan Matematik Tambahan

4. v = 660 sin 135°


sin x =d ( !3
!3 – 1
)( ×
!3 + 1
!3 + 1
)
= 600 × 0.7071
7
! 50
=d ( 3 + !3
2 )
= (660 × 0.7071) ×
= 471.43 km j–1
! 50
7 h= 3 +
2 (
!3
2
d )
!3
5. (a) sin g = sin (a + b) Maka, m = 3 , n = .
2 2
Bukti: 7. (a) a = 1, b = 2, c = 0
sin g = sin (180° – (a + b)) y = sin 2x
= sin 180° kos (a + b) – kos 180° sin (a + b) (b) y = 0.5, x ≈ 0.4, 1.2, 3.5, 4.3, 6.7
= (0)(kos (a + b) – (–1)(sin (a + b) y = –0.3, x ≈ 1.7, 3.0, 4.9, 6.1
= sin (a + b) y
g a+b
(b) kos = sin
2 2 ( ) 1
y = sin x

Bukti: y = 0.5
a + b + g = 180°
x
g = 180° – (a + b) 0 2 4 6 8 10
y = –0.3
g (a + b)
= 90° –
2 2 –1
g (a + b)
kos = kos 90° –
2 ( 2 )
g (a + b)
kos = kos 90° kos
2 2
+ Zon Sumatif
(a + b) Kertas 1
sin 90° sin
2
g a+b 1. (a) kos 57° = sin (90° – 57°)
kos = sin
2 2 ( ) = sin 33°
tan a + tan g = m
(c) tan a = 1
tan b tan g – 1 (b) kosek 40° =
sin 40°
Bukti: tan a = tan (180° – (b + a))
= 1
tan 180° – tan (b + g) kos (90° – 40°)
=
1 – tan 180° tan (b + g) =n
tan a = – tan (b + g)
2. (a) kot 315° = –kot (360° – 315°)
tan b + tan g
Maka, tan a = –
1 – tan b tan g( )

= –kot 45°
= –1
tan b + tan g
tan a = (b) sek 510° = sek (510° – 360°)
tan b tan g – 1
= sek 150°
(d) Bukti: tan (a + b + g) = tan 180° = –sek (180° – 150°)
tan (a + b) + tan g = –sek 30°
=0
1 – tan (a + b) tan g !3
tan (a + b) + tan g = 0 =–
2
tan a + tan b) (c) kos 5 π = kos 225°
+ tan g = 0 4
1 – tan a tan b = –kos (25° – 180°)
tan a + tan b + tan g (1 – tan a tan b = 0 = –kos 45°
tan a + tan b + tan g – tan g tan a tan b = 0 !2
=–
Maka, tan a + tan b + tan g = tan g tan a tan b 2

(
6. h = d tan A × tan B )
4
3 ( )
(d) kosek – π = kosek (–240°)
tan A – tan B = –kosek (240° – 180°)
tan π × tan π = –kosek 60°
=d ( 3
tan π – tan π
4
) =– 2
!3
3 3

=d ( !3 × 1
!3 – 1
)
53

Jw01 Full SpotlightA+ MateTambahan Tg5.indd 53 26/10/2022 3:40 PM


Matematik Tambahan Jawapan

3. Diberi kos x =
!5
dan 90° < x < 360°. (b) tan (225° – q) = tan 225° – tan q
3 1 + tan 225° tan q
(a) sek (180° – x) = 1
kos (180° – x) =
1– 2
3 ( )
= 1
kos 180° kos x + sin 180° sin x
1+ 2
3 ( )
1 1
= × 3
=
–kos x 3 5
=– 3
!5
y
4 ( )
5. kos π – b = kos π kos b + sin π sin b
2 2
= 0 – sin b
�5
x =–1
O p
–2
3 6. Diberi sin a = 3 dan kos b = 12 dengan keadaan
5 13
sudut a dan sudut b adalah tirus.
tan x – tan π
(
(b) tan x – π = ) 4 2( ) 2 (
kos π – a – b = kos π – (a + b) )
4 1 + tan x tan π = tan (a + b)
4
tan a + tan b
tan x – tan π =
= 4 1 – tan a tan b
1 + tan x tan π
4
=
3 + 5
4 ( ) ( )
12
2

=
!5
–1 1– 3
4 12
5
( )( )
1+
2
!5
(1) ( ) = 7 × 16
6 11
= 56
= ( 2 – !5
!5
)(
!5 + 2
!5
) 33
7. y = a kos bx, kala = 2π

( )
2 – !5 2 – !5 2 – !5 b
= atau × π = 2π
2 + !5 2 + !5 2 – !5 (a)
3 b
=
(2 – !5)
2

b= 2π ×3
4–5 π
= –(2 – ! 5 ) = 6
2

(b) π = 2π
(c) sin x =
2 ! 1 – kos
2
q 6 b
2π ×6
b=
π

=– ! 1– ( !35 )
2
= 12
(c) π = 2π
12 b
=– ! 3 –6! 5 b= 2π × 12
π
= 24
4. Diberi tan q = 2 , carikan nilai
3 8. (a) f(x) = 4 sin x – 3
(a) tan 2q = 2 tan 2q Nilai maksimum = 1
1 – tan q
Nilai minimum = –7

=
2 2
3 ( ) f(x)
1– 2
3
2
( ) f(x) = 4 sin x – 3
x

=
4
3 ( ) 0 2 4 6
9–4 –2
9
–4
= 4 × 9
3 5 –6
= 12
5

54

Jw01 Full SpotlightA+ MateTambahan Tg5.indd 54 26/10/2022 3:40 PM


Jawapan Matematik Tambahan

(b) g(x) = 3 kos x + 2 12. y = |1 + 2 sin x| untuk 0 < x < 2π.


Nilai maksimum = 5
Nilai minimum = –1 y
y = |1 + 2 sin x |
g(x) 3
2
6 g(x) = 3 kos x + 2
1
4
x
2 0 π 3π π ––
π –– 5π –– 7π 2π
3π ––
–1 – –
4 2 4 4 2 4
x
0 1 2 3 4 5 6 –2
y = 2 sin x

(c) h(x) = 1 – 2 kos 2x


Nilai maksimum = 3 y = |1 + 2 sin x|
Nilai minimum = –1 y

h(x) 3 y =│1 + 2 sin x│


h(x) = 1 – 2 kos 2x 2
3
2 1

1 x
0 π

π


–– π 5π
––

––

–– 2π
4 2 4 4 2 4
x
0 1 2 3 4 5 6
–1 13. Untuk mencari bilangan penyelesaian
sin 2x – |2 kos 2x| = 0 untuk 0 < x < 2π.
Lakarkan: y = sin 2x dan y = |2 kos 2x|
9. Bentuk: Graf sinus
y
Amplitud = 2 – 0 = 1 y =│2 kos 2x│
2
2
Bilangan kitaran = 2

()
0 1
Translasi:
1 x
0 π

π


–– π 5π
––

––

–– 2π
Jawapan: y = sin 2x + 1 –1 4 2 4 4 2 4
y = sin 2x
10. (a) Bentuk: Graf kos (b) 1
2
Amplitud: a = 3 Bilangan penyelesaian = 4
sin 2x + sin x
Bilangan kitaran: b = 1 14. Sebelah kiri =
2 1 + kos x + kos 2x

|
y = 2 kos x1
| = 2 sin x kos x + sin x
2 1 + kos x + (2 kos2 x – 1)
11. (a) Bentuk graf: tangen negatif sin x (2 kos x + 1)
Bilangan kitaran: 2, b = 1 =
kos x(1 + 2 kos x)
Jawapan: f(x) = –tan x = tan x
(b) f(x) = π 15. (a) Sebelah kanan = sin2 q(kosek 2 q – 1)
x
x 0 π 2π = sin2 q
1
sin2 q
–1 ( )
f(x) ∞ 1 0.5 = 1 – sin2 q
(0, ∞) (π, 1) (2π, 0.5) = kos2 q
(b) Diberi bahawa sin q = ! p2 – 1.
y
y = – tan x kos q = ! 2 – p2
4
2 π y
y=–
x
x
0 π π ––
3π 2π 5π

–2 2 2 1
�p 2 – 1
–4
x
O �2 – p 2

Bilangan penyelesaian = 2

55

Jw01 Full SpotlightA+ MateTambahan Tg5.indd 55 26/10/2022 3:40 PM


Matematik Tambahan Jawapan

16. Sebelah kanan = sek 2 q sin2 q 23. 4 sin (y – π) kos (y – π) = 1


1 1
= (sin2 q) 2 sin (y – π) kos (y – π) =
kos2 q 2
= tan2 q 1
sin 2(y – π) =
2
17. Sebelah kanan = sin q π
1 + kos q Sudut rujukan =
6
1 1
2 sin q kos q sin 2(y – π) positif, 2(y – π) dalam sukuan I dan II.
= 2 2
1 Untuk 0 < 2(y – π) < 4π
1 + 2 kos2 q – 1 π π 13
2 2y – 2π = , 2y = 2π + , y = π
1 1 6 6 12
2 sin q kos q 5π 5π 17
= 2 2 2y – 2π = , 2y = 2π + ,y= π
1 6 6 12
2 kos2 q
2 π π 25
1 2y – 2π = 2π + , 2y = 4π + , y = π
= tan q 6 6 12
2 5π 5π 19
1 2y – 2π = 2π + , 2y = 4π + ,y= π
18. Sebelah kiri = tan x + 6 6 12
tan x
=
tan2 x + 1 24. Diberi sin2 2q – sin2 q = 0. Cari nilai q
tan x untuk 0 < q < π.
sek 2 x sin2 2q – sin2 q = 0
=
tan x (2 sin q kos q)2 – sin2 q = 0
=
1
kos2 x sin x (
kos x
) 4 sin2 q kos2 q – sin2 q = 0
sin2 q(4 kos2 q – 1) = 0
1 2 Jika sin2 q = 0, sin q = 0, q = 0 rad, π rad
= ×
kos x sin x 2 1
Jika 4 kos2 q – 1 = 0, kos2 q =
2 4
= 1
sin 2x kos q = ±
2
(
19. kos 40° +
x
2 )
= 0.4848 π
Sudut rujukan = , q = , π
π 2
3 3 3
Sudut rujukan = kos–1 0.4848 π 2
= 61° Jawapan: 0, , π, π
3 3
(
kos positif, 40° +
x
2 )
dalam sukuan I dan IV. 25. 2 sin a – 2 kos a =
1
kos a + sin a
x
40° + = 61°, 360° – 61° = 290° 2(sin a – kos a)(kos a + sin a) = 1
2
x
= 21°, 259° (tak mungkin) 2(sin2 a – kos2 a) = 1
2
x = 42° –kos 2a = 1
2
20. sin y – kos y = 0. kos 2a = – 1
sin y = kos y 2
π
tan y = 1 Sudut rujukan = atau 60°
π 3
Sudut rujukan = 2a = 120°, 240°, 480°, 600°
4
tan y positif, y dalam sukuan I dan III a = 60°, 120°, 240°, 300°
π 5 π 2 4 5
y= , π Maka, a = , π, π, π
4 4 3 3 3 3
21. tan 2x + tan 0.25π = 0 !3
26. (a) kos q = , sin q = 1 , maka q = 1 π,
tan 2x = –tan 0.25π 2 2 6
Sudut rujukan = 0.25π, 2x dalam sukuan II dan IV.
π + q = 7π
Untuk 0 < 2x < 4π, 6
2x = π – 0.25π = 0.75π –1
(b) (i) tan (π + q) = 2
π + 0.25π = 1.25π !3
2π + 0.75π = 2.75π –
2
2π + 1.25π = 3.25π
1
=– ×– 2
Maka, x = 0.375π, 0.625π, 1.375π, 1.625π 2
5 !3
22. Diberi tan2 y = . 1
12 =
sek 2 y = 1 + tan2 y !3
=1+
12( )
5 2 (ii) sek (–q) = sek q
= 2
169 !3
=
144

56

Jw01 Full SpotlightA+ MateTambahan Tg5.indd 56 26/10/2022 3:40 PM


Jawapan Matematik Tambahan

27. Sebelah kiri = tan A(kot 2A + kosek 2A) 1


=
–(–1) sin f
= tan A
kos 2A
( +
1
sin 2A sin 2A ) =
1
= tan A (
kos 2A + 1
sin 2A ) p

Kaedah alternatif
= (
sin A 2 kos2 A + 1
kos A 2 sin A kos A ) kosek (π – f) =
1
sin (π – f)
2 sin A kos2 A 1
= =
2 sin A kos2 A sin f
=1 1
=
28. Diberi bahawa sin 66° = 0.9135. p
(a) kos 24° = sin (90° – 24°)
(d) sin (3π – f) = sin 3π kos f – kos 3π sin f
= sin 66°
= 0 – (–1) sin f
= 0.9135
=p
(b) sin 294° = –sin (360° – 294°)
= sin 66° Kertas 2
= –0.9135 1. kos 43° = p dan sin 20° = q
(c) kosek (–66°) = –kosek 66° (a) p + 2q2 = kos 43° + 2 sin2 20
=– 1 = 0.7314 + 2(0.117)
sin 66° = 0.9653
= –1.0946
y
0 – (–3) 3
29. (a) Amplitud = a = = ,
2 2
π = 2π , b = 2π × 3 = 6
3 b π 1
�p 2 – 1

Translasi:
0
–3
2
( )
,c=–3
2
O �2 – p 2
x

Amplitud = a = nilai maks – nilai min


2 (b) sin 63° = sin (43° + 20)
Kala = 2π , = sin 43° kos 20° + kos 43° sin 20°
b
b = bilangan kitaran dalam 0 < x < 2π = (! 1 – p2 )(! 1 – q2 ) + pq
π = ! (1 – p2)(1 – q2) + pq
(b) Kala =
3
y
π
30. sin f = p, 0 < f <
2
(a) sin (f) = –sin f
1
= –p p
y x
O �1 – p 2

1
p β 2. (a) 2 sin 5x kos 5x = sin 10x, maka k = 10
x (b) kos 5x = p
�1 – p 2 O
kos 5x = 1 – 2 sin2 5 x
2
π π π
– f) = kos kos f + sin sin f
(b) kos ( 2 sin2 5 x = 1 – kos 5x
2 2 2 2
2 5
1–p
= 0 + sin f sin x = 2
=p 2

!
1–p
Kaedah alternatif sin 5 x = ±
π 2 2
kos ( – f) = sin f
2 3. Diberi g = kos 35° dan h = sin 33°, ungkapkan dalam
=p
sebutan g dan/atau h
1 (a) kos 57° = sin (90° – 57°)
(c) kosek (π – f) =
sin (π – f) = sin 33°
1 = h
=
sin π kos f – kos π sin f

57

Jw01 Full SpotlightA+ MateTambahan Tg5.indd 57 26/10/2022 3:40 PM


Matematik Tambahan Jawapan

(b) Lakarkan graf y = kos 2x + 1


y
untuk 0 < x < 180°.
y
1
�1 – g 2 y = kos 2x + 1
35° 2
x
O g
1 x
y = –––
(b) sin 55° = kos (90° – 55°) 180°
x
= kos 35° 0 45° 90° 135° 180°
= g
–1
y

1 Untuk mencari bilangan penyelesaian


h
33°
x
(
1 – 1
sek 2 x kosek 2 x
= x –1
180° )
O �1 – h kos 2x + 1 = x
2

180°
Lakarkan y = x
(c) sin 2° = sin (35° – 33°) 180°
= sin 35° kos 33° – kos 35° sin 33° x 0 180°
= ! 1 – g2 ! 1 – h2 – gh y 0 1
= ! (1 – g2)(1 – h2) – gh Titik (0, 0) (180°, 1)
(d) kos 68° = kos (35° + 33°) Bilangan penyelesaian = 2
= kos 35° kos 33° – sin 35° sin 33°
6. 90° < b < 270°
= g! 1 – h2 – h! 1 – g2 (a) 4 tan2 b = 1
(e) sek 70° = sek 2(35°)
tan b = ± 1
1 2
=
kos 2(35°) Sudut rujukan = 26.57°, b dalam sukuan II dan
1 III
=
2 kos 35° – 1 Maka, b = 153.43°, 206.57°
1 (b) 2 – 3 sin b – kos 2b = 0
=
2g – 1 2 – 2 sin b – (1 – 2 sin2 b) = 0
! 1 – h2 1 – 3 sin b + 2 sin2 b = 0
(f) kos 33° = 2 sin2 b – 3 sin b + 1 = 0
h
sin b = 1 atau sin b = 0.5
4. tan x = ! 3 untuk 0 < x < π. Jika sin b = 1, b = 90°
Sudut rujukan = π , x dalam sukuan I Jika sin b = 0.5, b = 150°
3
Maka, b = 90°, 150°
x= π
3 7. 2 tan x + 3 tan (45° + x)
Untuk 0 < x < 2π
(a) tan 3x = ! 3 , untuk 0 < 3x < 6π
= 2 tan x + 3 (
tan 45° + tan x
1 – tan 45° tan x )
3x = π ,
3
4π ,
3
7π ,
3
10π , 13π , 16π
3 3 3
= 2 tan x + 3 (
1 + tan x
1 – tan x )
π 4π 7π 10π 13π 16π Untuk mencari nilai x untuk 0 < x < 180°
x= , , , , , 2 tan x + 3 tan (45° + x) = 0
9 9 9 9 9 9
(b) tan2 x = 3, tan x = ±! x
x dalam sukuan I, II, III dan IV
2 tan x + 3 (
1 + tan x
1 – tan x
=0 )
2 tan x(1 – tan x) + 3 + 3 tan x = 0
π 2π 4π 5π
x= , , , 2 tan x – 2 tan2 x + 3 + 3 tan x = 0
3 3 3 3
–2 tan2 x + 5 tan x + 3 = 0
5. (a) Sebelah kiri = 12 – 1
tan x = 3, tan x = –0.5
sek x kosek 2 x
= kos2 x – sin2 x Jika tan x = 3, x = 71.57°
= kos 2x Jika tan x = –0.5, x = 153.43°
Maka, x = 71.57°, 153. 43°

58

Jw01 Full SpotlightA+ MateTambahan Tg5.indd 58 26/10/2022 3:40 PM


Jawapan Matematik Tambahan

kos a – 1
y
kos a – sek a = kos a
8. (a)
tan a sin a y =│sin 2A│
kos a 1

= kos a – 1 × kos a
2

kos a sin a
1 – sin 2
a – 1 x
= 0 π

π


–– π 5π
––

––
sin a 4 2 4 4 2
= –sin a
–1
(b) kos a – sek a = a – 1 untuk 0 < a < 2π.
tan a π
kos a – sek a = a – 1
tan a π (c) |tan A(1 + kos 2A)| = 2k – 1 hanya mempunyai
3
–sin a = a – 1 1 penyelesaian untuk 0 < A < π
π 2
jika 2k – 1 = –1
Dua graf yang sesuai untuk mencari bilangan 2k = 0
penyelesaian bagi persamaan k=0
y = sin a dan y = 1 – a 3
π 11. Lakarkan graf y = 3 sin x untuk 0 < x < 2π.
y=1– a 2
π 3 1 1
sin x= x–
2 3π 3
a 0 π
3 x
y 1 0 3 sin x= –1
2 π
Titik (0, 1) (π, 0) x
Lakar y = –1
y
π

x 0 π
y = sin α
1
y –1 0
x Titik (0, –1) (π, 0)
0 π
– π 3π
–– 2π
2 2
–1 y
α
y=1–– 3
π
2
Bilangan penyelesaian = 3 x
y=––1
9. Lakarkan y = |3 sin 2x| untuk 0 < x < 2π 1 π

y x
0 π

π


–– π 5π
––

––

–– 2π
y =│3 sin 2x│ –1 4 2 4 4 2 4
3
–2
2 3
–3 y = 3 sin – x
1 2
x
0 π

π


–– π 5π
––

––

–– 2π Bilangan penyelesaian = 3
4 2 4 4 2 4

|3 sin 2x| = k, 0 < k < 3 12. Lakar y = |4 kos 2x| untuk 0 < x < 2π
Untuk mencari bilangan penyelesaian
10. (a) Sebelah kiri = tan A(1 + kos 2A)
2π|4 kos 2x| = 4π – x untuk 0 < x < 2π:
sin A
= (1 + 2 kos2 A – 1) 2π|4 kos 2x| = 4π – x
kos A
= 2 sin A kos A 4π – x
|4 kos 2x| =

= sin 2A
(b) Lakarkan graf y = |tan A(1 + kos 2A)| untuk |4 kos 2x| = 2 – x

3
0 < A < π. Lakar y = 2 – x
2 2π
Maka, lakarkan y = |sin 2A| x 0 2π
y 2 1
Titik (0, 2) (2π, 1)

59

Jw01 Full SpotlightA+ MateTambahan Tg5.indd 59 26/10/2022 3:40 PM


Matematik Tambahan Jawapan

(c) Lakarkan graf y = kos4 x – sin4 x atau


y
y = kos 2x untuk 0 < x < 2π.
y =│4 kos 2x│
4 Untuk mencari bilangan penyelesaian bagi
3 persamaan 2π(kos4 x – sin4 x) = π – x:
2 2π(kos4 x – sin4 x) = π – x
1
x
y = 2 – –– kos4 x – sin4 x = π – x
2π 2π
0 π π 3π 5π 3π 7π
x kos 2x = 1 – x
– – –– π –– –– –– 2π 2 2π
4 2 4 4 2 4
Lakar y = 1 – x
Bilangan penyelesaian = 8 2 2π
13. (a) 1 – 3 kos x = 0 untuk 0 < x < 6.283 rad x 0 π
3 kos x = 1, kos x = 1 1 0
3 y 2
Sudut rujukan = 1.231 rad, x dalam sukuan I
dan IV. Titik (0, 1 ) (π, 0)
2
x = 1.231 rad, 5.052 rad
(b) dan (c) y
Lakarkan graf y = 1 – 3 kos x untuk 0 < x < 2π. y = kos 2x
Untuk mencari bilangan penyelesaian bagi 1
persamaan π(1 – 3 kos x) = 2x:
π(1 – 3 kos x) = 2x x
0 π π 3π π 5π 3π 7π 2π
1 – 2 kos x = 2x –
4

2
––
4
––
4
––
2
––
4
π x
y = 1 – ––
Lakar y = 2x –1 2π
π
x 0 π Bilangan penyelesaian = 4
y 0 2 15. (a) sin x kos y = m
kos x sin y = n
Titik (0, 0) (π, 2) (i) sin (x + y) sin (x – y)
= (sin x kos y + kos x sin y)
y (sin x kos y – kos x sin y)
= (m + n)(m – n)
4 = m2 – n2
2x
y = ––
3 π sin x kos y
(ii) tan x =
2 tan y kos x sin y
m
1 =
n
x (iii) sin 2x sin 2y = (2 sin x kos x)(2 sin y kos y)
0 π

π


–– π 5π
––

––

–– 2π
–1 4 2 4 4 2 4 = 4(sin x kos y)(kos x sin y)
= 4mn
–2
(b) Sebelah kiri
= kos q + sin q
Bilangan penyelesaian = 2 1 + kos 2q 1 – kos 2q
14. (a) Sebelah kiri = kos4 q – sin4 q kos q(1 – kos 2q) + sin q(1 + kos 2q)
= (kos2 q + sin2 q)(kos2 q – sin2 q) =
1 – kos2 2q
= kos2 q – sin2 q kos q(2 sin2 q) + sin q(2 kos2 q)
= kos 2q =
sin2 2q
(b) Untuk menyelesaikan persamaan 2 kos q sin 2
q + 2 sin q kos2 q
kos4 q – sin4 q = 0.2445 untuk 0 < q < 180°: =
4 sin q kos2 q
2

kos4 q – sin4 q = 0.2445 2 kos q sin2 q + 2 sin q kos2 q


kos 2q = 0.2445 =
4 sin2 q kos2 q 4 sin2 q kos2 q
Sudut rujukan = 75.85°, 2q dalam sukuan I dan 1 1
IV, 0 < 2q < 360°. = +
2 kos q 2 sin q
2q = 75.85°, 284.15°
q = 37.93°, 142.08° = sin q + kos q
2 sin q kos q
= sin q + kos q
sin 2q

60

Jw01 Full SpotlightA+ MateTambahan Tg5.indd 60 26/10/2022 3:40 PM


Jawapan Matematik Tambahan

sin a + 90°
16. (a) (i) Sebelah kiri
= 2 sin A +
π
4 (
kos A +
π
4 ) ( ) 17. h =
sin
2
90°
( π
= 2 sin A kos + kos A sin
4 4)
π 2
sin a kos + kos a sin
90° 90°
( π
kos A kos – sin A sin
4 4)
π
= 2 2
sin 45°
2 2
!2 !2
=2
2 (
sin A +
2
kos A ) sin a kos 45° + kos a sin 45°
2 2
=
!2 !2
2 (
kos A –
2
sin A ) sin 45°

!2 !2
sin a ( )
2 2
!2
( )
+ kos a
2 2
!2

= 2[( kos A) – ( sin A) ] =


2 2

2 2
( )!2
2
= 2( kos A – sin A)
1 1
2 2
2
= kos2 A – sin2 A
2
=
( )(
!2
2 )
sin a + kos a
2 2
= kos 2A
(ii) 2 kos B + (
π
4
kos B – ) (
π
4 ) ( )!2
2
a a
( π
= 2 kos B kos – sin B sin
4 4)
π = sin + kos
2 2
18. (a) A(0, 1)
( π
kos B kos + sin B sin
4 4)
π
B(90°, 3) atau , 3( )
π
!2 !2
=2 ( kos B + sin B ) 2
2
!2
2
!2
C(120°, 0) atau ( )

3
,3
(
2
kos B +
2
sin B ) D(240°, 0) atau ( )

,3
3
!2 !2
= 2[( kos B) – ( sin B) ]
2 2

2 2 (b) (i)
y
= 2( kos B – sin B)
1 1
2 2
2 2 2
= kos2 B – sin2 B
= kos 2B
(b) 2 sin A + (π
4
kos B +) (
π
4
= 0.8660 ) –2π –π O π 2π
x

untuk 0 < A < 2π.


kos 2A = 0.8660 –2
Sudut rujukan = 30°, A dalam sukuan I, IV,
0 < 2A < 4π. 2π
2A = 30°, 330°, 390°, 690° (c) Kala =
0.5
A = 15°, 165°, 195°, 345° = 4π
1 11 13 23 (d) –π < x < π
Maka, A = π, π, π, π
12 12 12 12 kos(A + B)
(c) Untuk melakar graf 19. (a) (i) = 5
kos(A – B) 3
|
y = 2 sin x +
π
4( kos x +
π
4 ) (
+1 )| 3(kos A kos B – sin A sin B)
untuk –π < x < π: = 5(kos A kos B + sin A sin B)
Lakar graf y = |kos 2x| + 1 untuk –π < x < π. 3 kos A kos B – 5 kos A kos B
= 5 sin A sin B + 3 sin A sin B
y
y = –│kos 2x│+ 1
–2 kos A kos B = 8 sin A sin B
2 kos A kos B = – 8

sin A sin B 2
kot A kot B = – 4
1
(ii) kot A kot π = – 4
4
x kot A(1) = – 4
–π 3π π π 0 π π 3π π
– –– – – – –
4 2 4

4

2
––
4 tan A = – 1
4
Sudut rujukan = 14.04°, A dalam sukuan
II dan IV

61

Jw01 Full SpotlightA+ MateTambahan Tg5.indd 61 26/10/2022 3:40 PM


Matematik Tambahan Jawapan

A = 165.96°, 194.04°
= 2.897 rad, 3.387 rad
(ii) kosek ( 2x3 – π6 ) = –2
(b) kosek q + kot q = 3 …1 sin ( 2x – π ) = – 1
kosek 2 q – kot 2 q = 1 3 6 2
Faktorkan: Sudut rujukan = π ,
(kosek q + kot q)(kosek q – kot q) = 1 …2 6
Gantikan 1 dalam 2
3(kosek q – kot q) = 1
Sukuan III: 2x
3 (
– π =π+ π
6 6 )
2x =π+ π + π
(kosek q – kot q) = 1 …3 3 6 6
3
x = 2π
Persamaan 1 – 3:
(b) (i) y = 2x
2 kot q = 8 4 sin q + 5 kos q = 2(5 sin q – 4 kos q)
3
4 sin q + 5 kos q = 10 sin q – 8 kos q
kot q = 1 , tan q = 6 = 3 13 kos q = 6 sin q
4 8 4
!7 tan q = 13
20. (a) tan A = 6
3 q = 65.22°
!7 (ii) x2 + y2
(i) sin A = –
4 = (5 sin q – 4 kos q)2 + (4 sin q + 5 kos q)2
( )
(ii) kos 2π kos A + sin 2π sin A = (1) –3 + 0
4
= 25 sin2 q – 40 sin q kos q + 16 sin2 q +
40 sin q kos q + 25 kos2 q
=– 3 = 41 sin2 q + 41 kos2 q
4 = 41(sin2 q + kos2 q)
(
2 )
(iii) sin π – A = sin π kos A – kos 3 π sin A
3 3
2 2
= 41

= –1 –3 – 0
4 ( ) Bab 7 Pengaturcaraan Linear

= 3

4 Zon Formatif 7.1
3
2 (
Maka, kosek π – A = 3 )4 1. (a)
tan A + tan B y
(b) tan (A + B) =
1 – tan A tan B 6
3 11 1 4
+ y<–x+3
2 7 2 2
=
1–
3 7( )( )
2 11
–6 –4 –2 0 2
x
–2
11
= × (–21)
7
= – 47
tan (A + B) + tan C (b) y
tan (A + B + C) =
1 – tan (A + B) tan C 3
23 2
– 47 + 2
24 y>–x–1 1
= 5
1 – (– 47)
23
24 ( ) –3 –2 –1 0 1
x
–1
= – –1 105 × 24
24 1 105 (c)
= –1 y

2
21. (a) (i) |4 sin 3x | = 2, 0° < 3x < 120° x
–2 0 2 4 6
x 2
Jika 4 sin = 2
3 4 y<8–x–5
x 1 6 3
sin =
3 2
Sudut rujukan = 30°
x
= 30°, 150° (tak mungkin)
3
x = 90°

62

Jw01 Full SpotlightA+ MateTambahan Tg5.indd 62 26/10/2022 3:40 PM


Jawapan Matematik Tambahan

(d) 3. (a) Katakan x ialah peserta kursus A


y
y ialah peserta kursus B
x 3y > x
–8 –6 –4 –2 0 (b) Katakan x ialah pekerja perempuan
–1
4 –2 y ialah pekerja lelaki
y > –– x – 3
9 –3 y – x < 40
–4 (c) 200y + 100x < 2 000
2y + x < 20
2. (a) 4. (a) x < 4; y , 8
y y x<4

4 8 y<8
2
y = –x + 2
23 6
x
–4 –2 0 2 4 4
–2
–4
2
Lorek berada di bahagian atas garis putus- 0 x
putus. Maka, ketaksmaan linear rantau berlorek 1 2 3 4
diwakili oleh y . 2 x + 2. (b) x , 7; y > 1
3
(b)
y x<7
y
8
6
7 4 6
y = –x + 5
3
2
4
x
–3 –2 –10 1
–2 2
y>1
Lorek berada di bahagian bawah garis putus- 0 x
2 4 6 8
putus. Maka, ketaksmaan linear rantau
berlorek diwakili oleh y , 7 x + 5. (c) x > –2; y > 4
3 x > –2 y
(c)
y
6
4
3 y>4 4
8
2 y = – –x + 4
3 2
1
x x
–10 1 2 3 –2 0 2 4
Lorek berada di bahagian atas garis padu. (d) x + y > 2; y , 6
Maka, ketaksmaan linear rantau berlorek y y < 3x
diwakili oleh y > – 8 x + 4.
3 4
(d)
3 y > 2x
y
2
4 1
2 x
x 0 1 2 3 4
–2 0 2 4 6
–2 5 (e) x + y < 4; x + y . 1
–4 y = –x
2
–4
y

Lorek berada di bahagian bawah garis padu. 4


Maka, ketaksmaan linear rantau berlorek 3 x+y<4
diwakili oleh y < 5 x – 4. 2
2 1
x+y>1
x
0 1 2 3 4

63

Jw01 Full SpotlightA+ MateTambahan Tg5.indd 63 26/10/2022 3:40 PM


Matematik Tambahan Jawapan

(f) x > y; x . 1 7. Menggunakan Ujian Titik:


y x>1 Diberi (2, 1). Gantikan x = 2 dan y = 1.
Persamaan 1:
4 x+y.1
3 x>y 2+1.1
2 3 . 1 (Benar)
1 Oleh itu, (2, 1) adalah peyelesaian untuk
x ketaksamaan 2x + y , 8.
0 1 2 3 4
Persamaan 2:
(g) y < 2x; x > y + 2 2x + y , 8
y y < 2x 3 , 8 (benar)
Oleh itu, (2, 1) adalah peyelesaian untuk 2x + y , 8
8
Maka, titik (2, 1) ialah penyelesaian untuk x + y . 1
6 x>y+2 dan 2x + y , 8.
4
8. (a) Garis padu HK adalah selari dengan paksi-y
2
pada x = 6. Maka, persamaan HK ialah x = 6.
x
0 2 4 6 8 (b)
(h) y , 3x; y > 2x Keadaan
Garis Persamaan
y y < 3x rantau
4 Di sebelah kiri
HK x<6
3 y > 2x garis padu, x = 6
2
Di bahagian atas
1
garis padu,
x GK y > – 5x + 5
0 1 2 3 4 y = – 5x + 5 8
8
5. Di bahagian
x=0 bawah garis padu,
GH y < 1x + 5
y < 2x + 1 y = 1x + 5 6
8
6
y>x Maka, ketaksamaan yang memenuhi rantau
6
berlorek, GHK ialah x < 6, y > – 5 x + 5 dan
8
4 y < 1 x + 5.
6
2 x+y<8
9. y < x + 1; y . –3; x < 4
y=0
0 2 4 6 8 y

6
6.
4
y y < 2x – 3
2
4 y<x+1
x
2 –6 –4 –2 0 2 4 6
–2
y > –3
x –4
–4 –2 0 2 4
–2 –6
–4
x<4
y > 2x + 1

Ketaksamaan tidak mempunyai titik yang


bersilang. Maka, ketaksamaan ini tiada
penyelesaian.

64

Jw01 Full SpotlightA+ MateTambahan Tg5.indd 64 26/10/2022 3:40 PM


Jawapan Matematik Tambahan

10. y (b) Daripada graf di (a), titik maksimum ialah


(3.158, 3.684).
12
Gantikan x = 3 dan y = 3.
k = 2(3) + 5(3)
= 21
10

8
Zon Formatif 7.2

1. (a) y , 4(x + y)
6 (b) x > 0, y > 0, 2 500x + 500y < 50 000
x + y < 60
4 (c) x > 0, y > 0, 2x + 3y > 90, 3x + 2y < 180
2x – y > 40
2 2. (a) –
(b) k = 250x + 75y
x (c) k = 60x + 40y
0 2 4 6 8 10 12
3. (a) x = bilangan lampu L1,
Garis lurus A: y = bilangan lampu L2
Kecerunan, m = 12 – 0 Tukarkan masa minit kepada jam.
0–8
=–3 20 min = 1 jam, 30 min = 1 jam,
2 3 2
Pintasan-y, c = 12
10 min = 1 jam
Persamaan garis lurus A ialah y = – 3 x + 12. 6
2 x > 0, y > 0, 1 x + 1 y < 100
Rantau berada di bahagian bawah garis padu, 3 2
1 x + 1 y < 80
y < – 3 x + 12. 3 6
2
Garis lurus B: (b) y
Kecerunan, m = 9 – 0
0–9
= –1 500
Pintasan-y, c = 9
Persamaan garis lurus B ialah y = –x + 9. 400
Rantau berada di bahagian bawah garis padu,
y < –x + 9.
300 1
Garis lurus C: –x + 1
– y < 80
3 6
Kecerunan, m = 6 – 0
0 – 12 200
=–1
2 1
–x + 1
– y < 100
Pintasan-y, c = 6 100 2 2

Persamaan garis lurus C ialah y = – 1 x + 6.


2
Rantau berada di bahagian bawah garis padu x
0 100 200 300
y < – 1 x + 6.
2
11. (a) y
(c) Fungsi objektif, 15x + 10y = k
12 Katakan k = 150
15x + 10y = 150
10 Daripada graf, titik maksimum adalah pada
8
(210, 60).
y < –2x + 10 Gantikan x = 210 dan y = 60 ke dalam fungsi
6 objektif.
4 Keuntungan maksimum = 15(210) + 10(60)
5 x+5
y < – ––
12 = RM3 750
2
Maka, keuntungan maksimum ialah RM3  570
x dengan bilangan model lampu L1 dan L2
0 2 4 6 8 10 12 masing-masing ialah 210 unit dan 60 unit.

65

Jw01 Full SpotlightA+ MateTambahan Tg5.indd 65 26/10/2022 3:40 PM


Matematik Tambahan Jawapan

4. (a) x > 0, y > 0, x + 5y > 15, 5x + y > 15


y
(b)
y
120
15
100

12 80
60
9 40 200x1 + 500x2 = 30 000
5x + y > 15 20
6
x
0 20 40 60 80 100 120 140 160
3 300x1 + 400x2 = 27 000
x + 5y > 15
400x1 + 300x2 = 25 000
0 x
3 6 9 12 15
Fungsi objektif, 20 000x + 25 000y = k
(c) Fungsi objektif, 10x + 30y = k 4x + 5y = k
Katakan k = 300 Katakan k = 20
x + 3y = 30 4x + 5y = 20
Daripada graf, titik minimum adalah pada
Daripada graf, titik minimum adalah pada (25, 50).
(5, 5 .
2 2 ) Gantikan x1 = 25 dan x2 = 50 ke dalam fungsi
Gantikan x = 5 dan y = 5 ke dalam fungsi objektif.
2 2
objektif. Kos minimum
Kos minimum = 10 5 + 30 5
2 2 () () = 20 000(25) + 25 000(50)
= RM1 750 000
= 100
Maka, kilang 1 dan kilang 2 masing-masing perlu
5. Biarlah t mewakili bilangan iklan TV beroperasi selama 25 hari dan 50 hari dengan kos
r mewakili bilangan iklan radio minimumnya ialah RM1 750 000
t > 20, r > 30, 2r < t < 3r

y r = 30 Zon Sumatif
t = 3r
200
Kertas 1
160 1.
120 t = 120 y

80 t = 2r 6
x>3
40 5
x+y<5
x
0 20 40 60 80 4

3
Fungsi objektif, 8 000t + 2 000r = k
2
Kos minimum berlaku di bucu di mana t = 120 dan
t = 3r bertemu. Ini adalah titik r = 40, t = 120. Oleh 1
y>1
itu, ahli potilik tempatan itu harus membeli 40
x
iklan radio dan 120 iklan TV. 0 1 2 3 4 5
5. 40 iklan radio dan 120 iklan TV.
6. Katakan x1 ialah kilang 1
x2 ialah kilang 2
Kekangan:
400x1 + 300x2 > 25 000 (Gred tinggi)
300x1 + 400x2 > 27 000 (Gred sederhana)
200x1 + 500x2 > 30 000 (Gred rendah)

66

Jw01 Full SpotlightA+ MateTambahan Tg5.indd 66 26/10/2022 3:40 PM


Jawapan Matematik Tambahan

2. 4. (a)
y y

x < 80 8
100 7 x>2
y < 90 6
80 y>x
5
60 4
3 R
40
2
R x + y < 110 x+y<6
20 1
x
0 1 2 3 4 5 6 7
x
0 20 40 60 80 100
(b) Titik R yang mungkin:
(2, 2), (3, 3), (2, 3) dan (2, 4)
3.
5. I: x > 3
y
II: x + y , 12
III: 5x + 13y . 100
10 6.
y
8
8
6 R 7 x>2
6
4 y>x–1
5
2 4
3
x
0 1 2 3 4 5 2
1 2x + y > 8
Garis lurus A: x
Garis putus-putus adalah selari dengan paksi-y pada 0 1 2 3 4 5 6 7 8
x = 4. Rantau berada di sebelah kiri garis lurus.
Maka, ketaksamaan garis lurus A ialah x , 4. Daripada graf di atas, titik minimum adalah pada
(3, 2). Maka, nilai minimum y dalam rantau ini
Garis lurus B: ialah y = 2.
Kecerunan, m = 2 – 10 7.
0–4
=2
A B Jumlah
Pintasan-y, c = 2
Persamaan garis lurus B ialah y = 2x + 2. Bijirin 5 2 180
Rantau berada di bahagian bawah garis padu,
y < 2x + 2.
Tepung 3 3 135
Garis lurus C:
Garis padu adalah selari dengan paksi-x pada y = 4. Katakan x = bilangan roti A
Rantau berada di bahagian atas garis lurus. Maka, y = bilangan roti B
ketaksamaan garis lurus C ialah y > 4. Kekangan:
5x + 2y < 180
Maka, tiga ketaksamaan rantau R ialah x , 4,
x + y < 45
y < 2x + 2 dan y > 4.
x > 0, y > 0

67

Jw01 Full SpotlightA+ MateTambahan Tg5.indd 67 26/10/2022 3:40 PM


Matematik Tambahan Jawapan

Graf: 9. (a) x < 20, y < 30, y > x, y , 2x


y (b)
y
100
y < 30
80 30
5x + 2y = 180
60
25
40
20 y < 2x
20
x + y = 45
x 15
0 10 20 30 40 50
10 y>x
Fungsi objektif, 40x + 30y = k
Katakan k = 120
5 x < 20
4x + 3y = 12
Daripada graf, titik maksimum adalah pada (30, 15). x
Gantikan x = 30 dan y = 15 ke dalam fungsi 0 5 10 15 20 25
objektif.
Keuntungan maksimum = 40(30) + 30(15)
= RM1 650 (c) Fungsi objektif, 40x + 50y = k
Katakan k = 200
Maka, kedai roti Yum memerlukan 30 roti A
4x + 5y = 20
dan 15 roti B untuk memperoleh keuntungan
Daripada graf, titik maksimum adalah pada
maksimum, iaitu RM1 650.
(20, 30).
8. (a) 200x + 100y < 1 500 Gantikan x = 20 dan y = 30 ke dalam fungsi
2x + y < 15 objektif.
(b) (i) y > x
(ii) Keuntungan maksimum = 40(20) + 50(30)
y
= RM2 300
8 Peruntukan yang disediakan hanya RM2 000.
y>x 10. (a) x = bilangan X botol cecair
7
y = bilangan Y botol cecair
6 4x + 3y < 84 bermaksud botol cecair A yang
5 boleh dihasilkan adalah tidak boleh lebih
4 daripada 84  ml.
2x + y < 15 (b) 2x + 5y < 90
3
(c)
2
y
1
x
0 1 2 3 4 5 6 7 8
30
(c) (i) Fungsi objektif, k = 80x + 60y
Katakan k = 480 4x + 3y < 84
20
4x + 3y = 24
Daripada graf, titik maksimum adalah
pada (5, 5). 10
Gantikan x = 5 dan y = 5 ke dalam fungsi 2x + 5y < 90
objektif. x
Kos minimum = 80(5) + 60(5) 0 10 20 30 40 50
= RM700
(ii) Jika y . x dibuang, kos minimum akan (d) Daripada graf, titik maksimum adalah pada
menjadi (11, 13).
Kos minimum = 80(7) + 60(1) Maka, maksimum bilangan botol X dan Y
= RM620 masing-masing ialah 11 botol dan 13 botol.
Maka, bilangan van kecil yang diperlukan
perlu dikurangkan 1 van sahaja.

68

Jw01 Full SpotlightA+ MateTambahan Tg5.indd 68 26/10/2022 3:40 PM


Jawapan Matematik Tambahan

11. Katakan x bilangan meja yang dibeli


y
y bilangan kerusi yang dibeli
Kekangan: x > 0 12
y>0
10
5x + y < 100 A(0, 8)
x + y < 60 8
y 6
B(2, 4)
4
100
2 x + 2y = 10
90 C(10, 0)
x
80 0 2 4 6 8 10 12
70 2x + y = 8 5x + 7y = 38
C(0, 60)
60 Fungsi objektif, 50x + 70y = k
50 B(10, 50) Katakan k = 380
5x + 7y = 38
40
Daripada graf, titik minimum adalah pada (2, 4).
30 Gantikan x = 2 dan y = 4 ke dalam fungsi objektif.
20 Keuntungan maksimum
= 50(2) + 70(4)
10
A(20, 0) = RM380
x
0 10 20 30 40 50 60 70 Maka, strategi campuran yang boleh dibuat oleh
5x + y = 100 x + y = 60 pakar makanan ialah 2 kg makanan ‘I’ dan 4 kg
makanan ‘II’, Dengan itu, kos minimum campuran
Fungsi objektif, 250x + 75y = k ialah RM380.
13. (a) Katakan x ialah raket tenis
Bucu untuk rantau Nilai k yang
y ialah raket kriket
tersaur sepadan
Kekangan: x > 0
O (0, 0) 0 y>0
1.5x + 3y < 42
C (0, 60) 4 500 3x + y < 24
B (10, 50) 6 250 y

A (20, 0) 5 000 25
20
Menggunakan Ujian Titik, penjual itu boleh 3x + y = 24
mendapatkan keuntungan maksimum, iaitu 15
RM6  250 dengan menjual 10 buah peti ais dan 50 10
buah mesin basuh. 1.5x + 3y = 42
5
12 Katakan campuran mengandungi: x
x kg makanan ‘I’ 0 5 10 15 20 25 30
y kg makanan ‘II’.
Diberi: Daripada graf, titik maksimum adalah pada
Makanan (4, 12).
Bahan Keperluan Maka, bilangan raket tenis dan kriket
I II masing-masing ialah 4 dan 12 dengan
Vitamin A (unit/kg) 2 1 8 memaksimumkan masa penggunaan mesin
dan tenaga kerja.
Vitamin C (unit/kg) 1 2 10 (b) Fungsi objektif, k = 20x + 10y
Keuntungan maksimum
Kos (RM/kg) 50 70 = 20(4) + 10(12)
= 200
Kekangan: x > 0
y>0 14. (a) x + 2y < 70,
2x + y > 8 3x + 2y < 120,
x + 2y > 10 x < 2y

69

Jw01 Full SpotlightA+ MateTambahan Tg5.indd 69 26/10/2022 3:40 PM


Matematik Tambahan Jawapan

(b) Katakan k = 150


1.5x + y = 150
y Dari graf, titik minimum adalah pada
(500, 500).
50
Gantikan x = 500 dan y = 500 ke dalam
45
fungsi objektif
40 Kos minimum = 1.5(500) + 1(500)
35 = RM1 250
x < 2y (ii) Fungsi objektif, 0.5x + y = k
30
Katakan k = 500
25 0.5x + y = 500
20
Dari graf, titik minimum adalah pada
15 R (500, 1 000).
10 x + 2y < 70
Gantikan x = 500 dan y = 1 000 ke dalam
5 fungsi objektif
3x + 2y < 120
x Keuntungan maksimum
0 10 20 30 40 50 60 70 80 = 0.5(500) + 1(1 000)
= RM1 250
(c) (i) Diberi y = x + 10 16. (a) 5x + 3y > 60, x + y < 10, y > 3x
xmaksimum = 16 (b)
ymaksimum = 26
y
Mi goreng biasa = 16
Mi goreng istimewa = 26 24
y > 3x
(ii) Fungsi objektif, 5x + 7y = k 20
Katakan k = 35 16
5x + 7y = 35
Daripada graf, titik maksimum adalah 12
5x + 3y < 60
pada (24, 23). 8
Gantikan x = 24 dan y = 23 ke dalam 4
fungsi objektif. x + y > 10
x
Keuntungan maksimum 0 2 4 6 8 10 12
= 5(24) + 7(23)
= RM281 (c) (i) Apabila arca A ialah 30 unit,
x = , y = 6.
15. (a) x > 500,
Maka, bilangan minimum arca B dalam
y > x,
julat 4 < x < 8.
x + y < 1 500
(ii) Fungsi objektif, 100x + 80y = k
(b)
Katakan k = 180
y
10x + 8y = 18
Daripada graf, titik minimum adalah pada
(3, 7).
1 600 Gantikan x = 3 dan y = 7 ke dalam fungsi
x > 500
objektif.
1 200
Kos minimum = 100(3) + 80(7)
x>y = RM860

800 Bab 8 Kinematik Gerakan Linear

400
x + y < 1 500 Zon Formatif 8.1

1. (a) (i) 0m
x (ii) 12 m
0 400 800 1 200 1 600 (iii) –10 m
(b) (i) Sesaran seketika
(ii) • Apabila t = 0, zarah berada di 0, iaitu
(c) (i) Diberi kos = 1.5x + y sesaran = 0 m.
Fungsi objektif, 1.5x + y = k

70

Jw01 Full SpotlightA+ MateTambahan Tg5.indd 70 26/10/2022 3:40 PM


Jawapan Matematik Tambahan

• Apabila t = 2, zarah berada 12 m di (b)


sebelah kanan. Sesaran A ialah 12 m. s
• Apabila t = 4, zarah berada 10 m di
sebelah kiri. Sesaran B ialah –10 m. 20
2. (a) (i) a. t = 1, s = 5
15
t = 3, s = 9
t = 6, s = 0 10 s = t 2 – 8t + 15
t = 7, s = –7
(ii) s = –16 5
6t – t2 = –16 t
t2 – 6t – 16 = 0 0 2 4 6
(t + 2)(t – 8) = 0
t = –2, t = 8 4. (a) v = t2 – 2t – 15
ºt=8s t = 0, v = –5 m s–1
(iii) s = 5 (b) Zarah berhenti, v = 0
6t – t2 = 5 t2 – 2t – 15 = 0
t2 – 6t + 5 = 0 (t + 3)(t – 5) = 0
(t – 1)(t – 5) = 0 t = 5, t = –3
t = 1, t = 5 (Masa tidak boleh negatif)
(iv) s = 0 Maka, t = 5.
6t – t2 = 0 (c) Zarah bergerak ke kiri, v , 0
t(6 – t) = 0 t2 – 2t – 15 , 0
t = 0, t = 6
t = 6, zarah melalui O semula.
(b) Garis nombor:
t=1
t=3
t
–7 m 0 5m 9m –3 5
t=7 t=6
0<t<5
Graf sesaran-masa (d)
s = 6t – t2
v
t = 0, s = 0
s = 0, 6t – t2 = 0
20
t(6 – t) = 0
t = 0, t = 6 v = t 2 – 2t – 15
15
s
10
10 5
s = 6t – t 2
5
t
t –2 0 2 4 6 8
0 2 4 6 8 –5
–5
–10 –10

–15 –15

3. (a) s . 0 5. (a) v = 6t – 12
t2 – 8t + 15 . 0 t = 2, v = –12 m s–1
(t – 3)(t – 5) = 0 (b) 6t – 12 = 6
t = 3, t = 5 t = 3
(c) 6t – 12 . 0
t . 2 s
(d) 6t – 12 , 0
t , 2 s
t
3 5

0<t<3

71

Jw01 Full SpotlightA+ MateTambahan Tg5.indd 71 26/10/2022 3:40 PM


Matematik Tambahan Jawapan

6. (a) t = 0, a = –9 m s–2 Jarak saat ke-5 = 5 – 0


(b) a = 4t – 9 =5m
t=3 (c) Jumlah jarak 7 saat pertama
a = 3 m s–2 t = 0 ˜ t = 2, jarak = 4 m
(c) Halaju seragam, a = 0 t = 2 ˜ t = 5, jarak = 9 m
4t – 9 = 0 t = 5 ˜ t = 7, jarak = 16 m
t= 9 s Jumlah = 4 + 9 + 16 = 29 m
4
(d) Nyahpecutan, a , 0
4t – 9 , 0 Zon Formatif 8.2
t, 9 s 1. (a) Diberi fungsi sesaran,
4
7. (a) t = 5, s = 25 m s = 1 t4 – 5 t3 + 8t
(b) 10t – t2 = 0 4 3
t(10 – t) = 0 Fungsi halaju, v = ds
dt
t = 0, t = 10
v = t3 – 5t2 + 8
º t = 10, melalui O semula
(c) t = 6, s = 24 Fungsi pecutan, a = dv
dt
t = 7, s = 21 a = 3t2 – 10t
Jarak saat ke-7 (b) Diberi fungsi sesaran, s = 2(4 – 3t)3
= Jarak dari t = 6 hingga t = 7
= 24 – 21 Fungsi halaju, v = ds
dt
=3m v = –18(4 – 3t)2
(d) Jumlah jarak dalam 10 saat pertama
t = 0, s = 0 Fungsi pecutan, a = dv
dt
t = 5, s = 25 m (sebelum bergerak ke kiri) a = 108(4 – 3t)
t = 6, s = 24 m
(c) Diberi fungsi sesaran, s = 6t3 – 42
t = 7, s = 21 m t
t = 10, s = 0 Fungsi halaju, v = ds
dt
t=0 v = 18t2 + 83
t
t=5 Fungsi pecutan, a = dv
dt
0 21 24 25 a = 36t – 244
t
t = 10 t=7 t=6 2. s = 36 + 5t – t2
(a) ds = 5 – 2t
Jumlah jarak = 25 + 25 dt
= 50 m Halaju awal, t = 0
8. (a) t = 0, s = 5 v = 5 – 2(0)
s = 0, 5 + 4t – t2 = 0 = 5 m s–1
t = –1, t = 5 (b) Zarah berhenti, v = 0
t = 2, s = 9 m 5 – 2t = 0
t = 7, s = –16 m 2t = 5
s t = 5 s
2
10 (c) Sesaran maksimum, ds = 0, v = 0
s = 5 + 4t – t 2 dt
t = 2.5 s
5
(d) Halaju negatif, v , 0
t 5 – 2t , 0
0 2 4 6 8
–5 2t . 5
–10 t . 2.5 s
(e) Zarah bergerak ke kanan, v . 0
–15 5 – 2t . 0
–20 t , 2.5 s
1
3. (a) s = t + 4t – 15t
3 2

(b) Jarak saat ke-5 3


ds = v = t2 + 8t – 15
= Jarak dari t = 4 hingga t = 5
dt
t = 4, s4 = 5 m
Halaju awal, t = 0
t = 5, s5 = 0 m
v = –5 m s–1

72

Jw01 Full SpotlightA+ MateTambahan Tg5.indd 72 26/10/2022 3:40 PM


Jawapan Matematik Tambahan

(b) Zarah berhenti, z = 0 7. (a) s = t3 – 15 t2


t2 + 8t – 15 = 0 2
(t – 3)(t – 5) = 0 ds = v = 3t2 – 15t
t = 3 s, t = 5 s dt
(c) Halaju positif, v . 0 ds = a = 6t – 15
dt
t2 + 8t – 15 . 0
Zarah berpatah balik,
v = 0
3t2 – 15t = 0
3t(t – 5) = 0
t = 0, t = 5 (zarah berpatah balik)
t
3 5 s = 53 – 15 (5)2
2
= –62.5 m
0 < t , 3, t . 5 (b) Pecutan positif, a . 0,
4. (a) v = 3t2 – 18t – 21 6t – 15 . 0
dv = a = 6t – 18 t . 2.5 s
dt (c) Zarah berhenti, v = 0
Pecutan awal, t = 0 t = 5, a = 6(5) – 15
a = –8 m s–2 = 15 m s–2
(b) t = 2, a = 6(2) – 18
= –6 m s–2 Zon Formatif 8.3
(c) Halaju seragam, a = 0
6t – 18 = 0 1. (a) a = 4 – 2t
t=3s
(d) Pecutan negatif, a , 0

v = (4 – 2t) dt
v = 4t – t2 + c
6t – 18 , 0
t = 0, halaju awal, v = –4, c = –4
t,3s
º v = 4t – t2 – 4
3 9
5. (a) s = t – t + 6t
2
t = 4, v = 4(4) – 42 – 4
2
ds = v = 3t2 – 9t + 6 = –4 m s–1
dt (b) a = 0
dv = a = 6t – 9 4 – 2t = 0
dt t = 2
Halaju awal, t = 0, v = 6 m s–1 v = 4(2)2 – 22 – 4
(b) Pecutan awal, t = 0, a = –9 m s–2 = 0 m s–1
(c) Zarah berhenti, v = 0 2. (a) a = 3 – 2t
3t2 – 9t + 6 = 0
t2 – 3t + 2 = 0 ∫
v = (3 – 2t) dt
(t – 1)(t – 2) = 0 v = 3t – t2 + c
t = 1, t = 2 t = 0, v = –2, c = –2
t = 1, a = 6(1) – 9 = –3 m s–2 º v = 3t – t2 – 2
t = 2, a = 6(2) – 9 = 3 m s–2 t = 3, v = 3(3) – 32 – 2
= –2 m s–1
6. (a) v = 4t – t2
(b) Zarah berhenti, v = 0
v = 3, 4t – t2 = 3
3t – t2 – 2 = 0
t2 – 4t + 3 = 0
t2 – 3t + 2 = 0
(t – 1)(t – 3) = 0
(t – 1)(t – 2) = 0
t = 1, t = 3
t=1,t=2
t = 1, a = 4 – 2(1) = 2 m s–2
t = 3, a = 4 – 2(3) = –2 m s–2 (c) Halaju maksimum, dv = 0
dt
(b) Halaju maksimum, dv = 0 a=0
dt
a=0 3 – 2t = 0
4 – 2t = 0 t = 3
t = 2 2
v = 4(2) – 22 = 4 m s–1
Halaju maksimum = 3 3 – 3
2( ) ( ) –2
2
2

1
= ms –1
4
[vmaksimum, d v2 = –2 (, 0)]
2

dt

73

Jw01 Full SpotlightA+ MateTambahan Tg5.indd 73 26/10/2022 3:40 PM


Matematik Tambahan Jawapan

3. (a) v = 16 – 2t Di B, t = 4

s = (16 – 2t) dt sB = 43 – 11 (4)2 – 4(4)
2
s = 16t – t2 + c = –88 m
t = 0, s = 0, c = 0 Jarak AB = |sA – sB|
s = 16t – t2
= |–88 – (–8 1 )|
t = 3, s = 16(3) – 33 = 37 m 2
= 79.5 m
(b) Jarak dilalui dalam saat ke-6,
(c) (i)
s6 = 60, s5 = 55
|s6 – s5| = |60 – 55| t 0 4 6
=5m v –4 0 133
(c) 16 – 2t = 8
t = 4, v
s = 16(4) – 42
= 5 m 40
4. a = 16 – 2t 30
v = 3t 2 – 11t – 4

v = (16 – 2t) dt 20
v = 16t – t2 + c 10 B
t = 0, v = 5, c = 5 t
v = 16t – t2 + 5 0 1 2 3 4 5 6
–10 A

s = (16t – t2 + 5) dt
–20
s = 8t2 – t + 5t + c
3

3
t = 0, s = 0, c = 0 (ii) Jumlah jarak 6 saat pertama
s = 8t2 – t + 5t
3
= Luas A + Luas B
3

4
Luas A = (3t2 – 11t – 4) dt
(a) t = 2, s = 8(2)2 – 2 + 5(2)
3
0
3
= 34 1 m
3
[
= t3 – 11 t2 – 4t
2
4
0
]
= |–88|
(b) Pecutan sifar, 16 – 2t = 0 = 88 m
t=8s

6
s = 8(8)2 – 8 + 5(8)
3
Luas B = (3t2 – 11t – 4) dt
3 4

= 381 1 m
3
[
= t3 – 11 t2 – 4t
2
6
4
]
= 263 m
Zon Formatif 8.4
Jumlah jarak 6 saat pertama
1. (a) v = 3t2 – 11t – 4 =A+B
Zarah P berhenti di B, v = 0 = 88 + 263
3t2 – 11t – 4 = 0 = 351 m
(3t + 1)(t – 4) = 0 2. s = 6t2 – t3
t = – 1 , t= 4 v = dv
3 dt
Pecutan, a = dv = 6t – 11 = 12t – 3t2
dt
t = 4, a = 6(4) – 11 a = dv
dt
= 13 m s–2
= 12 – 6t

(b) s = (3t2 – 11t – 4) dt (a) t = 3, v = 12(3) – 3(3)2
= 9 m s–1
s = t3 – 11 t2 – 4t + c
2 (b) vmaksimum, dv = a = 0
t = 0, s = 0, c = 0 dt
s = t3 – 11 t2 – 4t 12 – 6t = 0
2 t=2
Di A, t = 1 Apabila t = 2, v = 12(2) – 3(2)2
sA = 13 – 11 (1)2 – 4(1) = 24 – 12
2
= 12 m s–1
= –8 1 m
2

74

Jw01 Full SpotlightA+ MateTambahan Tg5.indd 74 26/10/2022 3:40 PM


Jawapan Matematik Tambahan

(c) Kembali ke A, s = 0 (d)


6t2 − t3 = 0 t 0 2 4 6
t2(6 − t) = 0
t = 0, t = 6 v –12 0 4 0
(d) P berhenti, v = 0 s
12t − 3t2 = 0
3t(4 − t) = 0 8
v = 8t – t 2 – 12
t = 0, t = 4 (Selepas melalui A) 4
Masa kembali ke A, t = 6 B
t
(e) Jarak dalam 5 saat pertama 0 1 2 3 4 5 6
t = 0, s = 0 –4 A
t = 4, s = 32 –8
t = 5, s = 25 –12

t=0 1 –16

t=4

2
Luas A = (8t – t2 – 12) dt
A 25 32 0

t=5 2
[
= 4t2 – t – 12t
3
3 2
0
]
Jumlah jarak dalam 5 saat pertama
[ 2
= –10 – 0
3 ]
= (1) + (2) = 10 2 m
= 32 + (32 – 25) 3

5
= 32 + 7 Luas B = (8t – t2 – 12) dt
2
= 39 m
3. a = 8 – 2t [
= 4t2 – t – 12t
3
3 5
2
]

v = (8 – 2t) dt =9m
v = 8t – t2 + c Jumlah jarak 6 saat pertama
v = 8t – t2 – 12 = 10 2 + 9
3

s = (8t – t2 – 12) dt
= 19 2 m
s = 4t2 – t –12t + c
3
3
3 4. a = 8 – 6t
t = 0, s = 0, c = 0
s = 4t2 – t – 12t
3 ∫
v = (8 – 6t) dt
3 v = 8t – 3t2 + c
(a) Zarah melalui O semula, s = 0 t = 0, v = 5, c = 0
4t2 – t – 12t = 0
3
v = 8t – 3t2 + 5
3
t3 – 12t2 + 36t = 0 ∫
s = (8t – 3t2 + 5) dt
t(t – 6)(t – 6) = 0 s = 4t2 – t3 + 5t + c
t = 0, t = 6 t = 0, s = 4 dari titik P
Melalui O semula, t = 6 s c=4
s = 4t2 – t3 + 5t + 4
(b) vmaksimum, dv = a = 0 (a) t = 3, s = 4(3)2 – (3)3 + 5(3) + 4
dt
8 – 2t = 0 = 28 m
t=4s (b) t = 0, a = 8 m s–2
d2v = –2 (, 0) (c) Zarah melalui titik O semula, s = 4
dt2 4t2 – t3 + 5t + 4 = 4
º v adalah maksimum t3 – 4t2 – 5t = 0
t = 4, v = 8(4) – 42 – 12 t(t2 – 4t – 5) = 0
= 4 m s–1 t(t + 1)(t – 5) = 0
(c) Zarah berhenti, v = 0 t = 0, t = 5
8t – t2 – 12 = 0 (Masa melalui titik O semula)
t2 – 8t + 12 = 0 v = 8t – 3t2 + 5
(t – 2)(t – 6) = 0 = 8(5) – 3(5)2 + 5
t = 2 s, t = 6 s = –30 m s–1

75

Jw01 Full SpotlightA+ MateTambahan Tg5.indd 75 26/10/2022 3:40 PM


Matematik Tambahan Jawapan

5. (a) (i) 2t – 8 , 0 (a) a = p + qt


0<t,4 p + q(0) = 4

(ii) s = (2t – 8) dt p = 4
a = 4 + qt
s = t2 – 8t + c
s = 4, t = 3 ∫
v = (4 + qt) dt
4 = 32 – 8(3) + c q 2
c = 19 v = 4t + t +c
2
s = t2 – 8t + 19 t = 0, v = –8, c = –8
t = 0, s = 19 q
v = 4t + t2 – 8
OA = 19 m 2
q
(iii) Zarah berhenti, v = 0
2t – 8 = 0

s = (4t + t2 – 8) dt
2
q 3
t = 4 s = 2t + t – 8t + c
2
6
s = 42 – 8(4) + 19
t = 0, s = 0, c = 0
= 3 m
t = 1, s = –8
(iv) Zarah kembali ke A, s = 19 q
t2 – 8t + 19 = 19 2(1)2 + (1)3 – 8(1) = –8
6
t2 – 8t = 0 q = –12
t(t – 8) = 0 p = 4, q = –12
t = 0, t = 8 (masa kembali ke A) a = 4 – 12t
v = 2(8) – 8 (b) v = 4t – 6t2 – 8
= 8 m s–1
(b) Zarah tidak melalui titik tetap O, v = 0, t = 4, vmaksimum = dv = a = 0
dt
s=3m 4 – 12t = 0
t=4 t=0 t= 1
A 3
t= 1
0 3 19 3

6. (a) v , 0 3 ( ) ( )
v=4 1 –6 1 –8
3
2

t2 – 6t – 7 , 0
= –7 1 m s–1
(t + 1)(t – 7) , 0 3
8. (a) v = p + qt2
dv = a = 2qt
dt
t = 4, a = –8
2(q)(4) = –8
t
2 4 q = –1

0<t,7

s = (p – t2) dt
s = pt – t + c
3


(b) s = (t2 – 6t – 7) dt 3
Apabila t = 0, s = 0, c = 0
s = t – 3t2 – 7t + c
3

s = pt – t
3
3
t = 0, s = 0, c = 0 3
t = 6, s = –48
s = t – 3t2 – 7t
3

p(6) – 6 = –48
3
3
v = 0, t2 – 6t – 7 = 0 3
(t + 1)(t – 7) = 0 p = 4
t = –1, t = 7 (b) q = –1, a = –2t
t=7 t = 5, a = –2(5)
= –10 m s–2
s = 7 – 3(7)2 – 7(7)
3

3 (c) Zarah berhenti, v = 0


v = 4 – t2 = 0
= –81 2 m
3 t2 = 4
Tidak melalui B. t = 2 (t > 0)
s = 4t – t
3
7. a = p + qt
3
t = 0, v = –q
t = 2, s = 4(2) – 2
3

t = 0, a = 4 3
=51 m
3

76

Jw01 Full SpotlightA+ MateTambahan Tg5.indd 76 26/10/2022 3:40 PM


Jawapan Matematik Tambahan

(d) Jumlah jarak 5 saat pertama


[kaedah graf halaju masa] t
0 4
v

5
t
0 1 2 3 4 5
–5 t.4
(c) Jarak pada saat ke-6
–10
–15 v
v = 4 – t2 v = –3t 2 + 12t
–20 10
t
0 1 2 3 4 5 6

2
Jarak A = (4 – t ) dt
2 –10
0
–20
[
= 4t – t
3
3
] 2
0 –30

=51 m
3

6
Luas = (–3t2 + 12t) dt

5 5
Jarak B = (4 – t2) dt
2
= –t3 + 6t2 [ ]
6

[ ]
= 4t – t
3
3 5
2
5
= [–63 + 6(6)2] – [–53 + 6(–5)3]
= [4(3) – 3 ] – [4(2) – 2 ]
3 3 = |–25|

3 3 = 25 m
|
= –21 1
3
| (d) 25 m

1
= –21 m Zon Sumatif
3
Jumlah jarak 5 saat pertama Kertas 2
A + B = 5 1 + 21 1 1. (a) (i) t = 0, v = 8 m s–1
3 3 (ii) v , 0
2
= 26 m t2 – 6t + 8 , 0
3
9. v = pt2 + qt
dv = a = 2pt + q
dt
(a) t = 1, a = 6
t
2p(1) + q = 6 2 4
2p + q = 6 …1
Zarah berhenti seketika, 2,t,4
v = 0, t = 4 (iii) a . 0
p(4)2 + q(4) = 0 2t – 6 . 0
÷4 16p + 4q = 0 t . 3
4p + q = 0 …2 (b)
2p + q = 6 …1 y
Selesaikan 1 dan 2
p = –3, q = 12
(b) v = –3t2 + 12t
8
Zarah bergerak ke kiri, v , 0
–3t2 + 12t , 0 v = t 2 – 6t + 8
–3t2 + 12t = 0
–3t(t – 4) = 0 A
t = 0, t = 4 B
x
0 2 4

77

Jw01 Full SpotlightA+ MateTambahan Tg5.indd 77 26/10/2022 3:40 PM


Matematik Tambahan Jawapan

(d) v = 0, 12 + 4t – t2 = 0

2
(c) A = (t2 – 6t + 8) dt (t + 2)(t – 6) = 0
0

= 6 2 t = –2, t = 6
3
v
∫ (t – 6t + 8) dt
4
B= 2
v = 12 + 4t – t 2
2
15
= |–1 1 |
3 10
= 1 1 5
3
Jumlah jarak = 6 2 + 1 1 t
3 3 –2 0 2 4 6
=8m

6
2. (a) v = 2t – 3t2 + 5 Jarak = (12 + 4t – t2) dt
0
t = 0, v = 5 m s–1
= 72 m
(b) v = 0,
2t – 3t2 + 5 = 0 4. a = 8 – 2t
3t2 – 2t – 5 = 0 ∫
(a) v = (8 – 2t) dt
(3t – 5)(t + 1) = 0 = 8t – t2 + c
t= 5 Apabila t = 0, c = 9
3 º v = 8t – t2 + 9
(c) vmak, dv = 0 vmaksimum, dv = 0, a = 0
dt dt
2 – 6t = 0, 8 − 2t = 0
t = 1 t=4
3 Apabila t = 4, v = 8(4) − (4)2 + 9
3 ( ) ( )
vmak = 2 1 – 3 1 + 5
3
2
= 32 − 16 + 9
= 25 m s–1
= 1 – 3 1 + 5
3 9 ( ) (b) v . 0, 8t – t + 9 . 0
2

16 Katakan t2 − 8t − 9 = 0
= m s–1 (t + 1)(t − 9) = 0
3
(d)
v t
–1 9
10
t
0 1 2 3 4
–10
–20
0<t,9

–30 (c)
v = 2t – 3t 2 + 5
–40 v
v = 8t – t 2 + 9
20
5 4
Jarak = ∫ 3
0
(2t – 3t2 + 5) dt + ∫ 5
(2t – 3t2 + 5) dt 10
3 t
0
= 6 13 + 34 13 2 4 6 8
27 27
26

6
= 40 ms –1
Jumlah jarak = (8t – t2 + 9) dt
27 0
3. v = 12 + 4t – t2 = 126 m
(a) t = 0, v = 12 m s–1 5. v = 8 + 10t – 3t 2

(b) dv = 4 – 2t (a) t = 0, v = 8 m s–1


dt (b) v = 0, 8 + 10t – 3t2 = 0
t = 0, a = 4 m s–2 3t2 – 10t – 8 = 0
(c) dv = 0, 4 – 2t = 0 (3t + 2)(t – 4) = 0
dt
t=2 t = – 2, 4 s
3
v = 12 + 4(2) – 22
= 16 m s–1

78

Jw01 Full SpotlightA+ MateTambahan Tg5.indd 78 26/10/2022 3:40 PM


Jawapan Matematik Tambahan

(c) (b) Melalui O semula, s = 0


3t2 – 12t = 0
v
3t(t – 4) = 0
20 v = 8 + 10t – 3t 2 t = 0, t = 4
v = 6t – 12
t
0 2 4 6 = 6(4) – 12
–20 v = 12 m s–1
–40 (c) Jarak maksimum, v = 0
6t – 12 = 0
t = 2
Jumlah jarak s = 3(2)2 – 12(2)
=A+B = 12 – 24
∫ ∫
4 6
= (8 + 10t – 3t2) dt + (8 + 10t – 3t2) dt = –12
0 4
Jarak = 12 m
= 48 + 36
(d) Diberi s = 36
= 84 m
3t2 – 12t = 36
6. a = 2t – 3 ÷3 3t2 – 12t – 36 = 0
(a) 2t – 3 . 0 t2 – 4t – 12 = 0
t . 3 (t + 2)(t – 6) = 0
2
t = –2, t = 6
(b) a = ?, v = 8
v = 6(6) – 12

v = (2t – 3) dt v = 36 – 12
v = t2 – 3t + c, t = 0, v = –10 = 24 m s–1
º v = t2 – 3t – 10 8. s = – t + 9 t2 – 6t
3

v = 8, t2 – 3t – 10 = 8 3 2
t2 – 3t – 18 = 0 (a) t = 1, s = 1 + 9 (1)2 – 6(1)
3

(t + 3)(t – 6) = 0 3 2
t = –3, t = 6 = –1 5 m
6
a = 2(6) – 3
(b) v = –16 m s–1
= 12 – 3 ds = –t2 + 9t – 6
= 9 m s–2 dt
(c) –t2 + 9t – 6 = –16
t2 – 9t – 10 = 0
v (t + 1)(t – 10) = 0
t = –1, t = 10
5
(10)3 9
t t = 10, s = – + (10)2 – 6(10)
0 2 4 6 2 2
–5 = 56 m 2
–10
3
v = t 2 – 3t – 10 (c) Zarah melalui O semula, s = 0
s = 0, – t + 9 t2 – 6t = 0 …1
3

t2 – 3t – 10 = 0 3 2
(t + 2)(t – 5) = 0 1 × 6 2t3 – 27t2 + 36t = 0
Jumlah jarak t(2t2 – 27t + 36) = 0
=A+B t(2t – 3)(t – 12) = 0
t = 0, t = 3 , t= 12

5
= (t) dt t 3
9 2
0 s = – + t2 – 6t
7. a = 6 3 2
(a) v = 6 dt ∫ v = ds = –t2 + 9t – 6
dt
v = 6t + c dv
t = 0, v = –12 = –2t + 9
dt
v = 6t – 12 a = –2t + 9

s = (6t – 12) dt
()
t = 3 , a = –2 3 + 9
s = 3t2 – 12t + c 2 2
= 6 m s–2
t = 0, s = 0, c = 0
t = 12, a = –2(12) + 9
s = 3t2 – 12t
= –15 m s–2
t = 1, s = 3(1)2 – 12(1)
= –9

79

Jw01 Full SpotlightA+ MateTambahan Tg5.indd 79 26/10/2022 3:40 PM


Matematik Tambahan Jawapan

KERTAS MODEL SPM Jadi, ketinggian maksimum yang dicapai


oleh bola golf itu ialah 20 m.
Kertas 1
h(t)
Bahagian A
1. (a) (i) Selanjar
20
(ii) Ya
h(t) = 20t – 5t 2
(b) (i) f –1(x) = a
x
Diberi f –1(2) = 2.
Jadi, a =2
2
a=4
t
(ii) f 17(x) = 4 0 2 4
x
f 17(8) = 4 = 1 4. 2x + 2y + 3z = 22 ……1
8 2
2. (a) (i) – 4 < x < 12 3x – y + 4z = 19 ……2
(ii) B(12, 4) 5x + y + 2z = 21 ……3
(b) 2 × 2: 6x – 2y + 8z = 38 ……4
f(x)
1 + 4: 8x + 11z = 60 ……5
2 + 3: 8x + 6z = 40 ……6
f (x) =│2x – 5│ 5 – 6: 5z = 60
7
z = 20
5
5 =4
Gantikan z = 4 ke dalam 5: 8x + 11(4) = 60
8x + 44 = 60
8x = 60 – 44
= 16
x
0 5
– 6 x = 16
2 8
=2
f(x) < 3 Gantikan x = 2 dan z = 4 ke dalam 1:
|2x – 5| < 3 2(2) + 2y + 3(4) = 22
–3 < 2x – 5 < 3 2y + 16 = 22
2 < 2x < 8 2y = 22 – 16
1 < x < 4 2y = 6
Maka, julat nilai x untuk f(x) < 3 ialah
1<x<4 y= 6
2
3. (a) Diberi persamaan kuadratik ialah =3
x2 + px + q = 0 Jadi, x = 2, y = 3, z = 4.
Jadi, a + b = –p dan ab = q 26 = 26 × 4 – ! 3
5. (a)
Maka, a2b + b2a = ab(a + b) 4 + !3 4 + !3 4 + !3
= q(–p)
= –pq 26(4 – ! 3 )
=
(b) (i) Apabila bola golf menyentuh permukaan 16 – 3
padang, h(t) = 0 104 – 26! 3
=
20t – 5t2 = 0 13
5t(4 – t) = 0 = 8 – 2! 3
t = 0 atau t = 4
(b) 3 log10 (x 3! y2 ) = 2 – log10 x + log10 y
Jadi, masa bola golf menyentuh 2
permukaan padang sekali lagi ialah 4 saat. 3(log10 x + log10 y 3 ) = 2 – log10 x + log10 y
(ii) tmak = – b log10 x3 + log10 y2 = 2 – log10 x + log10 y
2a
= – 20 log10 x3 + log10 y2 + log10 x – log10 y = 2
2(–5)
log10 x 4y = 2
= 2
x 4y = 102
Apabila t = 2, h(2) = 20(2) – 5(2)2
= 40 – 20 x 4y = 100
= 20 y = 100
x4

80

Jw01 Full SpotlightA+ MateTambahan Tg5.indd 80 26/10/2022 3:40 PM


Jawapan Matematik Tambahan

6. (a) Katakan persamaan garis lurus AB ialah (ii) (2 – 1)! × 2! × 2! = 8


Y = mX + c, dengan keadaan m dan c ialah
pemalar, X = x2 dan Y = y – 2x. CDEF
Jadi, mAB = 16 – 7
5–2
=3
Menggantikan X = 2 dan Y = 7 ke dalam
Y = mX + c, 7 = 3(2) + c B A
7=6+c
c=1 9. (a) sin ˙ROT = 6
Persamaan bagi garis lurus AB = Y = 3X + 1 10
Maka, persamaan yang menghubungkan x dan ˙ROT = 36° 52ʹ
y ialah y – 2x = 3x2 + 1 ˙ROS = 36° 52ʹ × 2
y = 3x2 + 2x + 1 = 73° 44ʹ
(b) Menggantikan X = k dan Y = 10 ke dalam = 73° 44ʹ × 3.142
180°
Y = 3X + 1,
= 1.287 radian (Tertunjuk)
10 = 3k + 1
3k = 9 R
k=3
Jadi, nilai k ialah 3.
7. (a) Persamaan garis lurus AC dengan kecerunan 2 10 cm
6 cm
dan melalui titik A(3, 0) ialah
y – 0 = 2(x – 3)
y = 2x – 6
2x – y = 6 O T
2x – y = 6 …1
x + 2y = –2 …2 2π – 2(1.287)
(b) ˙POR =
1 × 2: 4x – 2y = 12 2
2 + 3: 5x = 10 = 1.855 radian
sPQR = 10(1.855)
x = 10 = 18.55 cm
5
=2 Q
Gantikan x = 2 ke dalam 1: 2(2) – y = 6
4 – y = 6!
y=4–6 P R
= –2
Jadi, koordinat bagi titik C ialah (2, –2). 10 cm
3 –2 2 3
(b) Luas ∆ABC = 1 O
2 0 0 –2 0
Jadi, perimeter kepingan logam
= 1 |4 + 6| = 2(18.55) + 2(12)
2
= 37.1 + 24
= 1 |10| = 61.1 cm
2
= 5 unit 2 3 – !x 3 – !x 3 + !x
10. (a) had = ×
8. (a) 2 × 12! × 4 = 39 916 800 x˜9 9–x 9–x 3 + !x
2!4!2!
= 9–x
2 … 4
K A (9 – x)(3 + ! x )
12! = 1
2!4!2! 3 + !x
(b) (i) (5 – 1)! × 2! = 48 = 1
3 + !9
AB
= 1
C
3+3
F = 1
6
D
(b) Fungsi keuntungan, P = 40x – (75 + 2x + 0.1x2)
E = 38x – 75 – 0.1x2

81

Jw01 Full SpotlightA+ MateTambahan Tg5.indd 81 26/10/2022 3:40 PM


Matematik Tambahan Jawapan

Untuk keuntungan maksimum, ˜ ˜


AM = 1 AB
dP = 0 2
dx
38 – 0.2x = 0 = 1 (–a + b )
2 ~ ~
0.2x = 38
= 1 a + 1 b
x = 7 2~ 2~
25 ˜ ˜ ˜
= 190 (ii) AC = AO + OC
Jadi, keuntungan maksimum ialah = –a + 2b
~ ~
38(190) – 75 – 0.1(190)2 = 7 220 – 75 – 3 610 ˜ 2˜
= RM3 535 AN = AC
3
11. (a) sin q kos q = 2
5 = 2 (–a + 2b )
3 ~ ~
2 sin q kos q = 4 2 4
5 =– a+ b
3~ 3~
sin 2q = 4
5 ˜ ˜ ˜
ON = OA + AN
kos 4q = kos [2(2q)]
= a – 2 a + 4 b
= 1 – 2 sin2 (2q) ~ 3~ 3~
= 1 a + 4 b
=1–2 4 2
5 () 3~ 3~
32 ˜ ˜
=1– (b) AB = k CD
25
=– 7
25
3
4( ) ( ) [( ) ( )]

–3
2
=k
10
n

–2
–2
(b) (i) Amplitud, a
= 1 [nilai maksimum – nilai minimum]
2
( ) [( )]
6
2
=k
12
n+2
Jadi, 12k = 6
= 1 [1 – (–3)] k = 1
2 2
= 1 (4) dan 1 (n + 2) = 2
2 2
n + 2 = 4
=2
n=4–2
Kala bagi graf ialah 2π =2
Jadi, 2π = 2π Bahagian B
b
b = 1 13. (a) (i) y = (2x + 5)! 4x – 5
1
dy
= 2(! 4x – 5 ) + 1 (4x – 5) 2 (4)(2x + 5)

Apabila a = 2 dan b = 1, fungsi graf ialah
dx 2
y = 2 kos x + c
2(2x + 5)
Oleh sebab (0, 1) terletak pada graf, = 2(! 4x – 5 ) +
! 4x – 5
1 = 2 kos 0 + c
1 = 2(1) + c 2(4x – 5) + 2(2x + 5)
=
c = –1 ! 4x – 5
Maka, a = 2, b = 1 dan c = –1. 12x
(ii) =
! 4x – 5
y
dy 12x
Jadi, = dengan nilai a ialah 12.
dx ! 4x – 5
3
y =│2 kos x – 1│ 12x
(ii) d [(2x + 5)! 4x – 5 ] =
dx ! 4x – 5
1 d 1 (2x + 5)! 4x – 5 = 12x
dx [ 12 ]
! 4x – 5
x
0 π 2π
Jadi,∫ x
! 4x – 5
1
dx = (2x + 5)! 4x – 5 + c
12
˜ ˜ ˜
12. (a) (i) AB = AO + OB dengan keadaan c ialah pemalar.
= –a + b
~ ~

82

Jw01 Full SpotlightA+ MateTambahan Tg5.indd 82 26/10/2022 3:40 PM


Jawapan Matematik Tambahan

(b) (i) Luas kawasan P = ∫ ( x4 ) dx


2
1 2
(ii) S4 =
[ ( )]
81 – 1
2
4

1– 1
=π –4 [ ]
x 1
2
2


= –2 – (–4)
= 2 unit 2 =
(
81– 1
16)
1
(ii) Luas kawasan Q = luas segi empat tepat – 2
luas kawasan P
=4–2 = 16 ( )
15
16
= 2 unit 2 = 15
14. (a) (i) X ~ B(8, 0.4)
P(X > 2) = 1 – P(X = 0) – P(X = 1) (b) (i) T1 = 1 xy
2
= 1 – 8C 0(0.40)(0.68) – 8C1(0.41)(0.67)
= 1 – 0.0168 – 0.0896 T2 = 1 y(x + 2)
2
= 0.8936
(ii) Varians = 120 = 1 xy + y
2
npq = 120
T3 = 1 y(x + 4)
n(0.4)(0.6) = 120 2
0.24n = 120 = 1 xy + 2y
n = 500 2
Jadi, jumlah penduduk di taman itu ialah Oleh sebab T2 − T1 = T3 − T2 = y, jadi
500 orang. luas ke semua segit tiga bersudut tegak
(b) (i) P(X , k) = 1 – 0.8849 membentuk janjang aritmetik dengan beza
= 0.1151 sepunyanya ialah y.
Jadi, nilai k ialah –1.2. (ii) Apabila x = 4 dan y = 8, a = T1
(ii) z = –1.2
x – 16 = –1.2 = 1 (4)(8)
2
5 = 16
x – 16 = –6
dan d = y = 8
x = –6 + 16
Sn = 616
= 10
n [2(16) + (n – 1)(8)] = 616
Jadi, nilai x ialah 10. 2
15. (a) (i) a(T7) = T4 n (32 + 8n – 8) = 616
a(ar 6) = ar3 2
n (8n + 24) = 616
a2r 6 = ar3
2

a = 13 ……1 8n2 + 24n = 1 232
r
8n + 24n – 1 232 = 0
2
a + T4 = 9
n2 + 3n – 154 = 0
a + ar 3 = 9 ……2
(n + 14)(n – 11) = 0
Gantikan 1 ke dalam 2: n = –14 atau n = 11
1 + 1 (r 3) = 9 Jadi, bilangan segi tiga yang boleh dibina
r3 r3 ialah 11.
13 + 1 = 9
r Kertas 2
1 =8 Bahagian A
r3
1. x ialah usia anak sulung
r3 = 1 y ialah usia anak kedua
8
z ialah usia anak bongsu
r= 1 Ketika ini, usia anak sulung lebih 4 tahun daripada
2
Gantikan r = 1 ke dalam 1: a = 1 jumlah usia adik-adiknya:
2 1 3
2 ( ) x = (y + z) + 4
x – y – z = 4 ……1
= 1
Dua tahun yang lalu, usia anak bongsu ialah 1 dari
1
8 ( ) beza usia abang-abangnya:
4
=8
z – 2 = 1 [(x – 2) – (y – 2)]
Jadi, sebutan pertama ialah 8 dan nisbah 4
sepunya ialah 1 . 4z – 8 = x – y
2 x – y – 4z = –8 ……2

83

Jw01 Full SpotlightA+ MateTambahan Tg5.indd 83 26/10/2022 3:40 PM


Matematik Tambahan Jawapan

Tujuh tahun yang akan datang, jumlah usia mereka (b) (i) h = 3 dan 2k = –2
adalah 51 tahun: k = –1
(x + 7)(y + 7) + (z + 7) = 51 (ii) B(6, 2)
x + y + z + 21 = 51 (iii) f(x) = (x – 6)2 + 2

1 – 2:
x + y + z = 30 ……3
3z = 12 ( 2 2 )
4. (a) (i) M = 3 + 5 , 5 + (–3) = (4, 1)
–1 + (–3)
1 + 3:
z=4
2x = 34
N= (–3
2
+ 5 ,
2 )
= (1, –2)
x = 17 1 – (–2)
mMN = =1
Gantikan x = 17 dan z = 4 ke dalam 1: 4–1
17 – y – 4 = 4 Persamaan garis lurus MN dengan
13 – y = 4 kecerunan 1 dan melalui titik M(4, 1)
y = 13 – 4 ialah y – 1 = 1(x – 4)
=9 y=x–3
Jadi, usia anak sulung ialah 17 tahun, usia anak kedua Pada paksi-x, y = 0
ialah 9 tahun dan usia anak bongsu ialah 4 tahun. Jadi, 0=x–3
2. (a) f(3) = 3 x=3
27 – a = 3 Maka, koordinat bagi titik P ialah (3, 0).
a = 27 – 3 (ii) m + 4n = 3
= 24 m+n
g(3) = 4 m + 4n = 3m + 3n
b =4 2m = n

9 m = 1
b = 36 n 2
Jadi, nilai a ialah 24 dan nilai b ialah 36. M(4, 1)
(b) gf(x) = g[f(x)] m

= 36
P(3, 0)
12 – (9x – 24)
= 36 n
36 – 9x
= 36
9(4 – x) N(1, –2)
= 4
4–x
Jadi, MP : PN = 1 : 2.
Jadi, gf : x ˜ 4 , x ≠ 4. (iii) Luas trapezium ACNM
4–x
(c) gf(x) = x 3 –3 1 4 3
= 1
4 =x 2 5 –1 –2 1 5
4–x
4 = 4x – x2 = 1 |–3 + 6 + 1 + 20 – (–15 – 1 – 8 + 3)|
2
x – 4x + 4 = 0
2

(x – 2)(x – 2) = 0 = 1 |24 – (–21)|


2
x=2
3. (a) 3x2 – 2kx + k + 4 = 0 = 1 |45|
2
a + b = 2k dan ab = k + 4 = 22 1 unit 2
3 3 2
Diberi a2 + b2 = 16 (b) dAQ = 5
9
Jadi, (a + b) – 2ab =
2 16 ! (x – 3)2 – (y – 5)2 = 5
9
x2 – 6x + 9 + y2 – 10y + 25 = 25
2k
3 ( ) (
2
–2 k +
3
4 = 16
9 ) x 2 + y2 – 6x – 10y + 9 = 0
Jadi, persamaan lokus bagi Q ialah
(
4k – 2k + 8 = 16 )
2

9 3 9 x 2 + y2 – 6x – 10y + 9 = 0.
4k2 – 6k – 24 = 16 5. (a) sin ˙SOR = 3
5
4k2 – 6k – 40 = 0 ˙SOR = 36° 52ʹ
2k2 – 3k – 20 = 0
(2k + 5)(k – 4) = 0 = 36° 52ʹ × 3.142
180°
k = – 5 atau k = 4 = 0.644 radian
2

84

Jw01 Full SpotlightA+ MateTambahan Tg5.indd 84 26/10/2022 3:40 PM


Jawapan Matematik Tambahan

Jadi, ˙POQ = 0.644 radian Maka, perubahan perentas dalam V


S = dV × 100
V
= 0.2πr 3
× 100
5 cm 4 πr 3
3 cm 3
= 15%
7. (a) 6 tan2 q + 13 sek q = 2
O R 6(sek 2 q – 1) + 13 sek q – 2 = 0
6 sek 2 q – 6 + 13 sek q – 2 = 0
(b) SPQ = 10(0.644) 6 sek 2 q + 13 sek q – 8 = 0
= 6.44 cm (3 sek q + 8)(2 sek q – 1) = 0
SST = 3 π
2( ) sek q = – 8 atau sek q = 1
3 2
= 4.713 cm (Abaikan)
Jadi, perimeter berlorek
kos q = – 3
= 6.44 + 4.713 + 5 + (10 – 3 – 4) 8
= 19.153 cm q = 180° – 67° 59ʹ, 180° + 67° 59ʹ
(c) Luas sektor POQ = 1 (10)2(0.644) = 112° 1ʹ, 247° 59ʹ
2 (b) (i) Amplitud, a
= 32.2 cm2
= 1 [nilai maksimum – nilai minimum]
2
Luas ∆SOR = 1 (4)(3)
2 = 1 [1 – (–5)]
= 6 cm2 2
= 1 (6)
2 ( )
Luas sektor SRT = 1 (3)2 π
2 2
= 7.07 cm2 =3
Jadi, luas kawasan berlorek = 32.2 – 6 – 7.07 Kala bagi graf ialah 4π
= 19.13 cm2
6. (a) z2 = x2 + 32 Jadi, 2π = 4π
z = ! x2 + 9

1
b ( )
1
dz = 1 (x2 + 9)– 2 (2x) 2πb = 4π
dt 2 b=2
= x
Apabila a = 3 dan b = 2, fungsi graf ialah
!x + 9
2
y = 3 sin 1 x + c.
4 2
Apabila x = 4, dz =
dt ! 42 + 9 Oleh sebab (0, –2) terletak pada graf,
= 4 –2 = 3 sin 1 (0) + c
5 2
–2 = 0 + c
c = –2
Jadi, dz = dz × dx
dt dx dt Maka, a = 3, b = 2 dan c = –2.
4
–2 = × dx (ii)
5 dt y
dx = –2 × 5
dt 4 y =│3 sin 1
– x – 2│
2
5 5
=–
2
Maka, jarak serenjang bot dari dermaga
menyusut sebanyak 2.5 m s–1. 2
1
(b) Diberi V = 4 πr 3 x
3 0 π 2π 3π 4π
Jadi, dV = 4πr 2
dr
dan dV = dV × dr
dr
= 4πr 2­× 0.05r
= 0.2πr 3

85

Jw01 Full SpotlightA+ MateTambahan Tg5.indd 85 26/10/2022 3:40 PM


Matematik Tambahan Jawapan

Bahagian B 10. (a) y = x2 + 1


8. (a) dy
= 2x
dx
log10 y dy
Apabila x = 1, = 2(1)
dx
0.8 =2
mAQ = – 1
0.7 2
–2 = – 1
0.6 x–1 2
0.5
– 4 = 1 – x
x = 5
0.4 Jadi, koordinat titik Q ialah (5, 0).
(b) Luas kawasan berlorek
0.3

= (x2 + 1) dx + 1 (5 – 1)(2)
1

0.2
0 2

0.1
= [1
3
3
1
0
]
1
x + x + (4)(2)
2

0 0.1 0.2 0.3 0.4 0.5 0.6 0.7 0.8 0.9


log10 x
= (1
3 )
+1 +4

= 5 1 unit 2
(b) y = axn 3
log10 y = log10 a + log10 xn

2
(c) Isi padu kisaran = π x2 dy
log10 y = n log10 x + log10 a 1

= π∫
2
Ini adalah dalam bentuk Y = mX + c, dengan (y – 1) dy
X = log10 x, Y = log10 y, m = n dan c = log10 a 1

Daripada graf di (a)


(i) Pintasan-log10 y = 0.11
[
= π 1 y2 – y
2 ]2
1

log10 a = 0.11 = π[(2 – 2) – ( 1 – 1)]


a = 10 0.11 2
= 1.288 = 1 π unit3
2
(ii) Kecerunan, n = 0.56 – 0.11 ˜ ˜
0.60 – 0 11. (a) (i) AM = 1 AB
= 0.75 2
9. (a) X ~ B(9, p) = 1 (–a + b )
2 ~ ~
(i) m = 5.4
9p = 5.4 =– 1a + 1b
2~ 2~
p = 0.6 ˜ ˜ ˜
(ii) P(X > 3) (ii) AP = AO + OP
= 1 – P(X = 0) – P(X = 1) – P(X = 2)
= –a + 1 b
= 1 – 9C 0(0.60)(0.49) – 9C1(0.61)(0.48) – ~ 3~
9
C2(0.62)(0.47) ˜ ˜ ˜
= 1 – 0.000262 – 0.003539 – 0.021234 (iii) OM = OA + AM
= 0.9750 = a – 1 a + 1 b
(b) X ~ N(2.4, 0.82) ~ 2~ 2~
(i) P(X . 2.5) = P Z . 2.5 – 2.4(
0.8 ) = 1 a + 1 b
2~ 2~
= P(Z . 0.125) ˜ ˜
= 0.4502 (b) OQ = h OM
(ii) m – 2.4 = –0.842
0.8 (
= h 1 a + 1 b
2~ 2~ )
m – 24 = –0.6736
= ha + 1 hb
1
m = 23.33 kg 2 ~ 2 ~
f(z) ˜ ˜
AQ = k AP

0.2 0.8 ~ 3~ (
= k –a + 1 b )
= –ka + 1 kb
~ 3 ~
z
–0.842 0

86

Jw01 Full SpotlightA+ MateTambahan Tg5.indd 86 26/10/2022 3:40 PM


Jawapan Matematik Tambahan

˜ ˜ ˜ (d) Di K, v=0
AQ = AO + OQ 12 – 6t = 0
–ka + kb = –a + 1 ha + 1 hb
1 6t = 12
~ 3 ~ ~ 2 ~ 2 ~ t=2
~ 3 ~ 2 (
–ka + kb = 1 h – 1 a + 1 hb
1
)
~ 2 ~
Apabila t = 2, s = 12(2) – 3(2)2
= 24 – 12
1
Jadi, h – 1 = –k = 12 m
2 Jadi, jumlah jarak yang dilalui oleh zarah dari
1
h + k = 1 O ke J melalui K = 12 + 12 + 15
2
= 39 m
h + 2k = 2 ……1
QR
1k = 1h 13. (a) (i) = 6
dan sin 20° sin 40°
3 2
QR = 6 × sin 20°
k = 3h sin 40°
2
= 3.193 cm
3
Gantikan k = h ke dalam 1:
2 (ii) ST 2 = 52 + 82 – 2(5)(8) kos 120°
ST = ! 129

2 ( )
h + 2 3h = 2
= 11.358 cm
4h = 2 (b) (i)
h= 1 P P
2
20°
Apabila h = , k = 3 h
1
60°
2 2
6 cm
= 3 1( )( )
2 2 6 cm

= 3
4 40°
Q R R R'
3.193 cm
Maka, h = 1 dan k = 3 .
2 4
Bahagian C (ii) Luas ∆PQR = 1 (3.193)(6)(sin 120°)
2
12. (a) v = 12 – 6t = 8.296 cm­2
a = dv Luas ∆PRRʹ = 1 (6)(6)(sin 60°)
dt 2
= –6 = 15.588 cm­2
Jadi, pecutan malar zarah ialah –6 m s–2.
Luas ∆PRʹQ = 8.296 + 15.588

(b) s = v dt
= 23.884 cm2
= ∫ (12 – 6t) dt
14. (a) (i) Untuk barang B,
= 12t – 3t2 + c I12/10 = 140
8.40 × 100 = 140
Apabila t = 0 dan s = 0, jadi c = 0. Oleh itu,
P10
pada masa t, s = 12t − 3t2
P10 = 8.40 × 100
Apabila zarah melalui O, s = 0 140
12t − 3t2 = 0 = RM6
3t(4 − t) = 0 (ii) Untuk barang D,
t = 0 atau t = 4 I12/10 = 130
Maka, zarah melalui O sekali lagi pada masa P12
× 100 = 130
4 saat. 4.50
(c) Di J, s = –15 m
P12 = 130 × 4.50
12t − 3t2 = –15 100
3t2 – 12t – 15 = 0 = RM5.85
t2 – 4t – 5 = 0 (b) I 12/10 = 132
(t + 1)(t – 5) = 0
112 + 140(4) + 2k + 130(3)
t = –1 atau t = 5 = 132
Apabila t = 5, v = 12 – 6(5) 10
2k + 1 062 = 1 320
= 12 – 30
2k = 258
= –18
k = 129
Jadi, halaju zarah di J ialah –18 m s–1.

87

Jw01 Full SpotlightA+ MateTambahan Tg5.indd 87 26/10/2022 3:40 PM


Matematik Tambahan Jawapan

112 + 126(4) + 129(2) + 136.5(3) (c) (i) 25 orang


(c) I 14/10 =
10 (ii) Titik optimum dalam rantau berlorek R
= 128.35 × 20 ialah (60, 120).
100 Jadi, k = 500(60) + 800(120)
= 128.35 = 30 000 + 96 000
P = 126 000
(d) 14 × 100 = 128.35
20 Maka, jumlah yuran maksimum sebulan
P14 = 128.35 × 20 yang boleh dikutip ialah RM126 000.
100
= RM25.67
15. (a) I: x + y < 180
II: y < 2x
III: y > 1 x
2
(b) Rujuk graf
y

200 y < 2x

160

120
x + y < 180
80 y>1
–x
2
40

x
0 40 80 120 160 200

88

Jw01 Full SpotlightA+ MateTambahan Tg5.indd 88 26/10/2022 3:40 PM

You might also like